You are on page 1of 335

Module 1 : A Crash Course in Vectors

Lecture 1 : Scalar And Vector Fields


Objectives
In this lecture you will learn the following
Learn about the concept of field
Know the difference between a scalar field and a vector field.
Review your knowledge of vector algebra
Learn how an area can be looked upon as a vector
Define position vector and study its transformation properties under rotation.
SCALAR AND VECTOR FIELDS
This introductory chapter is a review of mathematical concepts required for the course. It is assumed that the
reader is already familiar with elementary vector analysis.
Physical quantities that we deal with in electromagnetism can be scalars or vectors.
A scalar is an entity which only has a magnitude. Examples of scalars are mass, time, distance, electric charge,
electric potential, energy, temperature etc.
A vector is characterized by both magnitude and direction. Examples of vectors in physics are displacement,
velocity, acceleration, force, electric field, magnetic field etc.
A field is a quantity which can be specified everywhere in space as a function of position. The quantity that is
specified may be a scalar or a vector. For instance, we can specify the temperature at every point in a room.
The room may, therefore, be said to be a region of ``temperature field" which is a scalar field because the
is a scalar function of the position. An example of a scalar field in electromagnetism

temperature
is the electric potential.

In a similar manner, a vector quantity which can be specified at every point in a region of space is a vector
field. For instance, every point on the earth may be considered to be in the gravitational force field of the earth.
we may specify the field by the magnitude and the direction of acceleration due to gravity (i.e. force per unit
mass )

at every point in space. As another example consider flow of water in a pipe. At each point

in the pipe, the water molecule has a velocity

. The water in the pipe may be said to be in a

velocity field. There are several examples of vector field in electromagnetism, e.g., the electric field
magnetic flux density

, the

etc.

Elementary Vector Algebra :


Geometrically a vector is represented by a directed line segment. Since a vector remains unchanged if it is
shifted parallel to itself, it does not have any position information.
A three dimensional vector can be specified by an ordered set of three numbers, called its components . The
magnitude of the components depend on the coordinate system used. In electromagnetism we usually use
cartesian, spherical or cylindrical coordinate systems. (Specifying a vector by its components has the advantage
that one can extend easily to n dimensions. For our purpose, however, 3 dimension would suffice.)
A vector

is represented by

vector is given by

in cartesian (rectangular) coordinates. The magnitude of the

Click here for Animation


A unit vector in any direction has a magnitude (length) 1. The unit vectors parallel to the cartesian
coordinates are usually designated by

and

and

respectively. In terms of these unit vectors, the vector

is written

Any vector in 3 dimension may be written in this fashion. The vectors

are said to form a basis . In fact,

any three non-colinear vectors may be used as a basis. The basis vectors used here are perpendicular to one
another. A unit vector along the direction of

is

Vector Addition
Sum of two vectors

and

is a third vector. If

then

Geometrically, the vector addition is represented by parallelogram law or the triangle law, illustrated below.

Scalar Multiplication
The effect of multiplyimg a vector by a real number

is to multiply its magnitude by

without a change in

direction (except where


is negative, in which case the vector gets inverted). In the component
representation, each component gets multiplied by the scalar

Scalar multiplication is distributive in addition, i.e.

Two vectors may be multiplied to give either a scalar or a vector.


Scalar Product (The Dot products)
The dot product of two vectors

and

times the cosine of the angle (

is a scalar given by the product of the magnitudes of the vectors


) between the two

In terms of the components of the vectors

Note that
Dot product is commutative and distributive

Two vectors are orthogonal if

Dot products of the cartesian basis vectors are as follows

Exercise 1

Show that the vectors

and

are orthogonal.

Vector Product (The Cross Product)


The cross product of two vectors

is a vector whose magnitude is

, where

angle between the two vectors. The direction of the product vector is perpendicular to both
however, does not uniquely determine
perpendicular. The direction of

and

is the
. This,

as there are two opposite directions which are so

is fixed by a convention, called the Right Hand Rule.

Right Hand Rule :


Stretch out the fingers of the right hand so that the thumb becomes perpendicular to both the index (fore
finger) and the middle finger. If the index points in the direction of
then,

and the middle finger in the direction of

points in the direction of the thumb. The rule is also occasionally called the Right handed

cork screw rule which may be stated as follows. If a right handed screw is turned in the direction from
, the direction in which the head of the screw proceeds gives the direction of the cross product.
In cartesian basis the cross product may be written in terms of the components of

The following points may be noted :


Vector product is anti-commutative, i.e.,

Two vectors are parallel if their cross product is zero.


Vector product is distributive

and

as follows.

to

The cross product of cartesian basis vectors are as follows

and

Exercise 2

and

Find a unit vector which is perpendicular to the plane containing the vectors

[Ans.

Exercise 3

Vector

and

. Find the values of

and

such that the vectors

are parallel
[Ans.

.]

Area as a Vector Quantity


The magnitude of the vector also happens to be the area of the parallelogram formed by the vectors

and

. The fact that a direction could be uniquely associated with a cross product whose magnitude is equal to an
area enables us to associate a vector with an area element. The direction of the area element is taken to be the
outward normal to the area. (This assumes that we are dealing with one sided surfaces and not two sided ones
like a Mbius strip.
For an arbitrary area one has to split the area into small area elements and sum (integrate) over such
elemental area vectors

A closed surface has zero surface area because corresponding to an area element
which is oppositely directed.

, there is an area element

Click here for Animation


Scalar and Vector Triple Products
One can form scalars and vectors from multiple vectors. Scalar and vector triple products are often useful.
The scalar triple product of vectors

and

is defined by

Note that the scalar triple product is the same for any cyclic permutation of the three vectors

and

. In

terms of the cartesian components, the product can bew written as the determinant

Since

gives the area of a parallelogram of sides

the volume of a parallelopiped of sides


The vector triple product of

and

and

and

, the triple product

is defined by

. Since cross product of two

vectors is not commutative, it is important to identify which product in the combination comes first. Thus
is not the same as

Click here for Animation


Example 1

gives

Express an arbitray vector

as a linear combination of three non-coplanar vectors

and

Solution :
Let

. Since the cross product

with both vectors is zero. Taking the dot product of

is perpendicular to both

with

and

, its dot product

, we have

which gives

The coefficients

and

may be found in a similar fashion.

Exercise 4

Prove the following vector identity which is very useful and often used

For ease of remembering this formula is often known as bac-cab formula.

Position Vector and its Transformation under Rotation


Though a general vector is independent of the choice of origin from which the vector is drawn, one defines a
vector representing the position of a particle by drawing a vector from the chosen origin O to the position of the
particle. Such a vector is called the position vector . As the particle moves, the position vector also changes in
magnitude or direction or both in magnitude and direction. Note, however, though the position vector itself
depends on the choice of origin, the displacement of the particle is a vector which does not depend on the
choice of origin.
In terms of cartesian coordinates of the point , the position vector is

We will now derive the relationship between the

and the corresponding values

in a

coordinate system which is rotated with respect to the earlier coordinate system about an axis passing through
the origin. For simplicity consider the axis of rotation to be the z-axis so that the
change.

coordinate does not

Click here for Animation


In the figure P is the foot of the perpendicular drawn from the tip of the position vector on to the x-y plane.
.

Since the axis of rotation coincides with the z-axis, the z coordinate does not change and we have
The figure shows various angles to be equal to the angle of rotation
easily see

by use of simple geometry. One can

Since any vector can be parallely shifted to the origin, its tranformation properties are identical to the
transformation properties of the position vector. Thus under rotation of coordinate system by an angle
the z-axis the components of a vector

about

transform as follows :

Exercise 5
Show that the cross product of vectors satisfy the transformation property stated above.
Recap
In this lecture you have learnt the following
A field is a quantity that can be specified at every point in a certain region of space. A field ay be a scalar
or a vector
An area element can be regarded as a vector because in addition to having a magnitude a direction can
be associated with it. The direction is conventionally chosen as the outward normal to the area element.

Expressions for scalar and vector triple product were obtained.


While a vector like displacement does not depend on the origin of coordinate system, position vector
depends on the origin. The transformation properties of a position vector under rotation of coordinate
system was studied.

Module 1 : A Crash Course in Vectors


Lecture 2 : Coordinate Systems
Objectives
In this lecture you will learn the following
Define different coordinate systems like spherical polar and cylindrical coordinates

How to transform from one coordinate system to another and define Jacobian

Coordinate Systems :
We are familiar with cartesian coordinate system. For systems exhibiting cylindrical or spherical symmetry, it is
convenient to use respectively the cylindrical and spherical coordinate systems.
Polar Coordinates :

In two dimensions one defines the polar coordinate


the origin

and

of a point by defining

as the radial distance from

as the angle made by the radial vector with a reference line (usually chosen to coincide with

the x-axis of the cartesian system). The radial unit vector


taken respectively along the direction of increasing distance

and the tangential (or angular) unit vector


and that of increasing angle

shown in the figure.

Click here for Animation


Relationship with the cartesian components are

are

respectively, as

so that the inverse relationships are

By definition, the distance

. we will take the range of angles

to be

(It is possible to

). One has to be careful in using the inverse tangent as the arc-tan

define the range to be


. If

function is defined in

is negative, one has to add

to the principal value of

calculated by

the arc - tan function so that the point is in proper quadrant.


Example 2
A vector

has cartesian components

and

. Write the vector in terms of its radial and tangential

components.
Solution :
Let us write

Since

and

are basis vectors

and

Note that (see figure) the angle that


Similarly, the unit vector

makes

makes an angle

. Thus

with the x-axis (

with the x-axis and

) and

with the y-axis (

).

with the y-axis. Thus

The Jacobian :
When we transform from one coordinate system to another, the differential element also transform.
For instance, in 2 dimension the element of an area is
but
system.

. This extra factor

but in polar coordinates the element is not

is important when we wish to integrate a function using a different coordinate

If

is a function of

we may express the function in polar coordinates and write it as

in polar coordinates, the corresponding integral is

However, when we evaluate the integral


. In general, if

and
where

differential element

, then, in going from

to

, the

is given by the determinant

The differentiations are partial, i.e., while differentiating


constant. An useful fact is that the Jacobian of the inverse transformation is

, the variable

is treated as

because the detrminant of the

inverse of a matrix is equal to the inverse of the determinant of the original matrix.
Example 3
Show that the Jacobian of the transformation from cartesian to polar coordinates is

Solution :
We have

Using

and

, we have

Exercise 1

Show that the Jacobian of the inverse transformation from polar to cartesian is

Example 4
Find the area of a circle of radius

Solution :
Take the origin to be at the centre of the circle and the plane of the circle to be the
element in the polar coordinates is

a very well known result !


Example 5

, the area of the circle is

plane. Since the area

Find the integral

where the region of integration is a unit circle about the origin.

Using polar coordinates the integrand becomes


is from

to

. The range of i integration for

is from

to

and for

. The integral is given by

The radial integral is evaluated by substitution

so that

. The value of the integral is

Exercise 2
Evaluate

where the region of integration is the part of the area between circles of radii 1 and 2

that lies in the first quadrant.

(Ans.

Exercise 3
Evaluate the Gaussian integral
Hint : The integration cannot be done using cartesian coordinates but is relatively easy using polar coordinates
and properties of definite integrals. By changing the dummy variable

to

, one can write

so that we can write

Transform this to polar. Range of integration for

is from

to

and that of

is from

to

(why ?) ]

[Answer :

Differentiation of polar unit vectors with respect to time :


It may be noted that the basis vectors

and

, unlike

and

are not constant vectors but depend on the

position of the point. The time derivative of the unit vectors are defined as follows

One can evaluate the derivatives by laborious process of expressing the unit vectors

and

in terms of

constant unit vectors of cartesian system, differentiating the resulting expressions and finally transform back to
the polar form. Alternatively, we can look at the problem geometrically, as shown in the following figure.

In the figure, the positions of a particle are shown at time


while the unit vector
magnitude of

and

. The unit vectors

is shown in red

is shown in blue. It can be easily seen by triangle law of addition of vectors that the

and

is

. However, as the limit

direction of

while that of

Now,

is the angular velocity of the point, which is usually denoted by

is in the direction of

, the direction of

is in the

. Thus

, Thus we have,

Cylindrical coordinates :
Cylnidrical coordinate system is obtained by extending the polar coordinates by adding a z-axis along the height
of a right circular cylinder. The z-axis of the coordinate system is same as that in a cartesian system.
In the figure
Note that

is the distance of the foot of the perpendicular drawn from the point to the

plane.

here is not the distance of the point P from the origin, as is the case in polar coordinate systems.

(Some texts use

to denote what we are calling as

here. However, we use

to denote the distance from the

origin to the foot of the perpendicular to avoid confusion.) In terms of cartesian coordinates

so that the inverse relationships are

Click here for Animation


Exercise 4
Find the cylindrical coordinate of the point

[Hint : Determine

and

(Ans.

using above transformation]

The line element in the system is given by

and the volume element is

The Jacobian of transformation from cartesian to cylindrical is

as in the polar coordinates since

coordinate

remains the same.


Spherical Polar Coordinates :
Spherical coordinates are useful in dealing with problems which possess spherical symmetry. The independent
variables of the system are

. Here

is the distance of the point

from the origin. Angles

and

are similar to latitudes and longitudes.


Two mutually perpendicular lines are chosen, taken to coincide with the x-axis and z-axis of the cartesian system.
We take angle
z-axis (the angle

to be the angle made by the radius vector (i.e. the vector connecting the origin to P) with the
is actually complementary to the latitude). The angle

the line joining the origin to


The unit vectors

and

The surface of constant


angle

, the foot of the perpendicular from

to the x-y plane.

are respectively along the directions of increasing

are spheres of radius

and

about the centre. the surface of constant

about the z-axis. The reference for measuring

constant

is the angle between the x-axis and

is a cone of semi-

is the x-z plane of the cartesian system. A surface of

is a plane containing the z-axis which makes an angle

with the reference plane.

Example 6
Express unit vectors of spherical coordinate system in terms of unit vectors of cartesian system.
Solution :
From the point P drop a perpendicular on to the x-y plane. Denote

by

. The figure below shows the unit

vectors in both the systems. By triangle law of vector addition,

However,

. Substituting this in the expression for

the magnitude of

Transformation from spherical to cartesian :


Using the expression for

in terms of cartesian basis, it is seen that

, we get on dividing both sides by

and the inverse transformation

Range of the variables are as follows :

Exercise 5
A particle moves along a spherical helix. its position coordinate at time

is given by

Express the equation of the path in spherical coordinates.

(Ans.

The differential element of volume is obtained by constructing a closed volume by extending


respectively by
along

is

and

. The length elements in the direction of

(see figure). The volume element, therefore, is

Thus the Jacobian of transformation is

is dr, that along

and
while that

Click here for Animation

Example 7
Find the volume of a solid region in the first octant that is bounded from above by the sphere
and from below by the cone

Solution :
Because of obvious spherical symmetry, the problem is best solved in spherical polar coordinates. The equation to
sphere is

so that the range of

vaiable for our solid is from

i.e.

range of the azimuthal angle

becomes

The equation to the cone


cone is

to

. Solving, the semi-angle of

. Since the solid is restricted to the first octant, i.e., (


is from

to

), the

Exercise 7
Using direct integration find the volume of the first octant bounded by a sphere
Recap
In this lecture you have learnt the following
In addition to the cartesian coordinates, two other coordinate systems, viz., spherical polar coordinates
and cylindrical coordinates were introduced. The relationship of the components of a vector in various
coordinates was studied.
When we transform from one coordinate to another, length, area and volume elements also change.
Jacobian provides the transformation of such elements in different coordinate systems.
While the unit vectors in cartesian coordinates are fixed, the unit vectors associated with the position of a
moving particle changes as the particle moves, and are therefore, time dependent.
The differentiation of such time dependent unit vectors with respect to time was discussed.

Integration techniques to find out volumes of objects having different types of symmetry was studied.

Module 1 : A Crash Course in Vectors


Lecture 3 : Line and Surface Integrals of a Vector Field
Objectives
In this lecture you will learn the following
Line, surface and volume integrals and evaluate these for different geometries.
Evaluate flux
Line and Surface Integrals of a Vector Field
Since a vector field is defined at every position in a region of space, like a scalar function it can be integrated and
differentiated. However, as a vector field has both magnitude and direction it is necessary to define operations of
calculus to take care of both these aspects.
Line Integral :
If a vector field
be defined as

where

is known in a certain region of space, one can define a line integral of the vector function may

is the curve along which the integral is calculated. Like the integral of a scalar function the integral above

is also interpreted as a limit of a sum. We first divide the curve


into a large number of infinitisimally small line
segments such that the vector function is constant (in magnitude and direction) over each such line segment. The
integrand is then equal to the product of the length of the line segment and the component of
along the
segment. The integration is defined as the limit of the sum of contributions from all such segments in the same
manner as ordinary integration is defined.

Click here for Animation


The concept of line integral is very useful in many branches of physics. In mechanics, we define work done by a
force
in moving an object from an initial position A to a final position B as the line integral of the force along the
curve joining the end points. Except in the case of conservative forces, the line integral depends on the actual path
along which the particle moves under the force.

Example 8
A force

acts on a particle that travels from the origin to the point

. Calculate

the work done if the particle travels (i) along the path

along

straightline segments joining each pair of end points (ii) along the straightline joining the initial and final points.
Solution :
Along the path
the integral along

,
is zero. Along the path

and
joining

. Since
and

along this segment,


,

and

Along this path

. The integral

Along the third path connecting

to

and

The line integral is

The work done is, therefore,

Click here for Animation


In order to calculate the work done when the particle moves along the straightline connecting the initial and final
points, we need to write down the equation to the line in a parametric form. If

and

are the end points, the equation is

Substituting for the coordinates, we get the equation to the line as

Thus the differential elements are

and

. The line integral is given by

Example 9
Find the line integral of

over an anticlockwise circular loop of radius

with the origin as the centre

of the circle.
Solution :
The length element

has a magnitude

with the positive

. Since the unit vector along

makes an angle of

axis,

Click here for Animation


In polar coordinates,

and

(since the radius is 1). Thus

Exercise 1
A force
from

acts on a particle. Calculate the work done if the particle is taken


the

point

to

the

point

along

straight

line

segment

connecting

. What would be the work done if the particle directly moved


to the final point along the straightline connecting to origin.
,

(Ans.

.)

Exercise 2
A vector field is given by
Evaluate the line integral of the field around a circle of unit radius traversed in clockwise fashion.
(Ans.

Exercise 3
along a parabolic path

connecting the origin to the point

[Hint : Remember that the arc length along a curve is given by

. The curve can be

Evaluate the line integral of a scalar function


.

parametrized by

and

.] [Ans.

Surface Integral :
We have seen that an area element can be regarded as a vector with its direction being defined as the outward
normal to the surface. The concept of a surface integral is related to flow. Suppose the vector field represents the
rate at which water flows at a point in the region of flow. The flow may be measured in cubic meter of water
flowing per square meter of area per second. If an area is oriented perpendicular to the direction of flow, as shown
in the figure to the left, maximum amount of water would flow through the surface. The amount of water passing
through the area is the flux (measured in cubic meter per second).

If, on the other hand, the surface is tilted relative to the flow, as shown to the right, the amount of flux through
the area decreases. Clearly, only the part of the area that is perpendicular to the direction of flow will contribute to
the flux.
We define flux through an area element

as the dot product of the vector field

with the area vector

When
is parallel to
, i.e. if the surface is oriented perpendicular to the direction of flow, the flux is
maximum. On the other hand, a surface oriented parallel to the flow does not contribute to the flux.

Example 10
A hemispherical bowl of radius

is oriented such that the circular base is perpendicular to direction of flow.

Calculate the flux through the curved surface of the bowl, assumuing the flow vector

to be constant.

Solution :
Since the curved surface makes different angles at different positions, it is somewhat difficult to calculate the flux
through it. However, one can circumvent it by calculating the flux through the circular base.

Click here for Animation


Since the flow vector is constant all over the circular face which is oriented perpendicular to the direction of flow,
the flux through the base is

. The minus sign is a result of the fact that the direction of the surface is

opposite to the direction of flow. Thus we may call the flux through the base as inward flux . Since there is no
source or sink of flow field (i.e. there is no accumulation of water) inside the hemisphere, whatever fluid enters
through the base must leave through the curved face. Thus the outward flux from the curved face is

We may now generalize the above for a surface over which the field is not uniform by defining the flux through
area as the sum of contribution to the flux from infinitisimal area elements which comprises the total area
treating the field to be uniform over such area elements. Since the flux is a scalar, the surface integral, defined
the limit of the sum, is also a scalar.
We may now generalize the above for a surface over which the field is not uniform by defining the flux through
area as the sum of contribution to the flux from infinitisimal area elements which comprises the total area
treating the field to be uniform over such area elements. Since the flux is a scalar, the surface integral, defined
the limit of the sum, is also a scalar.

an
by
as

If
angle

is the

th surface element, the normal

with the vector field

to which makes an

at the position of the element, the total

flux

In the limit of

, the sum above becomes a surface integral

an
by
as

Click here for Animation


If the surface is closed, it encloses a volume and we define

to be the net outward flux. In terms of cartesian components

Example 11
A vector field is given by

. Evaluate the flux through each face of a unit cube whose

edges along the cartesian axes and one of the corners is at the origin.
Solution :
Consider the base of the cube (OGCD), which is the x-y plane on which
surface vector
since

is along

. On this face

direction. Thus on this surface flux

. For the top surface (ABFE),

and

. The flux from this surface is

. The

Click here for Animation


In a similar way one can show that flux from left side (AEOD) is zero while the contribution from the right side
(BFGC) is 1/2. The back face (EFGO) contributes zero while the front face (ABCD) contributes 1/2. The net flux,
therefore, is 3/2.
Exercise 4
through a rectangular surface in the x-y plane having

Find the flux of the vector field


dimensions

. The origin of the coordinate system is at one of the corners of the rectangle and the x-axis

along its length.


(Ans.

Click here for Animation


Example 12
Calculate directly the flux through the curved surface of the hemispherical bowl of Example 10.
Solution :
Use a spherical polar coordinates with the base of the hemisphere being the x-y plane and the direction of the
vector field as the z-axis. We have seen that an area element on the surface is given by
. Thus the flux

is given by

Click here for Animation


Exercise 5
Find the flux through a hemispherical bowl with its base on the x-y plane and the origin at the centre of the
base. The vector field, in spherical polar coordinates is

(Ans.

Example 13
A cylindrical object occupies a volume defined by

and

each of the surfaces when the object is in a vector field

. Find the flux through


.

Solution :
Because of cylindrical symmetry, it is convenient to work in a cylindrical

coordinates. The vector field

is given by

Bottom face has

and the normal to the face points in

direction. Thus the flux from this face

The top face has

and

. The flux is

The normal to the curved face is along

direction. An area element on the curved face is

flux from this face is

The net flux from the faces of the object is

Thus the

Exercise 6
Find the flux of the vector field
height 2. The base of the cylinder is in the
(Ans.

through surfaces of a right cylinder of radius 1 and


plane with the origin at the centre of the base.

Recap
In this lecture you have learnt the following
Line integral of a vector field along an arbitrary curve was defined. This is useful in applications, for
instance, in calculating work done under action of a force when a particle moves along a curve.
Concept was flux was understood by defining surface integral of a vector field.
Several problems involving line and surfave integrals were studied.

Module 1: A Crash Course in Vectors


Lecture 4 : Gradient of a Scalar Function
Objectives
In this lecture you will learn the following
Gradient of a Scalar Function

Divergence of a Vector Field

Divergence theorem and applications


Gradient of a Scalar Function :
Consider a scalar field such as temperature

in some region of space. The distribution of temperature

may be represented by drawing isothermal surfaces or contours connecting points of identical temperatures,

One can draw such contours for different temperatures. If we are located at a point
on one of these contours
and move away along any direction other than along the contour, the temperature would change.
The change

in temperature as we move away from a point


is given by

where the derivatives in the above expression are partial derivatives.

If the displacement from the initial position is infinitisimal, we get

to a point

Note that the change

involves a change in temperature with respect to each of the three directions. We define

a vector called the gradient of

, denoted by

or grad

as

using which, we get

Note that

, the gradient of a scalar

is itself a vector. If

is the angle between the direction of

and

where

is the component of the gradient in the direction of


. Thus,

is perpendicular to the surfaces of constant

. If

lies on an isothermal surface then


. When

and

are parallel,

has maximum value. Thus the magnitude of the gradient is equal to the maximum rate of change
of

and its direction is along the direction of greatest change.

. If a vector field can be written as a gradient of some some


The above discussion is true for any scalar field
scalar function, the latter is called the potential of the vector field. This fact is of importance in defining a
conservative field of force in mechanics. Suppose we have a force field

The line integral of

where the symbols

which is expressible as a gradient

can then be written as follows :

and

represent the initial and thec final positions and in the last step we have used an

similar to that derived for


above. Thus the line integral of the force field is independent of
expression for
the path connecting the initial and final points. If the initial and final points are the same, i.e., if the particle is
taken through a closed loop under the force field, we have

Since the scalar product of force with displacement is equal to the work done by a force, the above is a statement
of conservation of mechanical energy. Because of this reason, forces for which one can define a potential function
are called conservative forces.
Example 14
Find the gradient of the scalar function
Solution :

Exercise 1
Find the gradients of
(i)
(ii)
(iii)

(Ans.

Gradient can be expressed in other coordinate systems by finding the length elements in the direction of basis
vectors. For example, in cylindrical coordinates the length elements are

and

along

and

respectively. The expression for gradient is

The following facts may be noted regarding the gradient


1. The gradient of a scalar function is a vector
2.
3.
4.

Example 15
Find the gradient of
Solution :

in cylindrical (polar) coordinates.

In polar variables the function becomes

. Thus

Exercise 2
Find the gradient of the function
verify the answer.

of Example 15 in cartesian coordinates and then transform into polar form to

Exercise 3
Find the gradient of the function

in cylindrical coordinates.

(Ans.

) In spherical coordinates the length elements are

Hence the gradient of a scalar function

and

is given by

Exercise 4
Find the gradient of

(Ans.

.)

Exercise 5
A potential function is given in cylidrical coordinates as

Find the force field it represents and express

the field in spherical polar coordinates.


(Ans.

Divergence of a Vector Field :


Divergence of a vector field
is a measure of net outward flux from a closed surface
as the volume shrinks to zero.

enclosing a volume

where
is the volume (enclosed by the closed surface
) in which the point P at which the divergence is being
calculated is located. Since the volume shrinks to zero, the divergence is a point relationship and is a scalar.
Consider a closed volume

bounded by

. The volume may be mentally broken into a large number of

elemental volumes closely packed together. It is easy to see that the flux out of the boundary
is equal to the
sum of fluxes out of the surfaces of the constituent volumes. This is because surfaces of boundaries of two adjacent
volumes have their outward normals pointing opposite to each other. The following figure illustrates it.

We can generalize the above to closely packed volumes and conclude that the flux out of the bounding surface
of a volume

is equal to the sum of fluxes out of the elemental cubes. If

cube with

as the surface, then,

is the volume of an elemental

The quantity in the bracket of the above expression was defined as the divergence of

, giving

This is known as the Divergence Theorem.


from an infinitisimal volume over which variation of
is small so that one
We now calculate the divergence of
can retain only the first order term in a Taylor expansion. Let the dimensions of the volume element be
and let the element be oriented parallel to the axes.
Consider the contribution to the flux from the two shaded faces. On these faces, the normal is along the
directions so that the contribution to the flux is from the y-component of

Expanding

in a Taylor series and retaining only the fiirst order term

so that the flux from these two faces is

where

only and is given by

is the volume of the cuboid.

and

Combining the above with contributions from the two remaining pairs of faces, the total flux is

Thus

Comparing with the statement of the divergence theorem, we have

Recalling that the operator

is given by

and using

, we can write

The following facts may be noted :


1.The divergence of a vector field is a scalar

2.
3.

4. In cylindrical coordinates

5.In spherical polar coordinates

6. The divergence theorem is

Example 16
Divergence of
Divergence of position vector

is very useful to remember.

One can also calculate easily in spherical coordinate since

only has radial component

Exercise 6
Calculate the divergence of the vector field

using all the three coordinate systems.

(Ans. 0)

Example 17
A vector field is given by
unit cube bounded by planes
with the divergence theorem.

Solution :
Divergence of

is

The volume integral of above is

. Find the surface integral of the field from the surfaces of a


and

. Verify that the result agrees

Consider the surface integral from the six faces individually. For the face AEOD, the normal is along
face
along

so that

. Since

and on this face

, the integrand is zero. For the surface BFGC, the normal is

. On this face the vector field is

Consider the top face (ABFE) for which the normal is


and

. On this

. The surface integral is

so that the surface integral is

. On this face

. The contribution to the surface integral from this face is

For the bottom face (DOGC) the normal is along

and

. This gives

so that the integral

vanishes.
For the face EFGO the normal is along
giving

so that the surface integral is

. On this face

. The surface integral is zero. For the front face ABCD, the normal is along

and on this face

giving

. The surface integral is

Adding the six contributions above, the surface integral is

consistent with the divergence theorem.

Exercise 7

Verify the divergence theorem by calculating the surface integral of the vector field
the cubical volume of Example 17.

for

(Ans. Surface integral has value 3)

Example 18
In Example 13 we found that the surface integral of a vector field
height

is

over a cylinder of radius

and

. Verify this result using the divergence theorem.

Solution :
In Example 16 we have seen that the divergence of the field vector is 3. Since the integrand is constant, the
volume integral is

Example 19
A vector field is given by

. Verify Divergence theorem for a cylinder of radius 2 and height 5. The

origin of the coordinate system is at the centre of the base of the cylinderand z-axis along the axis.
Solution :
The problem is obviously simple in cylindrical coordinates. The divergence can be easily seen to be
. Recalling that the volume element is

, the integral is

In order to calculate the surface integral, we first observe that the end faces have their normals along
the field does not have any z- component, the contribution to surface integral from the end faces is zero.
We will calculate the contribution to the surface integral from the curved surface.
Using the coordinate transformation to cylindrical

and

. Since

Using these

The area element on the curved surface is

, where

where we have used

is the radius. Thus the surface integral is

Exercise 8
In the Exercise following Example 13, we had seen that surface integral of the vector field
through the surface of a cylinder of radius 1 and height 2 is

. Re-confirm the

same result using divergence theorem.


Example 20
A hemispherical bowl of radius 1 lies with its base on the x-y plane and the origin at the centre of the circular base.
Calculate the surface integral of the vector field

in the hemisphere and verify the

divergence theorem.
Solution :
The divergence of

where

is easily calculated

is the distance from origin. The volume integral over the hemisphere is conveniently calculated in spherical

polar using the violume element


is

to

. Since it is a hemisphere with

as the base, the range of

The surface integrals are calculated conveniently in spherical polar. There is no contribution to the flux from the
base because the outward normal points in the

direction but the z-component of the field is zero because the

base of the hemisphere is


.
In order to calculate the flux from the curved face we need to express the force field and the unit vectors in
spherical polar coordinates. Using the tranformation properties given earlier and observing that we only require the
radial component of the vector field since the area element is radially directed. Using

as the area

element, a bit of laborious algebra gives

Using

and

, the above integral can be seen to give the correct

result.
Recap
In this lecture you have learnt the following
Gradint of a scalar function was defined. Gradient is a scalar function.

The magnitude of the gradient is equal to the maxium rate of change of the scalar field and its direction is
along the direction of greatest change in the scalar function.

The net outward flux from a volume element around a point is a measure of the divergence of the vector
field at that point.

We derived trhe divergence theorem which shows that the volume integral of the divergence of a vector
function overany volume is equal to the outward flux through a surface which encloses this volume.

Divergence was calculated for functions in different coordinate systems and divergence theorem was verified.

Module 1 : A Crash Course in Vectors


Lecture 5 : Curl of a Vector - Stoke's Theorem
Objectives
In this lecture you will learn the following
Curl of a Vector field

Expression for curl in cartesian cylindrical and spherical coordinate

Dirac and Function


Curl of a Vector - Stoke's Theorem
We have seen that the line integral of a vector field

is essentially a sum of the component of

curve. If the line integral is taken over a closed path, we represent it as


i.e., if there exists a scalar function
cases, it is, in general, non-zero.

such that one can write

as

along the

. If the vector field is consevative,


, the contour integral is zero. In other

enclosing a surface
. We may split the contour into a large number of elementary surface
Consider a contour
areas defined by a mesh of closed contours.
Since adjacent contours are traversed in opposite directions, the only non-vanishing contribution to the integral comes
from the boundary of the contour

. If the surface area enclosed by the

cell is

, then

We define the quantity

as the curl of the vector

at a point P which lies on the surface

point relationship. The direction of

. Since the area

is, as usual, along the outward normal to the area element

is infinitisimal it is a
. For instance,

the x-component of the curl is given by

Thus

This is Stoke's Theorem which relates the surface integral of a curl of a vector to the line integral of the vector itself.
The direction of

and

in the diurection of

are fixed by the right hand rule, i.e. when the fingers of the right hand are curled to point

, the thumb points in the direction of

Curl in Cartesian Coordinates :


We will obtain an expression for the curl in the cartesian coordinates. Let us consider a rectangular contour ABCD in the
y-z plane having dimensions
corners is located at

. The rectangle is oriented with its edges parallel to the axes and one of the

. We will calculate the line integral of a vector field

along this contour. We assume the

field to vary slowly over the length (or the bredth) so that we may retain only the first term in a Taylor expansion in
computing the field variation.

Contribution to the line integral from the two sides AB and CD are computed as follows.
On AB :

ON CD :
Using Taylor expansion (retaining only the first order term), we can write

Thus the line integral from the pair of sides AB and CD is

In a similar way one can calculate the contributions from the sides BC and DA and show it to be

Adding up we get

In a very similar way, one can obtain expressions for the y and z components

One can write the expression for the curl of

by using the del operator as

Expression for curl in Cylidrical and Spherical Coordinates :


In the cylindrical coordinates the curl is given by

In the spherical coordinates the corresponding expression for the curl is

Example 21
Verify

Stoke's

theorem

for
and

for

the

rectangle

shown

below,

defined

by

sides

The line integrals along the four sides are

Since the normal to the plane is along


It can be checked that

, we only need

component of

to calculate the surface integral.

Thus

which agrees with the line integral calculated.

Exercise 1
A vector field is given by

. Calculate the line integral of the field along the triangular path

shown above. Verify your result by Stoke's theorem.


(Ans. 1)
(Hint : To calculate the line integral along a straightline, you need the equation to the line. For instance, the equation
. Check that

to the line BO is

.)

Example 22
. Calculate the line integral of the field along a circular path of

A vector field is given by


radius

in the x-y plane with its centre at the origin. Verify Stoke's theorem by considering the circle to define (i) the
.

plane of the circle and (ii) a cylinder of height


Solution :
may be calculated as

The curl of

Because

of

symmetry,

we

use

cylindrical

(polar)

. The unit vectors are

Substituting the above, the field

The line integral of

around the circular loop :

Since the line element is

On the circle

and its curl are given by

. The integral over

gives 1/2. Hence

coordinates.

The

transformations

are

Let us calculate the surface integral of the curl of the field over two surfaces bound by the circular curve.
(i) On the circular surface bound by the curve in the x-y plane, the outward normal is along

(ii) For the cylindrical cup, we have two surfaces : the curved face of the cylinder on which
circular face on which
only differ in their

. The contribution from the top circular cap is

values (the z-component of the curl is independent of

surface is (the area element is

(right hand rule). Thus

and the top

, as before because the two caps


). The surface integral from the curved

For both the terms of the above integral, the angle integration gives zero. Thus the net surface integral is
expected.

, as

Exercise 2
A vector field is given by

. Check the validity of the Stoke's theorm

and also calculating the


by calculating the line integral about the closed contour in the form of a circle at
surface integral of the open surface of the cylinder below it, as shown. (Hint : Express the curl in cylindrical coordinates
and take care of the signs of the surface elements from the curved surface and the bottom cap. Ans.

Exercise 3
Let

be a closed curve in the x-y plane in the shape of a quadrant of a circle of radius

If

. calculate the line integral of the field along the contour shown in a direction which is

anticlockwise when looked from above the plane (

). Take the surface of the quadrant enclosed by the curve as

the open surface bounded by the curve and verify Stoke's theorem.

(Ans.

Example 23
A vector field is given by

where

is the azimuthal angle variable of a spherical coordinate

system. Calculate the line integral over a circle of radius


in the x-y plane centered at the origin. Consider an open
surface in the form of a hemispherical bowl in the northern hemisphere bounded by the circle.
Solution :
On the equatorial circle

. Hence,

The expression for curl in spherical coordinates may be used to calculate the curl of
azimuthal component, the curl has radial and polar (

. Since the field only has

) components.

The area element on the surface of the northern hemisphere is

Hence the surface integral is

Exercise 4
Verify Stoke's theorem for a vector field

where the contour is an equatorial circle of radius

and is anticlockwise when viewed from above and the surface is the hemisphere shown in the preceding example.
(Ans.
Laplacian :

Since gradient of a scalar field gives a vector field, we may compute the divergence of the resulting vector field to
is called the Laplacian and is written as

obtain yet another scalar field. The operator div(grad)


If

is a scalar, then,

Example 24
Calculate the Laplacian of

Solution :

Adding similar contributions from

and

, we get

Laplacian in cylindrical and spherical coordinates:


In cylindrical :

In spherical :

Frequently the Laplacian of a vector field is used. It is simply a short hand notation for the componentwise Laplacian

Exercise 5
Show that

Dirac- Delta Function :


In electromagnetism, we often come across use of a function known as Diracfunction. The peculiarity of the
function is that though the value of the function is zero everywhere, other than at one point, the integral of the function
over any region which includes this singular point is finite. We define

with

where

is any function that is continuous at


. Strictly speaking,

, provided that the range of integration includes the point

is not a function in the usual sense as Riemann integral of any function which is

zero everywhere, excepting at discrete set of points should be zero. However, one can look at the
of a sequence of functions. For instance, if we define a function

Then

can be thought of as the limit of

as

such that

One can easily extend the definition to three dimensions

which has the property

provided, of course, the range of integration includes the point


Example :
Evaluate
Solution :

function as a limit

The range of integration includes the point

at which the argument of the delta function vanishes. Thus, the

value of the integral is


Exercise :

, where

Evaluate

and the integration is over a sphere of

radius 1.5 centered at (2,2,2)(Ans. -4) A physical example is the volume density of charge in a region which contains a
point charge

. The charge density is zero everywhere except at the point where the charge is located. However, the

volume integral of the density in any region which includes this point is equal to
point

itself. Thus if

is located at the

, we can write

Example :
Show that

is a delta function.

Solution :
As

, we have

using this it is easy to show that

Thus

However, the above is not true at the origin as


Interestingly, however, the integral of

diverges at

and is not differentiable at that point.

over any volume which includes the point

value of the integrand is zero everywhere excepting at the origin, the point
Consider an infinitisimally small sphere of radius

where the last integral is over the surface


the surface for which
the integral is

Hence

, we may replace

is not zero. As the

has to be treated with care.

with the centre at the origin. Using divergence theorem, we have,

of the sphere defined above. As the gradient is taken at points on


with

at all points on the surface. Thus the value of

Recap
In this lecture you have learnt the following
Curl of a vector field was defined.

Stokes theorem was established. According to Stoke's theorem the surface integral of the curl of a vector through
a surface is equal to the line integral of the field over any curve which binds the surface.

Stokes theorem was verified by calculating the curl for several cases of vector field.

Laplacian was defined and its expression in spherical polar and cylindrical coordinates was obtained.

We defined a generalized function called Dirac- Delta function which has the property that it vanishes
everywhere except at a point where its argument vanishes. Even so, the integral of the function over any region
of space which includes the point at which the argument of the delta function vanishes, is, in general, non-zero.

Module 2 : Electrostatics
Lecture 7 : Electric Flux
Objectives
In this lecture you will learn the following
Concept of flux and calculation of eletric flux throught simple geometrical objects

Gauss's Law of electrostatics

Applications of Gauss's Law to Calculate electric field due to a few symmetric charge distributions.
Electric Flux
The concept of flux is borrowed from flow of water through a surface. The amount of water flowing through a
surface depends on the velocity of water, the area of the surface and the orientation of the surface with respect
to the direction of velocity of water.
Though an area is generally considered as a scalar, an element of area may be considered to be a vector
because
It has magnitude (measured in m

).

If the area is infinitisimally small, it can be considered to be in a plane. We can then associate a
direction with it.
For instance, if the area element lies in the x-y plane, it can be considered to be directed along the z-direction.
(Conventionally, the direction of the area is taken to be along the outward normal.)

Click here for Animation


is chosen to represent the area in some convenient unit and its
In the figure above, the length of the vector
direction is taken to be along the outward normal to the area.
We define the flux of the electric field through an area
If

to be given by the scalar product

is the angle between the electric field and the area vector

For an arbitrary surface S, the flux is obtainted by integrating over all the surface elements

If the electric field is uniform, the angle

is constant and we have

Thus the flux is equal to the product of magnitude of the electric field and the projection of area perpendicular
to the field.

Click here for Animation


Unit of flux is N-m
it.

/C. Flux is positive if the field lines come out of the surface and is negative if they go into

Solid Angle :
The concept of solid angle is a natural extension of a plane angle to three dimensions. Consider an area element
dS at a distance

from a point P. Let be the unit vector along the outward normal to .

The element of the solid angle subtended by the area element at P is defined as

where is the projection of along a direction perpendicular to . If is the angle between

and , then,

Solid angle is dimensionless. However, for practical reasons it is measured in terms of a unit called steradian
(much like the way a planar angle is measured in terms of degrees).
The maximum possible value of solid angle is , which is the angle subtended by an area which encloses the
point P completely.
Example
A right circular cone has a semi-vertical angle

. Calculate the solid angle at the apex P of the cone.

Solution :
The cap on the cone is a part of a sphere of radius R, the slant length of the cone. Using spherical polar
coordinates, an area element on the cap is
sin
, where
is the polar angle and
is the
azimuthal angle. Here,

goes from 0 to

while

goes from 0 to

Thus the area of the cap is

Thus the solid angle at P is

Exercise
Calculate the solid angle subtended by an octant of a sphere at the centre of the sphere.
(Ans.

The flux per unit solid angle is known as the intensity .

Example 3
An wedge in the shape of a rectangular box is kept on a horizontal floor. The two triangular faces and the
rectangular face ABFE are in the vertical plane. The electric field is horizontal, has a magnitude

N/C

and enters the wedge through the face ABFE, as shown. Calculate the flux through each of the faces and
through the entire surface of the wedge.

The outward normals to the triangular faces AED, BFC, as well as the normal to the base are perpendicular to
. Hence the flux through each of these faces is zero. The vertical rectangular face ABFE has an area 0.06 m
. The outward normal to this face is perpendicular to the electric field. The flux is entering through this face
and is negative. Thus flux through ABFE is

To find the flux through the slanted face, we need the angle that the normal to this face makes with the
horizontal electric field. Since the electric field is perpendicular to the side ABFE, this angle is equal to the angle
. The area of the slanted face ABCD is 0.1 m

between AE and AD, which is

. Thus the flux

through ABCD is
The flux through the entire surface of the wedge is
Example 4
Calculate the flux through the base of the cone of radius

Click here for Animation


Solution :
The flux entering is perpendicular to the base. Since the outward normal to the circular base is in the opposite
sense, the flux is negative and is equal to the product of the magnitude of the field and the area of the base,
The flux, therefore is,

Example 5
Calculate the flux coming out through the curved surface of the cone in the above example.
Solution :

Click here for Animation

Consider a circular strip of radius


through the strip and the vector

at a depth
is

, where

element along the slope, the area of the strip is

We have,

to

is the semi-angle of the cone. If

is the length

. Thus,

. Further, r = h tan

, so that

Integrating from

from the apex of the cone. The angle between the electric field

Substituting, we get

, the height of the cone, the outward flux is

Example 6
A charge

is located at the center of a sphere of radius

. Calculate the flux going out through the surface

of the sphere.

Click here for Animation


By Coulomb's law, the field due to the charge

is radial and is given on the surface of the sphere by,

The direction of the area vector

, is also radial at each point of the surface

The integral over


is equal to the surface area of the sphere, which is,
surface of the sphere is

. The flux

. Thus the flux out of the

GAUSS'S LAW - Integral form


The flux calculation done in Example 4 above is a general result for flux out of any closed surface, known as
Gauss's law.
Total outward electric flux

through a closed surface

is equal to

times the charge enclosed

by the volume defined by the surface

Click here for Animation


Mathematicaly, the surface integral of the electric field over any closed surface is equal to the net charge
enclosed divided by

The law is valid for arbitry shaped surface, real or imaginary.

Its physical content is the same as that of Coulomb's law.

In practice, it allows evaluation of electric field in many practical situations by forming imagined
surfaceswhich exploit symmetry of the problem. Such surfaces are called Gaussian surfaces .

GAUSS'S LAW - Differential form


The integral form of Gauss's law can be converted to a differential form by using the divergence theorem. If
is the volume enclosed by the surface S,

If

is the volume charge density,

Thus we have

Applications of Gauss's Law


Field due to a uniformly charged sphere of radius with a charge
Gaussian surface is a cylinder of radius

and length

By symmetry, the field is radial. Gaussian surface is a concentric sphere of radius


the Gaussian surface is parallel to the field at every point. Hence For ,

. The outward normals to

so that
The field outside the sphere is what it would be if all the charge is concentrated at the origin of the sphere.
For , a fraction of the total charge is enclosed within the gaussian surface, so that
The field inside is
Exercise
Find the electric field both inside and outside a spherical shell of radius carrying a uniform charge .
Example
Find the electric field inside a sphere of radius which carries a charge density where
origin and is a constant.

is the distance from the

Solution :
By symmetry the field is radial. Take the gaussian surface to be a sphere of radius . The flus is . The charge
enclosed by the gaussian surface is

Click here for Animation

Thus

where

is a unit vector perpendicular to the line,directed outward for positive line charge and inward for

negative line charge.


Field due to an infinite charged sheet with surface charge density
Choose a cylindrical Gaussian pillbox of height
radius

(with

above the sheet and

below the sheet) and

Click here for Animation


The amount of charge enclosed is area times the surface charge density, i.e.,

. By symmetry, the

field is directed perpendicular to the sheet, upward at points above the sheet and downward for points below.
There is no contribution to the flux from the curved surface. The flux from the two end faces is
each,

i.e. a total outward flux of

. Hence

so that

where
is a unit vector perpendicular to the sheet, directed upward for points above and downwards for points
below (opposite, if the charge density is negative).
Field due to a uniformly charged sphere of radius

with a charge

By symmetry, the field is radial. Gaussian surface is a concentric sphere of radius


the Gaussian surface is parallel to the field

. The outward normals to

at every point. Hence

Click here for Animation


For

so that

The field outside the sphere is what it would be if all the charge is concentrated at the origin of the sphere.
For

, a fraction

The field inside is

of the total charge is enclosed within the gaussian surface, so that

Exercise 1
Find the electric field in the region between two infinite parallel planes carrying charge densities

and

Exercise 2
Find the electric field both inside and outside a spherical shell of radius

carrying a uniform charge

Exercise 3
Find the electric field both inside and outside a long cylinder of radius
density

carrying a uniform volume charge

(Hint : Take the gaussian surface to be a finite concentric cylinder of radius

(with

and

), as

shown)

Example 7
Find the electric field inside a sphere of radius
distance from the origin and

which carries a charge density

where

is a constant.

Solution :

By symmetry the field is radial. Take the gaussian surface to be a sphere of radius

. The flus is

is the

.
Click here for Animation
The charge enclosed by the gaussian surface is

Thus

(what is the dimension of

?)

Exercise 4
A very long cylinder carries a charge density
Find the electric field at a distance

, where
(Ans.

is the distance from the axis of the cylinder.


)

Exercise 5
A charge

(Ans.

is located at the center of a cube of side

. Find the flux through any of the sides.

Example 8
Find the flux through the curved surface of a right circular cone of base radius
. The cone has no charge and the electric field is normal to the base.

in an external electric field

Solution :
In Example 4, we calculated the flux through the base to be

. As the cone does not contain any

charge, by Gauss's law, the flux through the curved surface must be

which is the result we obtained in Example 4.


Recap
In this lecture you have learnt the following
Electric flux through a surface is surface integral of normalcomponent of electric field through the
surface.

Gauuss's of electrostatics states that the flux of electric field from any closed surface is proportional to
the charge enclosed by the surface. Both the integral and differential form of Gauss's law were studied.

Gauss's law helps us to deternmine electric field due to charge distributions having spatial symmetry.
Fields due to distributions showing spherical and cylidrical symmetry were studied.

Electric field due a charged sheet was obtained using Gauss's law.

Module 2 : Electrostatics
Lecture 7 : Electric Flux
Objectives
In this lecture you will learn the following
Concept of flux and calculation of eletric flux throught simple geometrical objects

Gauss's Law of electrostatics

Applications of Gauss's Law to Calculate electric field due to a few symmetric charge distributions.
Electric Flux
The concept of flux is borrowed from flow of water through a surface. The amount of water flowing through a
surface depends on the velocity of water, the area of the surface and the orientation of the surface with respect
to the direction of velocity of water.
Though an area is generally considered as a scalar, an element of area may be considered to be a vector
because
It has magnitude (measured in m

).

If the area is infinitisimally small, it can be considered to be in a plane. We can then associate a
direction with it.
For instance, if the area element lies in the x-y plane, it can be considered to be directed along the z-direction.
(Conventionally, the direction of the area is taken to be along the outward normal.)

Click here for Animation


is chosen to represent the area in some convenient unit and its
In the figure above, the length of the vector
direction is taken to be along the outward normal to the area.
We define the flux of the electric field through an area
If

to be given by the scalar product

is the angle between the electric field and the area vector

For an arbitrary surface S, the flux is obtainted by integrating over all the surface elements

If the electric field is uniform, the angle

is constant and we have

Thus the flux is equal to the product of magnitude of the electric field and the projection of area perpendicular
to the field.

Click here for Animation


Unit of flux is N-m
it.

/C. Flux is positive if the field lines come out of the surface and is negative if they go into

Solid Angle :
The concept of solid angle is a natural extension of a plane angle to three dimensions. Consider an area element
dS at a distance

from a point P. Let be the unit vector along the outward normal to .

The element of the solid angle subtended by the area element at P is defined as

where is the projection of along a direction perpendicular to . If is the angle between

and , then,

Solid angle is dimensionless. However, for practical reasons it is measured in terms of a unit called steradian
(much like the way a planar angle is measured in terms of degrees).
The maximum possible value of solid angle is , which is the angle subtended by an area which encloses the
point P completely.
Example
A right circular cone has a semi-vertical angle

. Calculate the solid angle at the apex P of the cone.

Solution :
The cap on the cone is a part of a sphere of radius R, the slant length of the cone. Using spherical polar
coordinates, an area element on the cap is
sin
, where
is the polar angle and
is the
azimuthal angle. Here,

goes from 0 to

while

goes from 0 to

Thus the area of the cap is

Thus the solid angle at P is

Exercise
Calculate the solid angle subtended by an octant of a sphere at the centre of the sphere.
(Ans.

The flux per unit solid angle is known as the intensity .

Example 3
An wedge in the shape of a rectangular box is kept on a horizontal floor. The two triangular faces and the
rectangular face ABFE are in the vertical plane. The electric field is horizontal, has a magnitude

N/C

and enters the wedge through the face ABFE, as shown. Calculate the flux through each of the faces and
through the entire surface of the wedge.

The outward normals to the triangular faces AED, BFC, as well as the normal to the base are perpendicular to
. Hence the flux through each of these faces is zero. The vertical rectangular face ABFE has an area 0.06 m
. The outward normal to this face is perpendicular to the electric field. The flux is entering through this face
and is negative. Thus flux through ABFE is

To find the flux through the slanted face, we need the angle that the normal to this face makes with the
horizontal electric field. Since the electric field is perpendicular to the side ABFE, this angle is equal to the angle
. The area of the slanted face ABCD is 0.1 m

between AE and AD, which is

. Thus the flux

through ABCD is
The flux through the entire surface of the wedge is
Example 4
Calculate the flux through the base of the cone of radius

Click here for Animation


Solution :
The flux entering is perpendicular to the base. Since the outward normal to the circular base is in the opposite
sense, the flux is negative and is equal to the product of the magnitude of the field and the area of the base,
The flux, therefore is,

Example 5
Calculate the flux coming out through the curved surface of the cone in the above example.
Solution :

Click here for Animation

Consider a circular strip of radius


through the strip and the vector

at a depth
is

, where

element along the slope, the area of the strip is

We have,

to

is the semi-angle of the cone. If

is the length

. Thus,

. Further, r = h tan

, so that

Integrating from

from the apex of the cone. The angle between the electric field

Substituting, we get

, the height of the cone, the outward flux is

Example 6
A charge

is located at the center of a sphere of radius

. Calculate the flux going out through the surface

of the sphere.

Click here for Animation


By Coulomb's law, the field due to the charge

is radial and is given on the surface of the sphere by,

The direction of the area vector

, is also radial at each point of the surface

The integral over


is equal to the surface area of the sphere, which is,
surface of the sphere is

. The flux

. Thus the flux out of the

GAUSS'S LAW - Integral form


The flux calculation done in Example 4 above is a general result for flux out of any closed surface, known as
Gauss's law.
Total outward electric flux

through a closed surface

is equal to

times the charge enclosed

by the volume defined by the surface

Click here for Animation


Mathematicaly, the surface integral of the electric field over any closed surface is equal to the net charge
enclosed divided by

The law is valid for arbitry shaped surface, real or imaginary.

Its physical content is the same as that of Coulomb's law.

In practice, it allows evaluation of electric field in many practical situations by forming imagined
surfaceswhich exploit symmetry of the problem. Such surfaces are called Gaussian surfaces .

GAUSS'S LAW - Differential form


The integral form of Gauss's law can be converted to a differential form by using the divergence theorem. If
is the volume enclosed by the surface S,

If

is the volume charge density,

Thus we have

Applications of Gauss's Law


Field due to a uniformly charged sphere of radius with a charge
Gaussian surface is a cylinder of radius

and length

By symmetry, the field is radial. Gaussian surface is a concentric sphere of radius


the Gaussian surface is parallel to the field at every point. Hence For ,

. The outward normals to

so that
The field outside the sphere is what it would be if all the charge is concentrated at the origin of the sphere.
For , a fraction of the total charge is enclosed within the gaussian surface, so that
The field inside is
Exercise
Find the electric field both inside and outside a spherical shell of radius carrying a uniform charge .
Example
Find the electric field inside a sphere of radius which carries a charge density where
origin and is a constant.

is the distance from the

Solution :
By symmetry the field is radial. Take the gaussian surface to be a sphere of radius . The flus is . The charge
enclosed by the gaussian surface is

Click here for Animation

Thus

where

is a unit vector perpendicular to the line,directed outward for positive line charge and inward for

negative line charge.


Field due to an infinite charged sheet with surface charge density
Choose a cylindrical Gaussian pillbox of height
radius

(with

above the sheet and

below the sheet) and

Click here for Animation


The amount of charge enclosed is area times the surface charge density, i.e.,

. By symmetry, the

field is directed perpendicular to the sheet, upward at points above the sheet and downward for points below.
There is no contribution to the flux from the curved surface. The flux from the two end faces is
each,

i.e. a total outward flux of

. Hence

so that

where
is a unit vector perpendicular to the sheet, directed upward for points above and downwards for points
below (opposite, if the charge density is negative).
Field due to a uniformly charged sphere of radius

with a charge

By symmetry, the field is radial. Gaussian surface is a concentric sphere of radius


the Gaussian surface is parallel to the field

. The outward normals to

at every point. Hence

Click here for Animation


For

so that

The field outside the sphere is what it would be if all the charge is concentrated at the origin of the sphere.
For

, a fraction

The field inside is

of the total charge is enclosed within the gaussian surface, so that

Exercise 1
Find the electric field in the region between two infinite parallel planes carrying charge densities

and

Exercise 2
Find the electric field both inside and outside a spherical shell of radius

carrying a uniform charge

Exercise 3
Find the electric field both inside and outside a long cylinder of radius
density

carrying a uniform volume charge

(Hint : Take the gaussian surface to be a finite concentric cylinder of radius

(with

and

), as

shown)

Example 7
Find the electric field inside a sphere of radius
distance from the origin and

which carries a charge density

where

is a constant.

Solution :

By symmetry the field is radial. Take the gaussian surface to be a sphere of radius

. The flus is

is the

.
Click here for Animation
The charge enclosed by the gaussian surface is

Thus

(what is the dimension of

?)

Exercise 4
A very long cylinder carries a charge density
Find the electric field at a distance

, where
(Ans.

is the distance from the axis of the cylinder.


)

Exercise 5
A charge

(Ans.

is located at the center of a cube of side

. Find the flux through any of the sides.

Example 8
Find the flux through the curved surface of a right circular cone of base radius
. The cone has no charge and the electric field is normal to the base.

in an external electric field

Solution :
In Example 4, we calculated the flux through the base to be

. As the cone does not contain any

charge, by Gauss's law, the flux through the curved surface must be

which is the result we obtained in Example 4.


Recap
In this lecture you have learnt the following
Electric flux through a surface is surface integral of normalcomponent of electric field through the
surface.

Gauuss's of electrostatics states that the flux of electric field from any closed surface is proportional to
the charge enclosed by the surface. Both the integral and differential form of Gauss's law were studied.

Gauss's law helps us to deternmine electric field due to charge distributions having spatial symmetry.
Fields due to distributions showing spherical and cylidrical symmetry were studied.

Electric field due a charged sheet was obtained using Gauss's law.

Module 2 : Electrostatics
Lecture 8 : Properties Of Conductors
Objectives
In this lecture you will learn the following
Properties of Conductors in equilibrium

Induced charges in a conductor

Electrostatic Shielding

Concept of electric potential and its calculation

Determination of electric field from a knowledge of potential


Properties of Conductors
A conductor (typically,a metal or ionic conductors like HCl or NaCl dissolved in water) allows free movement of
charges,
The electric field inside a conductor is zero. In an equilibrium situation, there cannot be an electric
field inside a conductor as this would cause charges (electrons or ions) to move around.
Charge density inside a conductor is zero. This follows from Gauss's law

As

, the charge density

(This does not suggest that there is no charge inside, only that the positive and negative charges cancel inside
a conductor.)

Free charges exist only on the surface of a conductor. Since there is no net charge inside, free
charges, if any, have to be on the surface.

At the surface of a conductor, the electric field is normal to the surface. If this were not so, the
charges on the surface would move along the surface because of the tangential component of the field,
disturbing equilibrium.

Click here for Animation


Induced Charges in a conductor:
The above properties of a conductor influence the behaviour of a conductor placed in an electric field. Consider,
for instance, what happens when a charge
is brought near an uncharged conductor. The conductor is
placed in the electric field of the point charge. The field inside the conductor should, however, be zero. his is
achieved by a charge separation within the conductor which creates its own electric field which will exactly
compensate the field due to the charge
. The separated charges must necessarily reside on the surface.

Another way of looking at what is happening is to think of the free charges in the conductor being attracted
towards (or repelled from) the external charge. Thus the surface of the conductor towards the external charge is
oppositely charged. To keep the charge neutrality, the surface away from the external charge is similarly
charged.
Example 9
A charge

is located in the cavity inside a conducting shell. In addition, a charge

conducting shell. Find the distribution of charge in the shell.

is distributed in the

Click here for Animation


Take a gaussian surface entirely within the conducting shell, completely enclosing the cavity. Everywhere on the
gaussian surface

. The flux and therefore, the charge enclosed is zero within the gaussian surface. As

the cavity contains a charge

, the inner surface of the cavity must have charge

, the charge on the outside surface is

distributed charge

. As the conductor has

located at the centre of the


The electric field outside the shell is same as that due to a point charge
sphere. The reason is that the charge in the cavity is compensated exactly by the induced charges in the
surface of the cavity. Thus the effect of the charge
distributed charge of

is not felt at points outside the sphere. The uniforml

over the surface of the sphere gives the same field as that of a point charge located

at the centre.
Exercise
Consider the system of conductors shown with two cavities. A charge
is kept at the center. (i) Determine
the charge distributions on the surfaces marked 1,2,3 and 4, (ii) Is the potential of surface 1 lower, higher or
same as that of surface 2 ? (iii) Is the potential of surface 4 lower, higher or same as that of surface 1 ?
(Answer : (i)

for 1 and 3, (ii)

for 2 and 4 (ii) equal (iii) lower.)

Example 10
Calculate the electric field outside a conductor carrying a surface charge density

Click here for Animation


Take a gaussian pillbox in the shape of a cylinder of height

with

inside and

outside the conductor.

Lat the cross sectional area be


normal to the surface. The electric field is normal to the surface. As the
field inside is zero and there is no tangential component of the field at the surface, the flux goes out only
through the outer cap of the cylider. The charge enclosed is
normal to the surface. applyinng Gauss's law

and the flux is

. The electric field is

Exercise 1
Two parallel, infinite plates made of material of perfect conductor, carry charges

and

. The plates have

finite thickness. Show that the charge densities on the two adjecent inside surfaces are equal and opposite
while that on the two outside
surfaces are equal.
(Hint : Field inside the plates due to four charged surfaces must be zero.)
ELECTROSTATIC POTENTIAL
Electrostatic force is a conservative force, i.e., the work done by the force in moving a test charge from one
point to another is independent of the path connecting the two points.
Example 11
Calculate the work done by the electric field due to a charge

located at the origin to take a unit test charge

from the point A to the point B, along the path shown.


Solution :
The force on the unit test charge due to the charge

is, by definition, the electric field due to the charge

. The work done by the field on the test charge is

where

is along the path connecting A and B.

We use the spherical polar coordinates to express the length element

As the force is central, only the radial component of

contributes to the integral.

which depends only on the end points. Line integral of the electric field around any closed path is zero.
Consider the line integral

. Since the integral is independent of the path of integration, we have

Click here for Animation


Since

along any particular path,

L.H.S. is the line integral of the electric field along the closed loop,

Since the work done by the electric field is independent of path, we may write

where

is a scalar function which depends only on the end points A and B of integration. By property

of integrals

where

is any arbitrary point. Thus

An intutive form for

is

where

is a scalar function which depends only on the position

potential of the position

is called the electrostatic

is the potential difference between the positions

and

. The

absolute value of the potential at a position is meaningless unless we define a reference position at which the
potential is zero. Since Coulomb force vanishes only at infinite distances from a source, it is convenient to take
infinity to be such a reference position. Thus, the potential at a position

is

It may be noted that such a reference point is an inappropriate choice for some infinite distribution of charges
(e.g. a line charge) where the field does not fall off fast enough to make the integral above vanish.
Example 12
Potential due to a uniform electric field

If the electric field is

and is directed along the x-direction,

choosing the reference point

to be at the origin,

Example 13
Obtain an expression for the potential at a distance
density

Solution

from a line charge distribution with a linear charge

The electric field due to the charge distribution at a point P located at a distance

where

is

is along the perpendicular from the point P to the line charge, as shown (the direction is opposite if

line charge density is negative). The potential difference between the point
obtained by calculating the value of the integral

from the point

independent of path, we calculate it along the path


integral from the path
the directions of

and

is zero as along this path

and a reference point

to the point

is

. As the integral is

, as shown. The contribution to the


is perpendicular to

. Along the path

are parallel.

Hence, if the perpendicular distance of

from the line charge is denoted by

Click here for Animation


One can see that taking

to be infinite will make the integral diverge. In this case, it it is convenient to take

the zero of the potential to be at unit distance from the line (


distance

). With this choice the potential at a

from the line is

Exercise 1
Find the potential at a height
above a uniformly charged infinite plane having a charge density
good reference point for the zero of the potential ?
[Ans.

, with

. What is a

Unit of Potential
Since potential is the energy per unit charge, the unit of potential is Joule/Coulomb, which is called a volt . The
unit of the electric field which we have so far been using as Newton/Coulomb is more commonly referred as
volt/meter.
Potential Function satisfies Superposition Principle
. The electric field at a point due to the distrbution of charges

Consider a collection of charges


obeys superposition principle. If

is the electric field at a point P due to the charge

at P is

The potential

where

at the point P (with respect to the reference point P

is the potential at P due to the charge

) is

Determining Electric Field from a knowledge of Potential


The potential at the position

where

is given by the expression

is a reference point such that

In one dimension,

Differentiate both sides with respect to the upper limit of integration, i.e.

In three dimension, we use the fundamental theorem on gradients

which gives

Comparing the above with eqn. (A) above,

In cartesian coordinates,

, the net electric field

Example 14
In a certain region of space, the potential function is given by the expression

where the potential is measured in volts and the distances in meters. Determine the electric field at the point

.
Solution :
Using

Substituting for

and

Exercise 2
The potential in a certain region of space is given by the function
Find the y-component of the electric field at

(Ans.

with respect to some reference point.

Exercise 3
Find the potential at a distance

from the mid-point of a charged line of length

carrying a total charge

Using this determine the electric field at the point. (Compare your result for the electric field with the field
calculated in Example 2.)

Electric Field is Irrotational, i.e. Curl


This follows from Stoke's theorem.

where the surface integral is over any surface bounded by the closed curve. As the surface
long as it is bounded by the same curve) , the integrand must vanish. Hence,

is arbitrary (as

Click here for Animation


Potential and Potential Energy
Potential and Potential energy are different, though they are related. Electric potential at a point is the potential
energy of a unit test charge kept at that point.
Potential is the property of the field and is defined at every point, whether or not a charge is located at the
point. It is the potential energy that the unit test charge would have if it happens to be located at that point.
The potential energy of a positive charge

increases if it is taken to a region of higher potential. If electric

force is the only force acting on the particle, its kinetic energy would decrease by a similar amount.
Let the charge have a velocity

at the position P

where the electrostatic potential is

.If it moves to a

position
P

, where the potential is

, then, the velocity

of the particle at this point is given by the energy

conservation

The work done on the particle in moving from a potential

to the potential

is given by the ``work-energy

theorem"

Volt,the unit of potential difference,may be interpreted as follows.If a charge of one coulomb moves through a

potential difference such that in the new position the potential is lower by 1 Volt,the kinetic energy of the
charge increases by 1Joule.
Electron Volt
In atomic and nuclear physics, a commonly used unit of energy is electron volt. An electron volt is the change
in the kinetic energy of an electron when it is taken through a potential difference of one volt. Thus,

Example 15
An

- particle with a kinetic energy of 1 MeV is projected towards a stationary nucleus with a charge

. Neglecting the motion of the nucleus, determine the distance of closest approach of the

- particle.

Solution :

Click here for Animation


Initial energy of

-particle = 1 Mev =

eV.

At the distance of closes approach, the velocity (and the kinetic energy) of the
- particle is zero. Hence, all its
kinetic energy has been converted into potential energy. The potential energy (reference at infinity) at a
distance

is

Equating this to the initial kinetic energy,

m.

Recap
In this lecture you have learnt the following
Conductors are those in which there are free carriers which conduct electricity. In an equilibrium
situation, electric field cannot exist inside a conductor.

Electric field may exist on the surface of a conductor which must then be directed normal to the surface
of the conductor.

When a conductor is placed in an electric field, charges may be induced on the surface of a conductor.

Electric field being a conservative field of force, the work done only depends on the end points and not on
the path connecting them. This enables us to define a potential for the field whose negative gradient
gives the electric field.

As the electric potential is a scalar, it is convenient to compute the potential for a charge distribution and
use this to calculate the electric field using superposition principle.

Module 2 : Electrostatics
Lecture 9 : Electrostatic Potential
Objectives
In this lecture you will learn the following
Electric Dipole and field due to a dipole

Torque on a dipole in an inhomogeneous electric field

Potential Energy of a dipole

Energy of a system of charges - discrete and continuous


Potential and Field due to an Electric Dipole
An electric dipole consists of two equal and opposite charges
The Electric Dipole Moment

and

is defined as a vector of magnitude

separated by a small distance

with a direction from the negative charge

to the positive charge. In many molecules, though the net charge is zero, the nature of chemical bonds is such
that the positive and negative charges do not cancel at every point. There is a small separation between the
positive charge centres and negative charge centres. Such molecules are said to be polar molecules as they
have a non-zero dipole moment. The figure below shows an asymmetric molecule like water which has a dipole
moment

C-m.

Click here for Animation

Click here for Animation


In the polar

where

coordinates shown in the figure

and

increasing

are unit vectors in the radial and tangential directions, taken respectively, in the direction of
and increasing

The electric potential at a point P with a position vector

is

Click here for Animation


If the distance

where

is small compared to

is the angle between

(i.e., if the point P is far away from the dipole), we may use

and the dipole moment vector

. This gives

Electric Field of a Dipole


A.CARTESIAN COORDINATES
It is convenient to define the cartesian axes in the following way. Let the dipole moment vector be taken along
the z-axis and position vector

of P in the y-z plane (We have denoted the point where the electric field is

calculated by the letter P and the electric dipole moment vector as


. Thus

). We then have

with

Since

is independent of

. The y and z components are

and

B. POLAR COORDINATES
In polar (

) coordinates, the radial and tangential components of the field are as follows :

GENERAL EXPRESSION
A respresentation independent form for the dipole field can be obtained from the above
We have

Using

, we get

This form does not depend on any particular coordinate system. Note that, at large distances, the dipole field
decreases with distance as

where as monopole field (i.e. field due to a point charge) decreases as

.
Dipole in a uniform Electric Field
The net force on the dipole is zero. There is a net torque acting on the dipole. If
the torque is

is the length of the dipole,

Expressing in vector form,

If

or

, (i.e. when the dipole is aligned parallel or antiparallel to the field) the torque vanishes
and unstable if

and the dipole is in equilibrium. The equilibrium is stable if

Example 16
The net electric force on a dipole is zero only if the field is uniform. In a non-uniform field, the dipole
separated by a distance

experiences a net force. Consider a dipole consisting of charges


, where

field

and

in an electric

are constants. Determine the net force on the dipole when the dipole

is aligned (a) parallel and (b)anti-parallel to the field.

Click here for Animation


Solution :
Consider the dipole shown above, where the charges are separated by a distance
. Let the field at the charge

moment is
charge

is

and on

is

be

and that at be

. There is a net force

write the force as

where the last equality is valid for an ideal dipole for which
In our case

and

so that the net force is

, so that the dipole


. The force on the

to the right (z-direction). We can

Work done in turning a dipole from equilibrium


If the dipole is twisted by an angle
agency

from its stable equilibrium position, work has to be done by the external

This work becomes the potential energy of the dipole in this position.
Exercise 4
C separated by a distance of

An electric dipole consisting of two charges


an equilibrium position in a uniform electric field of strength

m is in

N/C. Calculate the work done in rotating

the dipole to a position in which the dipole is perpendicular to the field.


(Ans.

J)

Energy of a Dipole
in the electric field, we have to add to the work done
To calculate energy of a dipole oriented at an angle
above, the energy of the dipole in the equilibrium position. This is equal to the work done in bringing the dipole
from infinity to the equilibrium position. The dipole may be aligned in the direction of the field at infinity without
any cost of energy. We may now displace the dipole parallel to the field to bring to the equilibrium position. As
the negative charge is displaced along the field by an additional distance

, the work done is

, which is the potential energy of the dipole in equilibrium.


The potential energy of the dipole at position

The energy is positive if

is

is acute and is negative if

is obtuse.

Potential Energy of a System of Charges


Assume all charges to be initially at infinity. We assemble the charges by bringing the charges one by one and
fix them in their positions. There is no energy cost in bringing the first charge
is no force field. Thus

and putting it at P

, as there

We now bring the second charge and take it to point P

. Since this charge moves in the potential field of the

first charge, the work done in bringing this charge is

where

is the potential at P

under the force exerted by bth

due to the charge at P

and

. The third charge

is to be brought to P

and is

and so on.
The work done in assembling

The extra factor of

excludes the terms

charges

, located respectively at

in the last expression is to ensure that each pair

. Since the potential at the

is

is counted only once. The sum

- th position due to all other charges is

we get

Energy of a continuous charge distribution


If

is the density of charge distribution at

, we can generalize the above result

(In case of a line charge or a surface charge distribution, the integration is over the appropriate dimension).
Since the integral is over the charge distribution, it may be extended over all space by defining the charge
density to be zero outside the distribution, so that the contribution to the integral comes only from the region of
space where the charge density is non-zero. Writing

From the differential form of Gauss's law, we have

With this

On using the vector identity

we get, using

The first integral can be converted to a surface integral by using divergence theorem and the surface can be
taken at infinite distances, where the electric field is zero. As a result the first integral vanishes and we have

Recap
In this lecture you have learnt the following
An electric dipole consist of a pair of equal and opposite charges separated by a small distance. The
dipolemoment is a vector whose magnitude is equal to product of the charge with the distance and has a
direction from the negative charge to the positive charge.

At large distances, the electric field due to an electric dipole varies as the inverse cube of distance from

the dipole.

A dipole does not experience a force in a uniform electric field. However, it experiences a torque in such
a field. In an inhomogeneous field, the dipole experiences a force.

The potential energy of a dipole in an electric field was calculated. The method was extended to compute
the potential energy of a general charge distribution. Expressions for both discrete and continuous charge
distribution were obtained.

Module 2 : Electrostatics
Lecture 10 : Poisson Equations
Objectives
In this lecture you will learn the following
Poisson's equation and its formal solution

Equipotential surface

Capacitors - calculation of capacitance for parallel plate, spherical and cylindrical capacitors

Work done in charging a capacitor


Poisson Equation
Differential form of Gauss's law,

Using

so that

This is Poisson equation. In cartesian form,

A formal soltion to Poisson equation can be written down by using the property Dirac - function discussed
earlier. It can be seen that

Operating with
to variable

operator on both sides (The subscrpt

We had shown that

substituting which the expression follows.


Equipotential surface

indicates that

here is to be taken with respect

Equipotential surfaces are defined as surfaces over which the potential is constant

At each point on the surface, the electric field is perpendicular to the surface since the electric field, being the
gradient of potential, does not have component along a surface of constant potential.

We have seen that any charge on a conductor must reside on its surface. These charges would move
along the surface if there were a tangential component of the electric field. The electric field must
therefore be along the normal to the surface of a conductor. The conductor surface is, therefore, an
equipotential surface.

Electric field lines are perpendicular to equipotential surfaces (or curves) and point in the direction from
higher potential to lower potential.

In the region where the electric field is strong, the equipotentials are closely packed as the gradient is
large.

Click here for Animation


The electric field strength at the point P may be found by finding the slope of the potential at the point P. If
is the distance between two equipotential curves close to P,

where

is the difference between the two equipotential curves near P.

Example 17
Determine the equipotential surface for a point charge.
Solution :

Let the point charge

be located at the origin. The equation to the equipotential surface is given by

Click here for Animation


Thus the surfaces are concentric spheres with the origin (the location of the charge) as the centre and radii
given by

The equipotential surfaces of an electric dipole is shown below.

Click here for Animation


Example 18

Determine the equipotential surface of an infinite line charge carrying a positive charge density

Solution :
Let the line charge be along the z- axis. The potential due to a line charge at a point P is given by

where

is the distance of the point P from the line charge. Since the line charge along the z-axis,
so that

The surface

is given by

i.e.

which represent cylinders with axis along the z-axis with radii

Click here for Animation


As

increases, radius becomes smaller. Thus the cylinders are packed closer around the axis, showing that

the field is stronger near the axis.


Exercise 1

Determine the equipotential surface of an infinite plane with charge density


Capacitance
Consider a spherical conductor of radius

carrying a charge

. The potential of the sphere is given by

The potential of the conductor is proportional to the charge it contains. This linear relationship is true in
general, independent of the shape of the conductor,

The constant of proportionality

is called the capacitance of the conductor. For the conducting sphere the

capacitance is

Unit of capacitance :
The M.K.S. unit of capacitance is Coulomb/Volt which is called a Farad. However, one Farad turns out to be very
large capacitance (the capacitance of the Earth is approximately 700 micro-Farad). A more practical unit of
capacitance is a micro-Farad (

) or a pico- Farad (pF) :

Capacitor :
A capacitor is essentially a device consisting of an arrangement of conductors for storing charges. As a
consequence, it also stores electrostatic energy. The simplest capacitor consists of two conductors, one carrying
a charge

and the other a charge

. Let

be the potential of the first conductor and

that of the

second. Since the conductor is an equipotential surface, the potential difference between the conductors
is also constant, and is given by

where the line integral is carried out along any path joining the two conductors. The electric field is proportional
to the charge

since if the charge on each conductor is multiplied by a constant

hence the electric field also gets multiplied by the same factor. Thus

, the charge density and

is proportional to the potential

difference

where
is the capacitance of the conductor pair.
A capacitor consisting of a single conductor (like the spherical conductor described above) may be considered to
be one part of a conductor pair where the second conductor containing the opposite charge is at infinity.
Parallel Plate Capacitor :
A parallel plate capacitor consists of two parallel metal plates, each of area
potential difference

separated by a distance

is maintained between the two plates. If the charge on the positive plate is

. A

and that

on the negative plate is

, the electric field in the region between the two plates is

Click here for Animation


The potential difference between the plates is

The capacitance

is

Spherical Capacitor :
The spherical capacitor consists of two concentric spherical conducting shells of radii

and

Click here for Animation


The electric field at a distance
radial and is given by

from the centre is calculated by using the Gaussian surface shown. The fileld is

The voltage drop between the shells is obtained by integrating the electric field along a radial path (the electric
field being conservative, the path of integration is chosen as per our convenience) from the negative plate to
the positrive plate.

The capacitance is

Cylindrical Conductor :
A cylindrical capacitor consists of two long coaxial conducting cylinders of length

and radii

and

. The

electric field in the space between the cylinders may be calculated by Gauss Law, using a pillbox in the shape of
a short coaxial cylinder of length

and radius

direction and depends only on the distance

. Neglecting edge effects, the field is in the radial

from the axis.

Click here for Animation


The contribution to the flux from the end caps of the pillbox is zero as the field is along the surface. The field at
a distance

where

is given by

is the charged enclosed by the pillbox, which is given in terms of the surface charge density

inner cylinder by

The field at a distance

is given by

The potential difference between the cylindrical conductors is

on the

Substituting

the capacitance is given by

Work Done in Charging a Capacitor :


Consider a parallel plate capacitor. The process of charging a capacitor consists of removing negative charges
(electrons) from the positive plate and depositing them on the negative plate.
, so that the potential difference between the

Suppose at a particular instant, the charge on the plates are


plates is

. To transport an infinitisimal charge

from the positive plate to the negative plate, the work

done by an external agency is

Total work done in charging the plates from

In terms of potential difference

to

is

This is the amount of energy stored in the capacitor.


One can also get the same expression by using the expression for the energy of a charge distribution derived
earlier

For a parallel plate capacitor

within the volume

of the capacitor and zero outside. Hence

Exercise 1
Obtain an expression for the energy of a spherical capacitor of radii

and

containing charges

[Ans.

Recap
In this lecture you have learnt the following
Poisson's equation relates the potential to charge density. A formal solution to Poisson's equation was
obtained.

A equipotential surface is one on which the potential is constant. The electric field on an equipotential
surface can only have component normal to the surface.

The potential of a conductor is proportional to the charge it contains, the constant of proportionality is
known as the capacitance of the conductor. A capacitor is a device to store charges and hence it also
stores electrostatic energy.

The capacitance for a parallel plate capacitor is proportional to the surface area and inversely
proportional to the separation between its plates.

Capacitance for spherical and cylindrical capacitors were calculated. The work done in charging a
capacitor was also calculated.

Module 2 : Electrostatics
Lecture 11 : Capacitance
Objectives
In this lecture you will learn the following
Capacitors in series and in parallel

Properties of dielectric

Conductor and dielectric in an electric field.

Polarization and bound charges

Gauss's Law for dielectrics


Capacitors in Combination :
Capacitors can be combined in series or parallel combinations in a circuit.
Parallel Combination
When they are in parallel, the potential difference across each capacitor is the same.

The

charge

on

each

capacitor
.

is

obtained

by

multiplying

with

the

capacitance,

i.e.

Since total charge in the capacitors is sum of all the charges, the effective capacitance of the combination is

Series Combination :
When capacitors are joined end to end in series, the first capacitor gets charged and induces an equal charge on
the second capacitor which is connected to it. This in turn induces an equal charge on the third capacitor, and
so on.

The net potential difference between the positive plate of the first capacitor and the negative plate of the last
capacitor in series is

The individual voltage drops are

so that

The effective capacitance is, therefore, given by

Example 19
Calculate the voltage across the 5

F capacitor in the following circuit.

Solution :
capacitors in series is equivalent to a 5

The equivalent circuit is shown above.The two 10


capacitor.5

in parallel with this equivalent capacitor gives 10

therefore consists of a 10

in series with the 20

combination, the potential drop across the 10


capacitor. The voltage drop across the 10

as the next equivalent.The circuit

capacitor. Since charge remains constant in a series

capacitor is twice as much as that across 20


(and hence across the given 5

) is

V.

Exercise 2
Determine the effective capacitance of the following capacitance circuit and find the voltage across each
capacitance if the voltage across the points a and b is 300 V.

[Ans. 8

F., 100V,200V,200V,200V,200V,100V]

Conductors and Dielectric


A conductor is characterized by existence of free electrons . These are electrons in the outermost shells of
atoms (the valence electrons) which get detatched from the parent atoms during the formation of metallic
bonds and move freely in the entire medium in such way that the conductor becomes an equipotential volume.
In contrast, in dielectrics (insulators), the outer electrons remain bound to the atoms or molecules to which
they belong. Both conductors and dielectric, on the whole, are charge neutral. However, in case of dielectrics,
the charge neutrality is satisfied over much smaller regions (e.g. at molecular level).
Polar and non-polar molecules :
A dielectric consists of molecules which remain locally charge neutral. The molecules may be polar or non-polar.
In non-polar molecules, the charge centres of positive and negative charges coincide so that the net dipole
moment of each molecule is zero. Carbon dioxide molecule is an example of a non-polar molecule.

Click here for Animation

Click here for Animation


In a polar molecules, the arrangement of atoms is such that the molecule has a permanent dipole moment
because of charge separation. Water molecule is an example of a polar molecule.
When a non-polar molecule is put in an electric field, the electric forces cause a small separation of the charges.
The molecule thereby acquires an induced dipole moment.
A polar molecule, which has a dipole moment in the absence of the electric field, gets its dipole moment aligned
in the direction of the field. In addition, the magnitude of the dipole moment may also increase because of
increased separation of the charges.

Click here for Animation


Conductor in an Electric Field
Consider what happens when a conductor is placed in an electric field, say, between the plates of a parallel
plate capacitor.

Click here for Animation


As the conductor contains free charges (electrons), these move towards the positive plate, making the surface
of the conductor closer to the positive plate of the capacitor negatively charged. These are called induced
charges. Consequently, the surface of the conductor at the end closer to the negative plate is positively
charged. The motion of charges continue till the internal electric field created by induced charges cancel the
external field, thereby making the field inside the conductor zero.
Dielectric in an Electric Field
A dielectric consists of molecules which may (polar) or may not (non-polar) have permanent dipole moment.
Even in the former case, the dipoles in a dielectric are randomly oriented because dipole energies are at best
comparable to thermal energy.

Click here for Animation 1

Click here for Animation 2

When a dielectric is placed in an electric field the dipoles get partially aligned in the direction of the field. The
charge separation is opposed by a restoring force due to attaraction between the charges until the forces are
balanced. Since the dipoles are partially aligned, there is a net dipole moment of the dielectric which opposes
the electric field. However, unlike in the case of the conductors, the net field is not zero. The opposing dipolar
field reduces the electric field inside the dielectric.
Dielectric Polarization
Electric polarization is defined as the dipole moment per unit volume in a dielectric medium. Since the
distribution of dipole moment in the medium is not uniform, the polarization
is the sum of the dipole moment vectors in a volume element

It can be checked that the dimension of

is a function of position. If

located at the position

is same as that of electric field divided by permittivity

. Thus the

source of polarization field is also electric charge, except that the charges involved in producing polarization are
bound charges .
Potential due to a dielectric
Consider the dielectric to be built up of volume elements

. The dipole moment of the volume element is

The potential at a point S, whose position vector with respect to the volume element is

is

The potential due to the whole volume is

where, we have used

Use the vector identity

Substituting

and

we get

The first integral can be converted to a surface integral using the divergence theorem giving,

The first term is the potential that one would expect for a surface charge density

where

where

is the unit vector along outward normal to the surface. The second term is the potential due to a

volume charge density

given by

The potential due to the dielectric is, therefore, given by

and the electric field

Gauss's Law in a Dielectric :


We have seen that the effect of polarization of a dielectric is to produce bound charges of volume density
and surface density
, given by

The total electric field of a system which includes dielectrics is due to these polarization charge densities and
other charges which may be present in the system. The latter are denoted as free charges to distinguish them
from charges attributable to polarization effect. For instance, the valence charges in a metal or charges of ions
embedded in a dielectric are considered as free charges.
The total charge density of a medium is a sum of free and bound charges

We can now formulate Gauss's law in the presence of a dielectric. Gauss's Law takes the form

Substituting

, we get

The electri displacement vector

is defined by

which has the same dimension as that of

The equation satisfied by

is thus,

which is the differential form of Gauss's law for a dielectric medium.


Integrating over the dielectric volume,

where

is the free charge enclosed in the volume. The volume integral can be converted to a surface

integral using the divergence theorem, which gives

Thus the flux over the vector

over a closed surface is equal to the free charged enclosed by the surface.

The above formulatons of Gauss's law for dielectric medium is useful because they refer to only free charges for
which we may have prior knowledge.
Constitutive Relation
Electric displacement vector
a relationship between

and

helps us to calculate fields in the presence of a dielectric. This is possible only if


is known.

For a weak to moderate field strength, the electric polarization


external electric field

. We define Electric Susceptibility

is found to be directly proportional to the

through

so that

where

is called the relative permittivity or the dielectric constant and

the medium. Using differential form of Gauss's law for

is the permittivity of

, we get

Thus the electric field produced in the medium has the same form as that in free space, except that the field
strength is reduced by a factor equal to the dielectric constant

Recap
In this lecture you have learnt the following
How to calculate effective capaciance when capacitors are in series and in parallel.

Dielectrics are material in which, unlike in conductors, the valence electrons are not detached from the
parent atoms.

In the absence of an electric field the dipoles in a dielectric have their moments directed randomly. In
the presence of an electric field, these dipoles get partially aligned in the direction of the field as a result
of which a dielectric acquires a net dipole moment.

The polarization effect can be attributed as arising out of bound charges in the dielectric. The charge
density in the medium consists of free and bound charges.

The net electric field in a dielectric is due to both polarization effect and the field produced by external
charges. One can define an electric displacement vector as a vector sum of fields due to polarization and
the electric field vector. Gauss's law can be modified appropriately for such a situation in a way that the
flux of the dsplacement vector is given by only the enclosed free charges.

Module 2 : Electrostatics
Lecture 12 : Conductors and Dielectric
Objectives
In this lecture you will learn the following
Properties of capacitors fielled with dielectric

Force on a dielectric

Calculation of effective capacitance for capacitor filled with different dielectrics


Capacitance filled with Dielectric
If a material of dielectric constant
factor

is inserted between the plates of a capacitor, the field

. The potential between the plates also reduces by the same factor

is reduced by a

Thus the capacitance

increases by a factor

Example 20
A parallel plate capacitor with plate separation 3.54mm and area 2m is initially charged to a potential
difference of 1000 volts. The charging batteries are then disconnected. A dielectric sheet with the same
thickness as that of the separation between the plates and having a dielectric constant of 2 is then inserted
between the capacitor plates. Determine (a) the capacitance , (b) potential difference across the capacitor
plates, (c) surface charge density (d) the electric field and (e) displacement vector , before and after the
insertion of the dielectric .
Solution :
(a) The capacitance before insertion of the dielectric is

After the insertion the capacitance doubles and becomes

F.

(b) Potential difference between the plates before insertion is given to be 1000 V. On introducing the dielectric
it becomes half, i.e. 500 V.

(c) The charge on each capacitor plate was


C/m

of

coulomb, giving a surface charge density

. The free charge density remains the same on introduction of the dielectric.

(d) The electric field strength

is given by

The electric field strength is reduced to

volt/meter on insertion.

(e) The displacement vector remains the same in both cases as the free charge density is not altered. It is
C/m

given by

Example 21
The parallel plates of a capacitor of plate dimensions
difference

and separation

are charged to a potential

and battery is disconnected. A dielectric slab of relative permittivity

is inserted between the

plates of the parallel plate capacitor such that the left hand edge of the slab is at a distance
most edge of the capacitor. Calculate (a) the capacitance and (b) the force on the dielectric.

from the left

Solution :
Since the battery is disconnected, the potential difference between the plates will change while the charge
remains the same. Since the capacitance of the part of the capacitor occupied by the dielectric is increased by a
factor

, the effective capacitance is due to two capacitances in parallel,

The energy stored in the capacitor is

Let

be the force we need to apply in the x-direction to keep the dielectric in place. For an infinitisimal
of

increment

, we have to do an amount of work


, so that

the field by

the differentiation is to be done, keeping the charge

Since

of work, which will increase the energy strored in

constant. Thus

is positive. This means the electric field pulls the dielectric inward so that an external agency

has to apply an outward force to keep the dielectric in position. Since the initial potential difference
by

is given

, one can express the force in terms of this potential

This is the force that the external agency has to apply to keep the left edge of the dielectric at
with which the capacitor pulls the dielectric in has the same magnitude.

. The force

Example 22
In the above example, what would be the force if the battery remained connected ?
Solution :
If the battery remained connected

does not remain the same, the potential

does. The battery must do

work to keep the potential constant. It may be realised that the force exerted on the dielectric in a particular
position depends on the charge distribution (of both free and bound charges) existing in that position and the
force is independent of whether the battery stays connected or is disconnected. However, in order to calculate
the force with battery remaining connected, one must, explicitly take into account the work done by the battery
in computing the total energy of the system. The total energy
external agency

and the other the work done by the battery, viz.,

supplied by the battery to keep the potential constant. Thus

which gives

Since

now has two parts, one the work done by the

is constant, we have

where

is the extra charge

Using these

(Note that if the work done by the battery were negnected, the direction of
will be wrong, though, because
we have used linear dielectrics, the magnitude, accidentally, turns out to be correct !)
In the previous example, we have seen that

giving

which is negative. Thus

is positive, as before,

Example 23
The space between the plates of a parallel plate capacitor is filled with two different dielectrics, as shown. Find
the effective capacitance.

Solution :
Take a Gaussian pill-box as shown. We have

as there are no free charges inside the dielectric. Contribution to the integral comes only from the faces of the
pill-box parallel to the plates and

. Hence,

where

Let

is the surface density of free charges.

be the potential difference between the upper plate and the interface between the dielectric and

that

between the interface and the lower plate. We have

Thus the effective capacitance is given by

where
and

and

are the capacitances for parallel plate capacitors with one type of dielectric with separations

between the plates respectively.

Example 24
A capacitor consists of an inner conducting sphere of radius

and an outer conducting shell of radius

The space between the spheres is filled with two different linear dielectrics, one with a dielectric constant

from

to

and the other with dielectric constant

from

while the inner conductor has a charge

outer shell has a charge

to

. The

. Determine the electric field for

and find the effective capacitance.


Solution :
The electric field is radially symmetric and may be obtained by applying Gauss's law for the displacement vector

where

is the free charge enclosed within a sphere of radius

. For

, the field is zero as the free

charges are only on the surface of the inner cylinder.

the electric field is

and for

For

, the field is zero. The fields are radial with the ineer sphere at a higher potential. The potential

difference is calculated by taking the taking the line integral of the electric field along any radial line.

The effective capacitance is

Example 25
A parallel plate capacitor has charge densities

on its plates which are separated by a disance

. The space

between the capacitor plates is filled with a linear but inhomogeneous dielectric. The dielectric constant varies
with distance from the positive plate linearly from a value 1 to a value 2 at the negative plate. Determine the
effective capacitance.
Solution :
As the dielectric is linear,

As the insertion of dielectric does not affect free charges, the displacement vector
would in the absence of the dielectric. Thus

Thus the electric field

is given by

remains the same as it

The field close to

is given by

positive charge density

, which shows that adjacent to the negative plate there is a

. To find the effective capacitance, we find the potential difference between the

plates by integrating the electric field

so that

The polarization

is given by

The volume density of bound charges, given by

is found as follows :

The bound charge density on the surface, given by

, has a value

to the negative plate (

on the dielectric adjacent

). As the dielectric is charge neutral, this requires a net volume charge of

in the dielectric. This can be verified by integrating over the volume charge density

given above.

Exercise
A parallel plate capacitor of plate area

and separation

, contains a dielectric of thickness

and of

dielectric constant 2, resting on the negatve plate.

A potential difference of

is maintained between the plates. Calculate the electric field in the region between

the plates and the density of bound charges on the surface of the dielectric.

[Ans. field in empty region

, within dielectric

, bound charge density

The permittivity of a medium filling the space between the plates of a spherical capacitor with raddi

]
and

) is given by

Find the capacitance of the capacitor, distribution of surface bound charges and the total bound charges in the
dielectric.
[Ans.

and

, bound charges on dielectric surface with radii

are respectively

and

Recap
In this lecture you have learnt the following
When a dielectric is inserted inside a dielectric, the electric fieldin the capacitor is reduced by a factor
known as the dielectric constant.

The capacitance of a dielectric is increased by insertion of a dielectric.

The dielectric experiences force in the capacitor due to the electric field.

Effective capacitance of various combinations of capacitors filled with dielectric were calculated.

Module 3 : Electromagnetism
Lecture 13 : Magnetic Field
Objectives
In this lecture you will learn the following
Electric current is the source of magnetic field.

When a charged particle is placed in an electromagnetic field, it experiences a force, called Lorentz force.

Motion of a charged particle in a magnetic field and concept of cyclotron frequency.

Find the trajectory of a charged particle in crossed electric and magnetic field.
MAGNETIC FIELD
Electric charges are source of electric fields. An electric field exerts force on an electric charge, whether the
charge happens to be moving or at rest.
One could similarly think of a magnetic charge as being the source of a magnetic field. However, isolated
magnetic charge ( or magnetic monopoles) have never been found to exist. Magnetic poles always occur in
pairs ( dipoles) - a north pole and a south pole. Thus, the region around a bar magnet is a magnetic field.
What characterizes a magnetic field is the qualitative nature of the force that it exerts on an electric
charge. The field does not exert any force on a static charge. However, if the charge happens to be moving
(excepting in a direction parallel to the direction of the field) it experiences a force in the magnetic field.
It is not necessary to invoke the presence of magnetic poles to discuss the source of magnetic field.
Experiments by Oersted showed that a magnetic needle gets deflected in the region around a current
carrying conductor. The direction of deflection is shown in the figure below.

Thus a current carrying conductor is the source of a magnetic field. In fact, a magnetic dipole can be
considered as a closed current loop.
Electric Current and Current Density
Electric current is the rate of flow of charges in electrical conductor. In a conductor the charges may have
random motion. However, the net drift velocity of the charges is zero, giving a zero net current. In the
presence of an external force field, the charges move with a net non-zero drift velocity, which gives a
current.
The direction of current has been defined, conventionally, as the direction in which the positive charges
move. In case of metallic conductors, the current is caused by flow of negatively charged electrons, whose
direction of motion is opposite to the direction of current. In electrolytes, however, the current is due to
flow of both positive and negative ions.
The current density

at a position

is defined as the amount of charge crossing a unit cross-

sectional area per unit time. In terms of the net drift velocity of the charges (taken opposite to the net drift
velocity of electrons),

where

is the volume density of the mobile charges.

The integral of the current density over a surface defines electric current, which is a scalar.

The unit of electric current is Ampere (= coulomb/sec) and that of the current density is A/m
wire with a small cross sectional area, the current density

where

. For a thin

may be taken as uniform. In this case,

is the component of the area vector parallel to the direction of current.

The Right Hand Rule


The direction of magnetic field due to a current carrying conductor is given by the right hand rule.
If one clasps the conductor with one's right hand in such a way that the thumb points in the direction of
the current (i.e. in the direction opposite to the direction of electron flow) then, the direction in which the
fingers curl gives the direction of the magnetic field due to such a conductor.

The Lorentz Force


We know that an electric field
, a charge

where

exerts a force

experiences an additional force

is the velocity of the charge. Note that


There is no force on a charge at rest.

on a charge

. In the presence of a magnetic field

A force is exerted on the charge only if there is a component of the magnetic field perpendicular to
the direction of the velocity, i.e. the component of the magnetic field parallel to
contribute to

does not

, which shows that the magnetic force does not do any work.

In the case where both

and

are present, the force on the charge

is given by

This is called Lorentz force after H.E. Lorentz who postulated the relationship. It may be noted that the
force expression is valid even when

and

are time dependent

Unit of Magnetic Field


From the Lorentz equation, it may be seen that the unit of magnetic field is Newton-second/coulombmeter, which is known as a Tesla (T). (The unit is occasionally written as Weber/m

as the unit of

magnetic flux is known as Weber). However, Tesla is a very large unit and it is common to measure
terms of a smaller unit called Gauss,

in

It may be noted that


is also referred to as magnetic field of induction or simply as the induction field.
However, we will use the term ``magnetic field".
Motion of a Charged Particle in a Uniform Magnetic Field
Let the direction of the magnetic field be taken to be z- direction,

we can write the force on the particle to be

The problem can be looked at qualitatively as follows. We can resolve the motion of the charged particle
into two components, one parallel to the magnetic field and the other perpendicular to it. Since the motion
parallel to the magnetic field is not affected, the velocity component in the z-direction remains constant.

where

is the initial velocity of the particle. Let us denote the velocity component perpendicular to the

direction of the magnetic field by

. Since the force (and hence the acceleration) is perpendicular to the

direction of velocity, the motion in a plane perpendicular to


sustain the circular motion is provided by the Lorentz force

is a circle. The centripetal force necessary to

where the radius of the circle

is called the Larmor radius, and is given by

The time taken by the particle to complete one revolution is

The cyclotron frequency

is given by

Motion in a Magnetic Field - Quantitative


Let the initial velocity of the particle be
to

as the

. we may take the direction of the component of

perpendicular

direction, so that

Let the velocity at time

be denoted by

we can express the force equation in terms of its cartesian component

where we have used

and

The last equation tells us that no force acts on the particle in the direction in which

acts, so that

The first two equations may be solved by converting them into second order differential equatons. This is
done by differentiating one of the equations with respect to time and substituting the other equation in the
resulting second order equation. For instance, the equation for

is given by

The equation is familiar in the study of simple harmonic motion. The solutions are combination of sine and
cosine functions.

where

is called the cyclotron frequency and


and
are constants. These constants have to be determined from
initial conditions. By our choice of x and y axes, we have

so that

Since

. Differentiating the above equation for

at

which shows that


difference of

, we have

and

. Thus, the velocity components at time

are given by

vary harmonically with time with the same amplitude but with a phase

. Equation of the trajectory may be obtained by integrating the equations for velocity

components

where

and

are constants of integration representing the initial position of the particle. The

equation to the projection of the trajectory in the x-y plane is given by

which represents a circle of radius

, centered about

. As the z- component of the velocity

is constant, the trajectory is a helix.

A plot of the motion of a charged particle in a constant magnetic field.


Motion in a crossed electric and magnetic fields
The force on the charged particle in the presence of both electric and magnetic fields is given by

Let the electric and magnetic fields be at right angle to each other, so that,

If the particle is initially at rest no magnetic force acts on the particle. As the electric field exerts a force on
the particle, it acquires a velocity in the direction of
particle.

. The magnetic force now acts sidewise on the

For a quantitative analysis of the motion, let


be taken along the x-direction and
along z-direction. As
there is no component of the force along the z-direction, the velocity of the particle remains zero in this
direction. The motion, therefore, takes place in x-y plane. The equations of motion are

(1)

(2)

As in the earlier case, we can solve the equations by differentiating one of the equations and substituting
the other,

which, as before, has the solution

with

Since

The constant

. Substituting this solution into the equation for

at

, the constant

, we get

, so that

may be determined by substituting the solutions in eqn. (1) which gives

Since the equation above is valid for all times, the constant terms on the right must cancel, which gives
. Thus we have

The equation to the trajectory is obtained by integrating the above equation and determining the constant
of integration from the initial position (taken to be at the origin),

The equation to the trajectory is

which represents a circle of radius

whose centre travels along the negative y direction with a constant speed

The trajectory resembles that of a point on the circumference of a wheel of radius


axis without slipping with a speed

, rolling down the y-

. The trajectory is known as a cycloid.

Exercise 1
Find the maximum value of
attained by the particle during the cycloidal motion and determine the speed
of the particle at such points.
(Ans.

speed

.)

Recap
In this lecture you have learnt the following
Just as a static electric charge is source of an electric field, the source of magnetic field is a moving charge,
i.e., electric current.
When a charged particle is is placed in a magnetic field, it experiences a velocity dependent force. The force
acts perpendicular to the direction of motion of the charge. it experiences a force, called Lorentz force.
The trajectory of a charged particle moving in a umiform magnetic field is circular.
The time period of revolution of a charged particle in a magnetic field depends on the strength of the
magnetic field and the charge to mass ratio of the particle.

In a crossed electric and magnetic field a charged particle moves in a helical path.

Module 3 : MAGNETIC FIELD


Lecture 14 : Force on a Current Carrying Conductor
Objectives
In this lecture you will learn the following
Calculate the force on a current carrying conductor in a magnetic field.

Find the torque on a current loop.

Define magnetic dipole and its magnetic moment.

Determine potential energy of a magnetic dipole

Force on a Current Carrying Conductor


A conductor has free electrons which can move in the presence of a field. Since a magnetic field exerts a force
on a charge moving with a velocity

, it also exerts a force on a conductor carrying a current.

Consider a conducting wire carrying a current

. The current density at any point in the wire is given by'

where
point.

is the number density of electrons having a charge

each and

is the average drift velocity at that

Consider a section of length

of the wire. If

perpendicular to the direction of

, the force on the electrons in this section is

where

is the amount of charge in the section

Thus the force on the conductor in this section is

is the cross sectional area of the section oriented

If

represents a vector whose magnitude is the length of the segment and whose direction is along the

direction of

, we may rewrite the above as

The net force on the conductor is given by summing over all the length elements. If

denotes a unit vector

in the direction of the current, then

Example 1
Force on a Straight Wire carrying current
If the wire is straight

is a constant vector. For a uniform magnetic field

The wire experiences a sidewise force. The direction of the force is given by Fleming's Left Hand Rule.
Fleming's Left Hand Rule
If one stretches out the forefinger, the middle finger and the thumb of one's left hand so that they are
mutually perpendicular to one another, then, if the middle finger points in the direction of current and the
forefinger points in the direction of the magnetic field, the thumb will point in the direction of the force.

Example 2
Force on a semi-circular current element
Consider a semicircular element in the plane of the paper ( the current enters the page at M along a direction
parallel to the negative z-axis and leaves at N in a direction parallel to the positive z-axis). These incoming
and outgoing sections of the conductor do not experience any force as
direction of the current in these.

is parallel (or anti-parallel) to the

The length element


vector

, the vector

. Since the current direction opposite to the direction of the tangential unit
is

Thus the force is

The force on the semicircular arc is obtained by integrating the above. By symmetry, the force on a current
element situated at a point Q on the second quadrant, situated symmetrically with P, has the same magnitude
and is directed along the radial direction at Q. On resolving the forces along the x and y directions, the
components parallel to the x-axis cancel for such symmetric pairs while the component parallel to the y- axis
add up. Thus

The magnitude of the force is seen to be equal to the product of

with the diameter of the semi-circle.

Example 3
Force on a conductor of arbitrary shape in a uniform field
For an wire of arbitrary shape,
Now,

is the sum of vectors along the curve from the initial point M to the final point N. By law of

vector addition, the sum is equal to the vector


Thus

where

connecting the two end points by a straightline segment.

is the vector connecting the two end points. (Using this, the result of the previous example trivially

follows).

Corollary : The force on a closed current loop in a constant magnetic field is zero.
Torque on a Current Loop in a Uniform Magnetic Field
Though the net force on a closed current loop in a uniform magnetic field is zero, it experiences a torque.
Consider a rectangular current loop PQRS of length

and width

. The loop is in a uniform magnetic field

which acts parallel to the x-axis. The loop, which is pivoted about an axis OO', carries a current
direction shown in the figure.

along the

The plane of the loop (i.e. the normal


to the loop) makes an angle
to the direction of the field. We take
the shorter sides PQ and RS (as well as the pivot axis OO') to be perpendicular to the field direction, OO' being
taken as the y-axis. The longer sides QR and QS make an angle
a current segment is

with the field direction. Since the force on

, the force is directed perpendicular to both

current in these segments. The force has a magnitude

and to the direction of the

and is directed oppositely on the two

sides. These forces are labelled

and

in the figure. (The forces are actually distributed along the lengths

and the cancellation occurs for the forces acting on symmetrically placed elements on these two arms.)
Further, since the lines of action of the forces acting on corresponding elements on these two sides are the
same, there is no torque.
and

The forces acting on the sides PQ and RS (labelled


have magnitude

respectively) are also equal and opposite and

. However, these forces do not act along the same line. The force on PQ acts parallel to

axis while that on RS acts parallel to

axis. Note that

axis is not in the plane of the loop. The

situation can be better visualized by redrawing the figure in the plane containing one of the longer sides and
.
The current enters the branch SR at S marked with

and reenters from QP at P, marked with

. The

directions of the forces on the branches SR and QP are shown. The magnitude of the torque about the pivot is

where

is the area of the loop.

The result above is independent of the shape of the loop. The following example for a circular loop gives an
identical result.
Example 4
A circular loop of radius

carrying a current

along the y-axis. A uniform magnetic field

is in the x-y plane. The loop is pivoted about its diameter

acts parallel to the x-axis. Calculate the torque on the loop.

Solution :
Consider a length element
into the plane of the figure.
The force on the element is

at an angle

to the y-axis. The force on the element is

is

The torque due to this force about the y-axis is

The net force on the loop is zero because of equal and opposite forces acting on diametrically opposite
elements of the loop. However, the torques due to all the elements are parallel to the y-direction and add up.
The net torque is obtained by integrating over all the elements

where

is the area of the loop.

Exercise 1
An coil of 10 turns is in the shape of an equilateral triangle of side 4 cm. The coil is in a magnetic field of
strength 0.01 T parallel to the plane of the loop. The current in the coil is 0.1 A. Find the torque acting on the
coil.
[Ans.

N-m]

Potential Energy of a Magnetic Dipole


A current loop does not experience a net force in a magnetic field. It however, experiences a torque. This is
very similar to the behaviour of an electric dipole in an electric field. A current loop, therefore, behaves like a
magnetic dipole.
We define the magnetic dipole moment

of a current loop to be a vector of magnitude

and direction perpendicular to the plane of the loop (as determined by right hand rule). If the loop has
turns,

. In a magnetic field, the dipole experiences a torque

The form of torque suggests that in a magnetic field the dipole tends to align parallel to the field. If the
orientation of the dipole is at some angle

to the field, there must be some potential energy stored in the

dipole. This is because, if we wish to bring the dipole from


oppose the torque due to the field and do work in the process.

to some arbitrary angle

, we have to

The work done is given by

This amount of work is stored as the additional potential energy of the dipole. In analogy with the case of
electric dipole in an electric field, the potential energy of the magnetic dipole in a magnetic field is given by

The energy is lowest when

and

are along the same direction and is maximum when they are anti-

parallel.
Recap
In this lecture you have learnt the following
A conductor carrying current experiences a force in a magnetic field.
A current loop is equivalent to a magnetic dipole with a magnetic moment equal to the product of the current
and the area of the loop.
A current loop does not experience a force in a magnetic field but experiences a torque which tends to align
the dipole moment parallel to the direction of the field.
A magnetic dipole has a potential energy in a magnetic field.

Module 3 : MAGNETIC FIELD


Lecture 15 : Biot- Savarts' Law
Objectives
In this lecture you will learn the following
Study Biot-Savart's law

Calculate magnetic field of induction due to some simple current configurations.

Define magnetic moment of a current loop.

Find force between two current carrying conductors.

Biot- Savarts' Law


Biot-Savarts' law provides an expression for the magnetic field due to a current segment. The field
due to a current segment
proportional to the length

is experimentally found to be perpendicular to


and to the current

inversely proportional to the square of the distance

and

at a position

. The magnitude of the field is

and to the sine of the angle between

and

of the point P from the current element.

Mathematically,

In SI units the constant of proportionality is

, where

The expression for field at a point P having a position vector

is the permeability of the free space. The value of

with respect to the current element is

is

For a conducting wire of arbitrary shape, the field is obtained by vectorially adding the contributions due to such current
elements as per superposition principle,

where the integration is along the path of the

current flow.
Example 5
Field due to a straight wire carrying current
The direction of the field at P due to a current element

is along

, which is a vector normal to the page (figure

on the left) and coming out of it.

We have,

where the plane of the figure is taken as the x-y plane and the direction of outward normal is parallel to z-axis. If
the distance of the oint P from the wire,

we have

Thus

be

The direction of the magnetic field at a distance

from the wire is tangential to a circle of radius

,as shown.

Since the magnetic field due to all current elements at P are parallel to the z-direction, the field at P due to a wire, the
ends of which make angles

Note that both the angles

and

and

at P is given by a straightforward integration

are acute angles.

If we consider an infinite wire (also called long straight wire), we have


a wire is

, so that the field due to such

where the direction of the field is given by the Right hand rule.
Exercise 1
A conductor in the shape of an n-sided polygon of side
field at the centre of the polygon.
[ Ans.

carries current

. Calculate the magnitude of the magnetic

.]

Example 6
Field due to a circular coil on its axis
Consider the current loop to be in the x-y plane, which is taken perpendicular to the plane of the paper in which the axis
, the
to the loop (z-axis) lies. Since all length elements on the circumference of the ring are perpendicular to
magnitude of the field at a point P is given by

The direction of the field due to every element is in the plane of the paper and perpendicular to
Corresponding to every element
gives a magnetic field

, as shown.

on the circumference of the circle, there is a diametrically opposite element which

in a direction such that the component of

perpendicular to the axis cancel out in pairs.

The resultant field is parallel to the axis, its direction being along the positive z-axis for the current direction shown in the
figure. The net field is

In terms of the distance

of the point P and the radius

, we have

The direction of the magnetic field is determined by the following Right Hand Rule.

If the palm of the right hand is curled in the direction of the current, the direction in which the thumb points gives the
direction of the magnetic field at the centre of the loop. The field is, therefore, outward in the figure shown.
, i.e. the field due to circular loop at large distances is given by

Note that for

where

is the magnetic moment of the loop. The formula is very similar to the field of an electric dipole. Thus

a current loop behaves like a magnetic dipole.


Example 7

A thin plastic disk of radius


angular velocity

has a uniform surface charge density

. Find the field at a distance

. The disk is rotating about its own axis with an

along the axis from the centre of the disk.

The current on the disk can be calculated by assuming the rotating disk to be equivalent to a collection of concentric
current loops. Consider a ring of radius

and of width

. As the disk is rotating with an angular speed

charge on the ring essentially behaves like a current loop carrying current
The field at a distance

due to this ring is

, the rotating
.

The net field is obtained by integrating the above from

The integral above may easily be evaluated by a substitution

The field at the centre of the disk (

to

. The result is

) is

Exercise 2
Find the magnetic moment of the rotating disk of Example 7.
[Ans.

Example 8
Two coaxial circular coils of radius
by a distance

each carry current I each in the same sense. The centres of the coils are separated

. Determine the field along the axis. The set up is called ``Helmholtz coil" when the distance

between the centres of the coils equals the radius


a coil is nearly uniform.

of each of the loop. The field in the region between the coils of such

If the distance
along the axis is measured from the mid points of the line joining the centres of the two coils, the field
strength due to the left coil at P is

and that due to the right coil is

The net field at P, due to both coils add up and is given by

We can express the above in a power series using a binomial expansion. Up to


written as

and a similar expression for the second term with


powers of

replacing

, the terms in the expansion may be

. Adding the terms and retaining only up to

we get

For the case of a Helmholtz coil,


given by,

, and the expression for field is independent of

It can be seen that the field along the axis is nearly uniform in the region between the coils.

up to its third power, and is

Example 9
Consider a solenoid of
turns. The solenoid can be considered as stacked up circular coils. The field on the axis of the
solenoid can be found by superposition of fields due to all circular coils. Consider the field at P due to the circular turns
between

and

from the origin, which is taken at the centre of the solenoid. The point P is at

the length of the solenoid, the number of turns within

and

is

, where

turns per unit length.


The magnitude of the field at P due to these turns is given by

The field due to each turn is along

; hence the fields due to all turns simply add up. The net field is

. If

is

is the number of

The integral above is easily evaluated by substituting

The limits of integration on

are

and

as shown in the figure. With the above substitution

For a long solenoid, the field on the axis at points far removed from the ends of the solenoid may be obtained by
substituting

and

, so that,

The field is very nearly constant. For points on the axis far removed from the ends but outside the solenoid,

so

that the field is nearly zero.


Example 10
Determine the field at the point located at the centre P of the semi-circular section of the hairpin bend shown in the
figure.

Solution :
The field at P may be determined by superposition of fields due to the two straight line sections and the semicircular arc.
The contribution due to all three sections add up as the field due to each is into the plane of the paper.

The field due to each straight line section is obtained by putting


Example 5 above. The field due to each wire is

and

in the expression obtained in

For the semi-circular arc, each length element on the circumference is perpendicular to

, the vector from the length

element to the point P. Thus

The net field due to the current in the hairpin bend at P is

Example 11
Force due to the first wire at the position of the second wire is given by
where

is a unit vecor out of the page. The force experienced by the second wire in this field is

Thus the force between the wires carrying current in the same direction is attractive and is

per unit

length. A generalization of the above is given by the mathematical expression for the force between two arbitrary current
loops.

where

is the position vector of the element

with respect to

Once the integration is carried out, the expression above can be shown to be symmetrical between the two circuits. To
show, we express the vector triple product

so that

The

integrand

in

the

first

integral

is

an

exact

differential

with

respect

to

the

integral over

The integral above being an integral of a gradient over a closed path vanishes. Thus

which is explicitly symmetric between the two circuits, confirming the validity of Newton 's third law.
Exercise 3
Determine the magnetic field at the point P for the two geometries shown in the figures below.

as

[Ans . (a)

(b)

Recap
In this lecture you have learnt the following
The magnetic field due to a current element is determined by Biot-Savart's law.

The magnetic field due to some configurations like a line segment, a circular coil, a disk etc. was calculated
using Biot-savart's law.
A Helmholtz coil is used to produce a uniform magnetic field over a limited region of space.

A force is exerted on a current element placed in a magnetic field.

Two current carrying circuits exert force on each other because the magnetic field due to one circuit exerts force on
the current elements of the other circuit.

Module 3 : MAGNETIC FIELD


Lecture 16 : Ampere's Law
Objectives
In this lecture you will learn the following
Establish Ampere' law in integral form.

Calculate the magnetic field for certain current configuration using Ampere's law.

Derive the differential form of Ampere's law.

Ampere's Law
Biot-Savart's law for magnetic field due to a current element is difficult to visualize physically as such
elements cannot be isolated from the circuit which they are part of. Andre Ampere formulated a law based on
Oersted's as well as his own experimental studies. Ampere's law states that `` the line integral of magnetic
field around any closed path equals

times the current which threads the surface bounded by such closed

path. . Mathematically,

In spite of its apparent simplicity, Ampere's law can be used to calculate magnetic field of a current
distribution in cases where a lot of information exists on the behaviour of
. The field must have enough
symmetry in space so as to enable us to express the left hand side of (1) in a functional form. The simplest
application of Ampere' s law consists of applying the law to the case of an infinitely long straight and thin wire.
Example 12
By symmetry of the problem we know that the magnitude of the field at a point can depend only on the
distance of the point from the wire. Further, the field is tangential to the circle of radius
given by the right hand rule.
Thus the integral around the circle is

Equating this to

, we get

which is consistent with the result obtained from Biot-Savart's law.

, its direction being

Let us see if the result above is consistent with a path which is not circular, as shown in the figure. The field
at every element

of the path is perpendicular to

. From geometry, it can be seen that

Thus

We need to specify the direction along which the path is traversed. This is done by Right Hand Rule. If we curl
the fingers of our right hand along the path of integration, the direction along which the thumb points is the
direction of current flow.

For the case where the path of integration lies totally outside the path of the current, for every element

at

P, there exists another element at P' for which


taken, the contributions from such pairs add to zero

has opposite sign. Thus when complete line integral is

Combining these, we get Ampere's law in the form of Eqn. (1)

Example 13
Calculate the field due to a uniform current distribution in an infinite wire of cross sectional radius

Solution :
Let the cross section of the wire be circular with a radius
. Take the current direction to be perpendicular to
the page and coming out of it. Symmetry of the problem demands that the magnitude of the field at a point is
dependent only on the distance of the point from the axis of the wire. Consider an amperian loop of radius
As before we have

If

(as in loop 1), the entire current is enclosed by the loop. Hence

If

(loop 2), the current enclosed is proportional to the area, i.e.

so that

so that

The field distribution with distance is as shown.


Exercise 1
A long wire of cross sectional radius

carries a current

. The current density varies as the square of the

distance from the axis of the wire. Find the magnetic field for

( Hint : First show that the current density


for

. Answer :

for

and for

and obtain an expression for current enclosed


and

for

.)

Exercise 2
A hollow cylindrical conductor of infinite length carries uniformly distributed current I from
Determine magnetic field for all

(Answer : Field is zero for

for

and

for

.)
Exercise 3
A coaxial cable consists of a solid conductor of radius
radius

with a concentric shell of inner radius

and outer

. The space between the solid conductor and the shell is supported by an insulating material.

A current
goes into the inner conductor and is returned by the outer shell. Assume the cuurent densities to
be uniform both in the shell and in the inner conductor. Calculate magnetic field everywhere.
(Ans.

inside the inner conductor,

conductor,

between the shell and the inner

Exercise 4
Determine the magnetic field in a cylindrical hole of radius

inside a cylindrical conductor of radius

cylinders are of infinite length and their axes are parallel, being separated by a distance
carries a current

. The

. The conductor

of uniform density.

(Hint : The problem is conveniently solved by imagining currents of equal and opposite densities flowing in the
hole and using superposition principle to calculate the field. Answer : The field inside the hole is constant
)

Example 14
We take the solenoid to be closely wound so that each turn can be considered to be circular. We can prove
that the field due to such a solenoid is entirely confined to its interior, i.e. the field outside is zero, To see this
consider a rectangular amperian loop parallel to the axis of the solenoid.

Field everywhere on AB is constant and is

. Likewise the field everywhere on CD is

. By Right

hand rule, the field on AB is directed along the loop while that on CD is oppositely directed. On the sides AD
and BC, the magnetic field direction is perpendicular to the length element and hence
everywhere on these two sides. Thus

is zero

By Ampere's law, the value of the integral is zero as no current is enclosed by the loop. Thus
. The field outside the solenoid is, therefore, independent of the distance from the axis of the
solenoid. However, from physical point of view, we expect the field to vanish at large distances. Thus
.
To find the field inside, take an amperian loop EFGH with its length parallel to the axis as before, but with one
of the sides inside the solenoid while the other is outside. The only contribution to

comes from the

side GH. Thus,

where

is the current through each turn and

because the number of turns threading the loop is

is number of turns per unit length.


. Hence,

is independent of the distance

from the axis.


Exercise 5
A toroid is essentially a hollow tube bent in the form of a circle. Current carrying coils are wound over it. Use
an amperian loop shown in the figure to show that the field within the toroid is
number of turns and

the circumference of the circular path.

, where

is the

Note that as the circumference of the circular path varies with the distance of the amperian loop from the
toroid axis, the magnetic field in the toroid varies over its cross section. Take the inner radius of the toroid to
be 20cm and the outer radius as 21cm. Find the percentage variation of the field over the cross section of the
toroid.
(Ans. 2.9%)
Example 15
An infinite conducting sheet carries a current such that the current density is
amperian loop as shown.

The contribution to the line integral of


PQ and RS the direction of
.

Exercise 6

from the sides QR and SP are zero as

is parallel to the path. Hence

per unit length. Take an

is perpendicular to

. For
giving

Calculate the force per unit area between two parallel infinite current sheets with current densities

and

in the same direction.

( Ans.

Ampere's Law in Differential Form


We may express Ampere's law in a differential form by use of Stoke's theorem, according to which the line
integral of a vector field is equal to the surface integral of the curl of the field,

The surface

is any surface whose boundary is the closed path of integration of the line integral.

In terms of the current density

where

, we have,

is the total current through the surface

. Thus, Ampere's law

is equivalent

to

which gives
You may recall that in the case of electric field, we had shown that the divergence of the field to be given by
. In the case of magnetic field there are no free sources (monopoles). As a result the
divergence of the magnetic field is zero

The integral form of above is obtained by application of the divergence theorem

Thus the flux of the magnetic field through a closed surface is zero.
Recap
In this lecture you have learnt the following

Ampere's law was stated in integral form and used to calculate magnetic field in symmetric situations.

Calculation of magnetic field was done due to a a long straight wire, a coaxial cable, a solenoid, a toroid and a
current sheet done using Ampere's law.
Using Stoke's theorem Ampere's law can be expressed in a differential form.

Module 3 : MAGNETIC FIELD


Lecture 17 : Vector Potential
Objectives
In this lecture you will learn the following
Define vector potential for a magnetic field.

Understand why vector potential is defined in a gauge.

Calculate vector potential for simple geometries.

Define electromotive force and state Faraday's law of induction


Vector Potential
For the electric field case, we had seen that it is possible to define a scalar function
whose negative gradient is equal to the electric field :

called the ``potential"

. The existence of such a scalar function is a

consequence of the conservative nature of the electric force. It also followed that the electric field is
irrotational, i.e.
.
For the magnetic field, Ampere's law gives a non-zero curl

Since the curl of a gradient is always zero, we cannot express


then violate Ampere's law.
However, we may introduce a vector function

This would automatically satisfy

as a gradient of a scalar function as it would

such that

since divergence of a curl is zero.

is known as vector

potential . Recall that a vector field is uniquely determined by specifying its divergence and curl. As

is a

physical quantity, curl of


is also so. However, the divergence of the vector potential has no physical
meaning and consequently we are at liberty to specify its divergence as per our wish. This freedom to choose
a vector potential whose curl is

and whose divergence can be conveniently chosen is called by

mathematicians as a choice of a gauge . If

is a scalar function any transformation of the type

gives the same magnetic field as curl of a gradient is identically zero. The transformation above is known as
gauge invariance . (we have a similar freedom for the scalar potential

of the electric field in the sense that

it is determined up to an additive constant. Our most common choice of

is one for which

at infinite

distances.)
A popular gauge choice for

is one in which

which is known as the ``Coulomb gauge". It can be shown that such a choice can always be made.
Exercise 1
Show that a possible choice of the vector potential for a constant magnetic field
. Can you construct any other

(Hint : Take

in z-direction, express

is given by

in component form and take its curl.)

Biot-Savart's Law for Vector Potential


Biot-Savart's law for magnetic field due to a current element

may be used to obtain an expression for the vector potential. Since the element

does not depend on the

position vector of the point at which the magnetic field is calculated, we can write

the change in sign is because

Thus the contribution to the vector potential from the element

is

The expression is to be integrated over the path of the current to get the vector potential for the system

Example 16
Obtain an expression for the vector potential at a point due to a long current carrying wire.
Solution :
Take the wire to be along the z-direction, perpendicular to the plane of the page with current flowing in a
direction out of the page. The magnitude of the field at a point P is
the tangential unit vector

The direction of

with its direction being along

at P,

makes an angle

with the x direction, where

. Thus

Hence we have

We wish to find a vector function


function fits the requirement

whose curl is given by the above. One can see that the following

In the following, we will derive this directly from the expression for Biot-Savart's law. If
from an element of length

at

is the distance of P

of the wire, we have,

Thus

If the above integral is evaluated from

to

, it diverges. However, we can eliminate the

infinity in the following manner. Let us take the wire to be of length

so that

The integral is evaluated by substituting

where

In terms of

, so that

. We get

and

, we have

Thus to leading order in

As expected, for

, the expression diverges. However, since

is, the constant term (which diverges in the limit of


given by

itself is not physical while curl of


) is of no consequence and

is

which is the same as Eqn. (1)


Example 17
Obtain an expression for the vector potential of a solenoid.
Solution :
We had seen that for a solenoid, the field is parallel to the axis for points inside the solenoid and is zero

outside.

Take a circle of radius

Since

is axial,

perpendicular to the axis of the solenoid. The flux of the magnetic field is

is directed tangentially to the circle. Further, from symmetry, the magnitude of

is constant on the circumference of the circle.


Use of Stoke's theorem gives

Thus

where

is the unit vector along the tangential direction.

Exercise 2
Obtain an expression for the vector potential inside a cylindrical wire of radius
(Ans.

carrying a current

The existence of a vector potential whose curl gives the magnetic field directly gives

as the divergence of a curl is zero. The vector identity

can be used to express Ampere's law in terms of vector potential. Using a Coulomb gauge in which
, the Ampere's law

is equivalent to

which is actually a set of three equations for the components of

, viz.,

which are Poisson's equations.


Electromagnetic Induction :
We have seen that studies made by Oersted, Biot-Savart and Ampere showed that an electric current
produces a magnetic field. Michael Faraday wanted to explore if this phenomenon is reversible in the sense
whether a magnetic field could be source for a current in a conductor. However, no current was found when a
conductor was placed in a magnetic field. Faraday and (Joseph) Henry, however, found that if a current loop
was placed in a time varying magnetic field or if there was a relative motion between a magnet and the loop a
transient current was established in the conducting loop. They concluded that the source of the electromotive
force driving the current in the conductor is not the magnetic field but the changing magnetic flux associated
with the loop. The change in flux could be effected by (i) a time varying magnetic field or by (ii) motion of the
conductor in a magnetic field or (iii) by a combined action of both of these. The discovery is a spectacular
milestone in the sense that it led to important developments in Electrical engineering like invention of
transformer, alternator and generator.
Shortly after Faraday's discovery, Heinrich Lenz found that the direction of the induced current is such that it
opposes the very cause that produced the induced current (i.e. the magnetic field associated with the induced
current opposes the change in the magnetic flux which caused the induced current in the first place). Lenz's
law is illustrated in the following.

In the figures the loops are perpendicular to the plane of the page. The direction of induced current is as seen
towards the loop from the right. Note that the magnetic field set up by the induced current tends to increase
the flux in the case where the magnet is moving away from the loop and tends to decrease it in the case
where it is moving towards the loop,
Mathematically, Faraday's law is stated thus : the electromotive force is proportional to the rate of change of
magnetic flux. In SI units, the constant of proportionality is unity.

where

is the flux associated with the circuit and the minus sign is a reminder of the direction of the current

as given by Lenz's law. If the loop contains

turns, the equation becomes

Though the flux is a scalar, one can fix its sign by considering the sign of the area vector which is fixed by the
usual right hand rule. The dot product of

and

then has a sign.

Recap
In this lecture you have learnt the following
Vector potential for a magnetic field was defined.

Vector potential can be deteremined up to an additive term which is gradient of a scalar function.

Equivalent vector potentials which give the same magnetic field are connected by a gauge transformation.

Vector potentials were calculated in a few simple cases.

Module 3 : MAGNETIC FIELD


Lecture 18 : Electromagnetic Induction
Objectives
In this lecture you will learn the following
Study in detail the principle of electromagnetic induction.

Understand the significance of Lenz's law.

Solve problems involving motional emf.


Motional Emf :
Consider a straight conductor AB moving along the positive x-direction with a uniform speed
uniform magnetic field pointing into the plane of the page, i.e. in

. The region is in a

direction.

The fixed positive ions in the conductor are immobile. However, the negatively charged electrons experience a Lorentz
force

, i.e. a force along the

direction. This pushes the electrons from the end

A to the end B, making the former positive with respect to the latter. Thus an induced electric field is established in the

conductor along the positive

direction. The acceleration of electrons would stop when the electric field is built to a

strength which is strong enough to annul the magnetic force. This electric field

is the origin of what is

known as motional emf . The motion of charges finally stops due to the resistance of the conductor
If the conductor slides along a stationary U- shaped conductor, the electrons find a path and a current is established in
the circuit. The moving conductor thereby becomes a seat of the motional emf. We may calculate the emf either by
considering the work that an external agency has to do to keep the sliding conductor move with a uniform velocity or
by direct application of Faraday's law.
, a force
acts on the wire in the negative x direction. In order to maintain the uniform
If the induced current is
velocity, an external agent has to exert an equal and opposite force on the sliding conductor. Since the distance moved
in time

is

where

, the work done by the external agency is

is the amount of charge moved by the seat of emf along the direction of the current. The emf is an

electric potential difference.


Thus, the emf is equivalently the work done in moving a unit charge. Thus,

This emf corresponds to the potential difference beteen the ends A and B.
An alternate derivation of the above is to consier the flux linked with closed circuit. Taking the origin at the extreme
left end of the circuit, the area of the circuit in the magnetic field is
the fixed end. The flux linked with the circuit is, therefore,
given by

where

is the distance of the sliding rod from

. The rate at which flux changes is therefore

If
is not perpendicular to the plane of the circuit, we will need to take the perpendicular component of
in the
above formula.
Note that, in the illustration above, the magnetic flux linked with the circuit is increasing with time in the negative z
direction. The direction of the induced current is, therefore, such that the magnetic field due to the current is along the
positve z-direction, which will oppose increase of flux. Hence the current, as seen from above, is in the anticlockwise
direction.
Example 18
In the above circuit if the part of the fixed rails parallel to the sliding conductor has a resistance

and the rest of the

circuit may be considered resistanceless, obtain an expression for the velocity of the conductor at time
that it starts with an initial velocity

at

. Explain the change in the kinetic energy of the sliding rod.

Solution :
Since the emf is

, assuming

, the current flowing through the sliding conductor is

The force acting on the sliding conductor is

The force is a retarding one, slowing down the conductor in accordance with Lenz's law.
(This is the amount of force an external agency must apply on the sliding conductor in the positive x-direction to keep
the rod moving with a constant speed.) Thus the equation of motion of the conductor is

The equation may be solved by separating the variables and integrating from time

to

. We get

which gives

i.e.

We can calculate the power dissipated in the circuit in this time by using

which is precisely the change in the kinetic energy of the conductor

, so that

Exercise 1
A pair of parallel conducting rails are inclined at an angle

to the horizontal. The rails are connected to each other at

the ground by a conducting strip. A conductor of resistance


, oriented parallel to the strip can slide down the incline
along the rails. The resistances of the rails and the strip are negligible.
A uniform magnetic field
exists in the vertical direction. The slider is released at some height. Show that the slider
attains a terminal speed given by

where

is the distance between the rails.

Exercise 2
In the above problem, show that after attaining the terminal velocity, the change in the potential energy of the slider is
equal to the Joule heat produced in the slider.
Hint for Solution :
In the preceding exercise, show that the current is given by
amount of heat produced in time

is

. In time

. The rate of Joule heat is

, the slider moves through a distance

. The

. The change in the

potential energy is
Example 19

A conductor AB is moving with a speed

parallel to a long straight wire carrying a current

. What

is the potential difference between the ends A and B ?


Solution :
From the discussion above, the electric field acting at any point on the moving conductor is Lorentz force per unit
charge, i.e.,

. Since the field is not constant along AB, the emf is obtained by integrating the electric field

along AB.
The field at an element of width

at a distance

from the wire is

. Since

and

are perpendicular,

One can obtain the same expression by a direct application of Faraday's law. It is not necessary to have a physical
circuit to calculate the potential difference. One can imagine the conductor AB to be a rod sliding along a rail. As the
rod moves, the area of the closed region ACDB increases. We cansider an area element of width
from the wire. Since

and the area vector are parallel,

Integrating,

Thus

since the rate of increase of the length of the rectangle is

at a distance

Example 20
A rail gun is essentially a linear electromagnetic accelerator which uses Lorentz force to propel an armature (connected
to a projectile) between two parallel rails carrying high current. The current enters through one of the rails and returns
through the other rail. The cuurent in the rails produce magnetic field in the region between the rails. The armature is
subject to a force parallel to the rails.
The separation between the rails is small compared to the length of the rails so that the magnetic field due to the rails
may be approximated as due to infinitely long wires carrying current. However, because the armature provides a path
for the current, there is no current in the rails beyond the armature. The magnetic field is, therefore, equal to that due
to semi-infinite wire (i.e. reduced by a factor of 2 from that of an infinite wire).

In the figure shown, the radii of cross section of the rails are
fields due to both the wires are in the
to the rails. The field at a distance

each and the separation between them is

direction, so that the force

acts along the

. The

direction, parallel

from the origin shown is

The force on the armature is obtained by integrating the force on the element

of the armature

The force on the armature is

A typical rail gun design to achieve high acceleration of a projectile, a pulse current in excess of 100 kAmp is used.
With a rail separation of

cm and

the acceleration produced is in excess of

cm, we have

m/s

. For a projectile mass of 20 gms.

Example 21
and width , having a resistance
falls under gravity with its smaller side
A rectangular conducting loop of length
remaining vertical during the fall. A non-uniform magnetic field exists, directed horizontally and perpendicular to the

plane of the loop. The magnitude of the magnetic field increases linearly with distance

, where

is the distance of the top of the loop from the position from which the loop falls. Obtain an expression for the velocity
of the loop with distance of fall and find the terminal velocity.
Solution :
Let the axes be as shown, with the y-axis out of the plane of the paper. The induced current in the loop is clockwise so
as to oppose the flux increase as the loop falls. The velocity of the loop is
The flux through a strip of width

at position

is

.
. The net flux through the loop is

The induced emf is

The induced current is

. When the loop falls, the sides of the loop are subject to magnetic forces

The forces on the sides QR and SP cancel. The magnetic force on the sides PQ and RS are

The equation of the motion of the loop having a mass

which has a solution

is

The terminal velocity is

, which can also be directly obtained by equating

with

Exercise 3
A rectangular loop of width

and height

falls under gravity into a region of constant magnetic field

. The loop

has a mass
and resistance
. The magnetic field, which remains perpendicular to the plane of the loop, is
constant within the pole pieces and is zero outside.

(a) Find the current in the loop when the speed of the loop is
region (ii) it is wholely inside the filed.

and (i) it is partly inside and partly outside the field

(b) Find the force acting on the loop in both the cases above.
(c) Determine the terminal velocity of the loop.
Assume that the pole pieces are much deeper than the height
(Answer : (a) (i)

counter-clockwise (ii) zero. (b)

speed in case (i) is

acts only on the lower edge. (c) terminal

. Once wholely inside, the speed increases with time as

.)

Exercise 4
A triangular current loop in the shape of an right isoceles triangle of base
magnetic field with a uniform speed
of the figure). Find the emf at time (i)

m/s. At

m enters a region of constant

a corner of the base is at the edge of the field region (top

s and (ii)

s.

(Ans. (i) 0.6 volts (ii) zero.)


Exercise 5
Repeat the above exercise for an equilateral triangle with side

m.

(Answer : 0.7 volts)


Exercise 6
(Mathematically difficult problem) : A circular loop of radius
carrying a current

perpendicular to the wire with a uniform speed

(Answer

the

flux

by

integrating

from the wire (

). The loop moves

. Calculate the motional emf developed.

(Hint : Consider the field at an element of area


Calculate

lies in the same plane as a long straight conductor

. The centre of the loop is at a distance

from

. The distance of the element from the wire is


to

and

from

to

.
.

Use

Recap
In this lecture you have learnt the following
Changing magnetic flux gives rise to an emf. Faraday's law relates the emf to the rate of change of flux.

Flux change may arise due to time dependent magnetic field or due to a change in the area through which the flux

is changing.
The direction of emf is such that it opposes the change in magnetic flux. The direction of the induced current is
given by Lenz's law.
Motional emf is the emf induced in a conductor which is moving in a magnetic field. The source of motional emf is
the Lorentz force acting on the charge carriers.

Module 3 : MAGNETIC FIELD


Lecture 19 : Time Varying Field
Objectives
In this lecture you will learn the following
Relate time varying magnetic field with emf generated.

Define mutual inductance and calculate it in simple cases.

Define self inductance.

Calculate energy stored in a magnetic field.

Time Varying Field


Even where there is no relative motion between an observer and a conductor, an emf (and consequently an
induced current for a closed conducting loop) may be induced if the magnetic field itself is varying with time as
flux change may be effected by change in magnetic field with time. In effect it implies that a changing
magnetic field is equivalent to an electric field in which an electric charge at rest experiences a force.
Consider, for example, a magnetic field

whose direction is out of the page but whose magnitude varies

with time. The magnetic field fills a cylindrical region of space of radius
varying and be given by

Since

does not depend on the axial coordinate

. Let the magnetic field be time

as well as the azimuthal angle

independent of these quantities. Consider a coaxial circular path of radius

, the electric field is also


which encloses a time

varying flux. By symmetry of the problem, the electric field at every point of the cicular path must have the
same magnitude

and must be tangential to the circle.

Thus the emf is given by

By Faraday's law

Equating these, we get for

For

, the flux is

, so that

and the electric field foir

is

Exercise 1

A conducting circle having a radius

at time

is in a constant magnetic field

plane. The circle expands with time with its radius becoming

perpendicular to its

at time

. Calculate the

emf developed in the circle.


(Ans.

Mutual Inductance
According to Faraday's law, a changing magnetic flux in a loop causes an emf to be generated in that loop.
Consider two stationary coils carrying current. The first coil has
second coil contains

turns. The current in the first coil is the source of a magnetic field

around the coil. The second loop encloses a flux

turn of the loop. If the current

The variation of
is

turn and carries a current

in the first coil is varied,

causes an emf

, where

,and consequently

to be developed in the second coil. Since

. The

in the region

is the surface of one

will vary with time.

is proportional to

. The emf, which is the rate of change of flux is, therefore, proportional to

, so
,

where

is a constant, called the mutual inductance of the two coils, which depends on geometrical factors

of the two loops, their relative orientation and the number of turns in each coil.

Analogously, we can argue that if the second loop carries a current

which is varied with time, it generates

an induced emf in the first coil given by

For instance, consider two concentric solenoids, the outer one having
with

turns per unit length and inner one

turns per unit length. The solenoids are wound over coaxial cylinders of length

in the outer solenoid is

, the field due to it is

, which is confined within the solenoid. The

flux enclosed by the inner cylinder is

If the current in the outer solenoid varies with time, the emf in the inner solenoid is

so that

each. If the current

If, on the other hand, the current

in the inner solenoid is varied, the field due to it

which

is non-zero only within the inner solenoid. The flux enclosed by the outer solenoid is, therefore,

If

is

varied,

the

One can see that

emf

in

the

outer

solenoid

is

giving

This equality can be proved quite generally from Biot-Savart's law. Consider two circuits shown in the figure.

The field at

, due to current in the loop

where

. We have seen that

(called the primary ) is

can be expressed in terms of a vector potential

, by Biot-Savart's law

The flux enclosed by the second loop, (called the secondary ) is

, where

Clearly,

It can be seen that the expression is symmetric between two loops. Hence we would get an identical
expression for

. This expression is, however, of no significant use in obtaining the mutual inductance

because of rather difficult double integral.


Thus a knowledge of mutual inductance enables us to determine, how large should be the change in the
current (or voltage) in a primary circuit to obtain a desired value of current (or voltage) in the secondary
, we represent mutual inductance by the symbol

circuit. Since

, where

secondary circuit is given by


Units of

. The emf

in the

is the variable current in the primary circuit.

is that of Volt-sec/Ampere which is known as Henry (h)

Example 22
Consider two parallel rings
centres. Radius of
in

and

with radii

is much smaller than that of

and

respectively with a separation

, so that the field experienced by

between their
due to a current

may be taken to be uniform over its area. Find the mutual inductance of the rings.

Solution :
The field experienced by the smaller ring may be taken to be given by the expression for the magnetic field of
a ring along its axis. We had earlier shown that at a distance
axis is given by

The flux enclosed by

is

By Faraday's law the emf in

is

from the centre of the ring, the field along the

which gives

The expression above is obviously not symmetrical between the loops. This is because of our assumption of
uniform field over

. The approximation will be legitimate if the dimensions of

is taken to be a dipole,

, so that from

is negligibly small, i.e. if

, the other loop looks like a point. In such a case,

In the next example we assume that the current is changing in the dipolar loop and determine the emf
generated in the larger loop.
Example 23
By considering the current in

to be time varying determine the change in flux of the larger coil and hence

determine the mutual inductance.


Solution :
Since the field over the larger loop cannot be considered uniform, we need to use expressions for the
magnetic field due to a magnetic dipole. The field is conveniently expressed in terms of its radial and
tangential components. For a point dipole, the field components are given by expressions similar to the ones
we derived for electric dipole. By replacing the electric dipole moment
and permittivity factor

by the permeability of vacuum

by the magnetic dipole moment


, we have

In the figure, the plane of the loop is normal to the page and the current direction is anticlockwise as seen
from the right, so that the magnetic moment vector is as shown.

Consider an element of area

of the larger ring at the position

where the angle is measured

with respect to some reference line in the plane of the ring. From the figure it can be seen that

so that,

The angle between

and

is

where we have substituted


is obtaied by integrating over

so that

for the magnetic moment of the current loop. The flux enclosed by
from 0 to

and over

from 0 to

The integration over

gives

. Using the expressions for

and

in terms of the variable

we get, for the flux

The integration can be easily performed by substituting

. After a bit of an algebra one gets

On substituting the limits and using a binomial expansion to retain leading order term when

, we

get

which gives the same expression for mutual inductance as in the previous example.
Exercise 1
The figure shows two coplanar and concentric rings of radii

and

where

. Determine the

mutual inductance of the coils. Solve the problem by considering the current to be changing in either of the
coils.

(Ans.

).

Exercise 2
A toroidal coil of rectangular cross section, with height

has

tightly wound turns. The inner radius of the

torus is

and the outer radius

. A long wire passes along the axis.

The ends of the wire are connected by a semi-circular arc. Find the mutual inductance. Show explicitly that
.
(Hint : When the current flows in the turns of toroid, the field at a distance

from the toroid axis is

. The semi-circular area traps flux only in one rectangular turn of height

and width

.)

Answer : (

Self Inductance :
Even when there is a single circuit carrying a current, the magnetic field of the circuit links with the circuit
itself. If the current happens to be time varying, an emf will be generated in the circuit to oppose the change
in the flux linked with the circuit. The opposing voltage acts like a second voltage source connected to the
circuit. This implies that the primary source in the circuit has to do additional work to overcome this back emf
to establish the same current. The induced current has a direction determined by Lenz's law.
If no ferromagnetic materials are present, the flux is proportional to the current. If the circuit contains
turns, Faraday's law gives

where
is known as Self Inductance of the circuit. By definition,
follows, on integrating,

Since

when

is a positive quantity. From the above it

, the constant is zero and we get

The self inductance can, therefore, interpreted as the amount of flux linked with the circuit for unit current.
The emf is given by

Example-24
Obtain an expression for the self inductance in a toroid of inner radius

, outer radius

and height

Solution :
We have seen that the field inside the toroid at a distance

from the axis of the toroid is given by

.
The flux through one turn of the coil is the integral of this field over the cross section of the coil

The flux threading

turns is

The self inductance is thus given by

Energy Stored in Magnetic Field


Just as capacitor stores electric energy, an inductor can store magnetic energy. To see this consider an L-R
circuit in which a current

is established. If the switch is thrown to the position such that the battery gets

disconnected from the circuit at


emf, the circuit is described by

With the initial condition

, the current in the circuit would decay. As the inductor provides back

, the solution of the above is

As the energy dissipated in the circuit in time


battery is disconnected is

is

, the total energy dissipated from the time the

Thus the energy initially stored must have been


energy

. If an inductor carries a current

, it stores an

. Thus the toroidal inductor discussed earlier stores an energy

when it carries a current

. We eill now show that this is also equal to the volume integral of

Consider the magnetic field in the toroid at a distance


is given by

from the axis. We have seen that the magnetic field

. Thus the value of

Considering the toroid to consist of shells of surface area


. The volume integral of

at this distance is
and thickness

, the volume of the shell is

is therefore,

which is exactly the expression for the stored energy derived earlier.
Recap
In this lecture you have learnt the following
The emf generated in one circuit due to a changing current in a second circuit is due to mutual inductance
between the circuits.
Mutual inductance is symmetric.

In a few simple cases mutual inductance was calculated.


A circuit gives rise to a back emf because of change in the current in the same circuit. A circuit can have a
self inductance.
Magnetic field can store energy and the energy density of the field can be calculated from a knowledge of the
self inductance.

Module 3 : MAGNETIC FIELD


Lecture 20 : Magnetism in Matter
Objectives
In this lecture you will learn the following
Study magnetic properties of matter.

Express Ampere's law in the presence of magnetic matter.

Define magnetization and H-vector.

Understand displacement current.

Assemble all the Maxwell's equations together.

Study properties and propagation of electromagnetic waves in vacuum.

Magnetism in Matter
In our discussion on electrostatics, we have seen that in the presence of an electric field, a dielectric gets
polarized, leading to bound charges. The polarization vector is, in general, in the direction of the applied
electric field.
A similar phenomenon occurs when a material medium is subjected to an external magnetic field. However,
unlike the behaviour of dielectrics in electric field, different types of material behave in different ways when an
external magnetic field is applied.
We have seen that the source of magnetic field is electric current. The circulating electrons in an atom, being
tiny current loops, constitute a magnetic dipole with a magnetic moment whose direction depends on the
direction in which the electron is moving. An atom as a whole, may or may not have a net magnetic moment
depending on the way the moments due to different elecronic orbits add up. (The situation gets further
complicated because of electron spin, which is a purely quantum concept, that provides an intrinsic magnetic
moment to an electron.)
In the absence of a magnetic field, the atomic moments in a material are randomly oriented and consequently
the net magnetic moment of the material is zero. However, in the presence of a magnetic field, the substance
may acquire a net magnetic moment either in the direction of the applied field or in a direction opposite to it.
The former class of material is known as paramagnetic material while the latter is called diamagnetic.
When a paramagenetic material is subjected to a magnetic field, the atomic moments are oriented in the
direction of the magnetic field. However, the current directions in adjacent current loops being in opposite
directions, there is no current within the volume of the material. However, the currents do not cancel at the
surface of the substance on which there is a net current which gives rise to its own magnetic field.

We define magnetization
of a sample as the net magnetic moment per unit volume. The unit of
magnetization is Ampere/meter. From the figure it can be seen that the surface current flows along the side
faces. Denoting the normal to the face by

, the surface current density

(in A/m) is given by

The magnetization current is called bound current because the electron is not free to move through the
material as they would in a conductor, but are attached to a particular atom or molecule.
If, however, the magnetization is not uniform within the sample, the internal currents do not cancel and a
magnetization current exists even in the bulk. It can be shown that the bound current density

is given by

Ampere's Law in Presence of Magnetization


Since magnetization of a material produces bound current, it would modify Ampere's law for magnetic field.
Consider a solenoid wound around a hollow cylinder with
magnetic field in the solenoid is uniform and is given by

turns per unit length carrying a current

. If now, a magnetic material is inserted

in the hollow of the cylinder, the material gets magnetized with a magnetization
density

has a magnitude

. The surface current

and has the dimension of current per unit length.

A unit length of the solenoid has an effective current given by the sum of free current
magnetization current

. The

. Ampere's law would then give the magnetic field in the solenoid as

and the

In analogy with the way we introduced displacement vector


to account for the effect of a dielectric in
electrostatics, we introduce a new field, which we will call H-field

Dimensions of
and splitting

is the same as that of magnetization


, where

, i.e. those of A/m. Using

denotes the free current which arises due to mobile charges and

is the magnetization (bound) current, we get

Note that just as the displacement field in a dielectric is determined by free charges, the H-field is determined
only by free currents.
Define magnetic susceptibility

through the relation

Using this, we get

The permeability of the medium is defined as

Exercise 1

Show that Ampere's law may also be expressed as

Ferromagnets
In certain substances like Fe, Co, Ni etc. magnetization not only depends on the applied magnetic field but it
also depends on the history of the material. In a phenomenon known as hysteresis , a sample of a
ferromagnet may exhibit magnetization even when no magnetic field is present exhibiting a memory effect.
When a sample of such a material which has no initial magnetic moment is subjected to a magnetic field, the
magnetization increases with increasing strength of magnetic field and soon saturates when all the atomic
moments have got aligned in the direction of the magnetic field. If field strength is now decreased gradually,
the magnetization decreases. However, the magnetization curve does not retrace its path. When the field
strength has been reduced to zero, the sample still has some magnetization left. In order to bring the sample
to a state of zero magnetization, a coercive field

must be applied in the reverse direction.

Diamagnetism
Diamagnetism, being a consequence of Lenz's law is present in all substance though its effect may be masked
because of other strong magnetic effects. However, certain substances like Bismuth are strongly diamagnetic.
The effect arises because when an atom is placed in a magnetic field, the flux through the atomic orbit
changes. This results in an induced current being generated which opposes the changing flux. The effect is
equivalent to the atom developing an induced magnetic moment opposite to the direction of the applied field.
Displacement Current
We have seen that the magnetic field due to a current is given by Ampere's law

where the current passes through the surface of the boundary over which the above integral is taken. Maxwell
pointed out that the equation is logically inconsistent.
Consider a parallel plate capacitor being charged by a battery. During the process of charging a current passes
through the terminals of the battery. The current produces a magnetic field around it. Consider an Amperian
loop

located just outside one of the plates. Let the plane of the loop

be parallel to the plates. On

applying Ampere's law to such a loop, we get the value of the above integral to be non-zero as there is a
current passing through the wires. However, consider a second surface

which is located just inside the

plates of the capacitor and does not intersect the wire at all. The flux of the current (the the surface integral
of the current density) through this surface is zero. However, as both
loop

and

are bounded by the same

, the flux through both must be the same. This is clearly inconsistent.

To remove the apparent inconsistency, we need to revise our notion of what constitutes a current. We
generally believe that current being a flow of charge must be the same at all cross sections of a series circuit.
This is obviously not true for circuits containing capacitors. Consider an RC circuit. When the switch S is
closed, the capacitor charges and the current is given by

This does not hold inside the capacitor as the dielectric between the plates is a dielectric (could be air) and
does not conduct electricity. In order to preserve the continuity of current inside the dielectric, we need to
reconcile the apparent inconsistency. Since the current through the wire

is the rate of flow of

charge through the wires, we may express it in terms of the electric field inside the capacitor plates. Since the
electric field inside the capacitor is given by

where

, an equivalent expression for current is

is the electric flx through the capacitor.

Maxwell suggested that a term

should be considered as current through the dielectric. He termed

this current as displacement current. The Ampere's law is modified to read

Equation above is now logically consistent because inside the dielectric


and the value of the integral is
given by the second term. The current exists whenever the electric field, and hence the electric flux, is
changing with time. Using Stoke's theorem, we may convert the above to a differential form

Exercise-2
Show the above relationship.
Example-25
A parallel plate capacitor with circular plates of radii 10 cm separated by 5 mm is being charged by an
external source. The charging current is 0.2A. Find (a) the rate of change of potential difference between the
plates and (b) the displacement current.

Solution :
The

capacitance

is

F.

Given

V/s. The displacement current is

Thus the displacement current is 0.2 A.


Exercise-3
A 50 pF parallel plate capacitor is being charged so that the voltage is increasing at a rate 300 V/s. The
capacitor plates are circular with radii 10 cm each. Calculate (i) the displacement current density and (ii) the
magnetic field strength at a distance of 5 cm from the axis of the capacitor.
(Ans. (i)

Maxwell's Equations

A/m

(ii)

T.)

We are now in a position to collect together all the laws governing the dynamic of electromagnetic field. These
equations are collectively known as Maxwell's equations.
Gauss's Law of Electrostatics :

Gauss's Law of Magnetism :

Faraday's Law

Ampere-Maxwell Law

These equations are to be supplemented by Lorentz force equation.


In terms of free charges and currents, the first and the fourth of Maxwell's equations are generally expressed
in terms of the vectors

and

with the displacement vector

and the H-vector defined as

The polarization vector


and the magnetization vector
respectively by constitutive relations

are related to the vectors

Electromagnetic Waves
In the absence of any source of charge or current, Maxwell's equations in free space are as follows :

and

The last two equations couple the electric and the magnetic fields. If
zero. This implies that

is a function of position. Further, if

. In such a case

also varies with time since the

is time dependent,

is non-

itself changes with time, so does


operator cannot cause time variation. Thus,

in general, a time varying magnetic field gives rise to an electric field which varies both in space and time. It
will be seen that these coupled fields propagate in space.
We will first examine whether the equations lead to transverse waves. For simplicity, assume that the electric
field has only x-component and the magnetic field only y-component. Note that we are only making an
assumption regarding their directions - the fields could still depend on all the space coordinates
addition to time

, in

Gauss's law gives

Since only

Thus

, this implies

is independent of
is independent of

coordinate and can be written as

. A similar analysis shows that

coordinate and can be written explicitly as

Consider now the time dependent equations eqns. (3) and (4). The curl equation for
component

Since

showing that

show that

gives, taking z-

, this gives

is independent of

also depends only on

and hence depends only on

and

. Thus the fields

and

and

. In a similar manner we can

do not vary in the plane containing

them. Their only variation takes place along the z-axis which is perpendicular to both
direction of propagation is thus

direction.

and

. The

To see that propagation is really a wave disturbance, take y-component of Eqn. (3) and x-component of Eqn.
(4)

To get the wave equation for

, take the derivative of eqn. (5) with respect to

and substitute in eqn. (6)

and interchange the space and time derivatives,

Similarly, we can show, We get

Each of the above equations represents a wave disturbance propagating in the z-direction with a speed

On substituting numerical values, the speed of electromagnetic waves in vacuum is


Consider plane harmonic waves of angular frequency

and wavlength

m/sec.
. We can express the

waves as

The amplitudes

an

are not independent as they must satisfy eqns. (5) and (6) :

Using Eqn. (5) we get

The ratio of the electric field amplitude to the magnetic field amplitude is given by

Fields
and
are in phase, reaching their maximum and minimum values at the same time. The electric
field oscillates in the x-z plane and the magnetic field oscillates in the y-z plane. This corresponds to a
polarized wave . Conventionally, the plane in which the electric field oscillates is defined as the plane of
polarization. In this case it is x-z plane. The figure shows a harmonic plane wave propagating in the zdirection. Note that

and the direction of propagation

form a right handed triad.

Example-26
The electric field of a plane electromagnetic wave in vacuum is
V/m,

. Determine the state of polarization and the direction of propagation of the wave.

Determine the magnetic field.


Solution :
Comparing with the standard form for a harmonic wave

so that

m. the direction of propagation is x-direction. Since the electric field oscillates in

x-y plane, this is the plane of polarization. Since

must be perpendicular to both the electric field direction

and

has

the

direction

of

propagation,
T. Thus

only

z-component

with

an

amplitude

Exercise-4
The magnetic field of a plane electromagnetic wave is given by

Determine the electric field and the plane of polarization.


(Ans. Strength of electric field is

V/m)

Wave Equation in Three Dimensions


We can obtain the wave equation in three dimensions by using eqns. (1) to (4). On taking the curl of both
sides of eqn. (3), we get

Using the operator identity

wherein we have used

, and substituting eqn. (4) we get

A three dimensional harmonic wave has the form

or

Instead of using

the trigonometric form, it is convenient to use the complex exponential form

and later take the real or imaginary part of the function as the case may be. The derivative of
given as follows :

Since

we have,

In a similar way,

Thus for our purpose, the differential operators

and

may be equivalently replaced by

is

Using these, the Maxwell's equations in free space become

We can see that

and

form a mutually orthogonal triad. The electric field and the magnetic field are

perpendicular to each other and they are both perpendicular to the direction of propagation.
Generation of Electromagnetic Waves
We have looked for solutions to Maxwell's equations in free space which does not have any charge or current
source. In the presence of sources, the solutions become complicated. If
only have a steady electric field. If

= constant, i.e. if

, we

varies uniformly with time, we have steady currents which gives us both

a steady electric field as well as a magnetic field. Clearly, time varying electric and magnetic fields may be
generated if the current varies with time, i.e., if the charges accelerate. Hertz confirmed the existence of
electromagnetic waves in 1888 using these principles. A schematic diagram of Hertz's set up is shown in the
figure.

The radiation will be appreciable only if the amplitude of oscillation of charge is comparable to the wavelength
of radiation that it emits. This rules out mechanical vibration, for assuming a vibrational frequency of 1000
cycles per second, the wavelength work out to be 300 km. Hertz, therefore, made the oscillating charges
vibrate with a very high frequency. The apparatus consists of two brass plates connected to the terminals of a
secondary of a transformer. The primary consists of an LC oscillator circuit, which establishes charge
oscillations at a frequency of

. As the primary circuit oscillates, oscillations are set up in the

secondary circuit. As a result, rapidly varying alternating potential difference is developed across the gap and
electromagnetic waves are generated. Hertz was able to produce waves having wavelength of 6m. It was soon
realized that irrespective of their wavelength, all electromagnetic waves travel through empty space with the
same speed, viz., the speed of light.

Depending on their wavelength range, electromagnetic waves are given different names. The figure shows the
electromagnetic spectrum. What is known as visible light is the narrow band of wavelength from 400 nm
(blue) to 700 nm (red). To its either side are the infrared from 700 nm to 0.3 mm and the ultraviolet from 30
nm to 400 nm. Microwaves have longer wavelength than the infrared (0.3 mm to 300 mm) and the radio
waves have wavelengths longer than 300 mm. The television broadcast takes place in a small range at the
end of the microwave spectrum. Those with wavelengths shorter than ultraviolet are generally called rays .
Prominent among them are x-rays with wavelengths 0.03 nm t0 30 nm and

-rays with wavelengths shorter

than 0.03 nm.


Poynting Vector
Electromagnetic waves, like any other wave, can transport energy. The power through a unit area in a
direction normal to the area is given by Poynting vector , given by

As
units

and

form a right handed triad, the direction of

ismeasured in watt/m

The magnitude of

is along the direction of propagation. In SI

for the electromagnetic wave travelling in vacuum is given by

where we have used the relationship between

and

in free space. For harmonic waves, we have

The average power transmitted per unit area, defined as the intensity is given by substituting the value 1/2
for the average of the square of sine or cosine function

Example-27
Earth receives 1300 watts per squar meter of solar energy. assuming the energy to be in the form of plane
electromagnetic waves, compute the magnitude of the electric and magnetic vectors in the sunlight.
Solution :
From the expression for the average Poynting vector

which gives

V/m. The corresponding rms value is obtained by dividing by

V/m. The magnetic field strength is

T.

Exercise-5
A 40 watt lamp radiates all its energy isotropically. Compute the electric field at a distance of 2m from the
lamp.
( Ans. 30 V peak)
Radiation Pressure
We have seen that electric field, as well as magnetic field, store energy. The energy density for the electric
field was seen to be

and that for the magnetic field was found to be

electromagnetic waves, where

. For the

, the total energy density is

where we have used

In addition to carrying energy, electromagnetic waves carry momentum as well. The relationship between
energy (

) and momentum (

) is given by relatistic relation for a massless photons as

the energy density of the electromagnetic waves is given by

. Since

, the momentum density, i.e. momentum

per unit volume is

Since the direction of momentum must be along the direction of propagation of the wave, the above can be
converted to a vector equation

If an electromagnetic wave strikes a surface, it will thus exert a pressure. Consider the case of a beam falling
normally on a surface of area
which absorbs the wave. The force exerted on the surface is equal to the
rate of change of momentum of the wave. The momentum change per unit time is given by the momentum
contained within a volume

. The pressure, obtained by dividing the force by A is thus given by

which is exactly equal to the energy density

If on the other hand, the surface reflects the wave, the pressure would be twice the above value.
The above is true for waves at normal incidence. If the radiation is diffuse, i.e., if it strikes the wall from all
directions, it essentially consists of plane waves travelling in all directions. If the radiation is isotropic, the
intensity of the wave is the same in all directions. The contribution to the pressure comes from those waves
which are travelling in a direction which has a component along the normal to the surface. Thus on an
average a third of the radiation is responsible for pressure. The pressure for an absorbing surface is
while that for a reflecting surface is

The existence of radiaton pressure can be verified experimentally. The curvature of a comet's tail is attributed
to the radiation pressure exerted on the comet by solar radiation.
Exercise-6
Assuming that the earth absorbs all the radiation that it receives from the sun, calculate he radiation pressure
exerted on the earth by solar radiation.
(Ans. Assuming diffuse radiation

N/m

Recap
In this lecture you have learnt the following
The response of different substances to magnetic field are different. In paramagnetic material the
magnetization in parallel to the direction of the external field. Diamagnetism aroses due to Lenz's law.
A ferromagnetic material shows memory effect, i.e., hysteresis.
Ampere's law is modified in the presence of a magnetic material. The ampere's law for the H-field is
determined by free currents.
When a current passes through circuit containing a capacitor, the continuity of current is established by
defining a displacement current through the capacitor.
By solving the Maxwell's equations in free space, propagating electromagnetic waves was studied.
Electromagnetic waves can carry both energy and momentum.

Module 4 : THERMOELECTRICITY
Lecture 21 : Seebeck Effect
Objectives
In this lecture you will learn the following
Seebeck effect and thermo-emf.

Thermoelectric series of metals which can be used to form thermocouples.

Peltier effect, i.e. absorption or evolution of heat at junctions of dissimilar metals if a current exists in
the circuit.

Thomoson effect of heat exchange in a circuit of a single metal in which a temperature gradient exists.
Seebeck Effect :
In 1821 Thomas Seebeck, a German physicist discovered that when two dissimilar metal ( Seebeck used
copper and bismuth) wires are joined at two ends to form a loop, a voltage is developed in the circuit if
the two junctions are kept at different temperatures. The pair of metals forming the circuit is called a
thermocouple . The effect is due to conversion of thermal energy to electrical energy.
The existence of current in the closed circuit may be confirmed by

the deflection of a magnetic needle caused by the magnetic field of the current

Joule heating produced in the wires

closing the circuit with a capacitor to accumulate measurable charge

placing a sensitive ammeter or a galvanometer in the circuit

measuring the amount of chemiucal deposit at the electrodes of an electrochemical cell.

Click here for animations


Thermoelectric Power

Click here for animations


The open circuit potential difference in the circuit whose junctions are maintained at temperatures
and

is given by

where the coefficient of proportionality is known as the thermoelectric power or the Seebeck
coefficient. The term thermoelectric power is a misnomer because it does not measure any power and
is measured in volt/ K. By convention, Seebeck coefficient's sign is the sign of the potential of
is positive, convertional current flows from A
the cold end with respect to the hot end. Thus if
to B at the hot junction. Seeback coefficient is not a constant but is dependant on temperature. The
temperature dependence of a commercial thermocouple is usually expressed as a polynomial expansion
in powers of temperature
. For instance, for a thermocouple with Platinum as one of the metals and
an alloy of Pt-Rh (90:10) the open circuit voltage is given approximately by a quadratic

so that the thermoelectric power is given by

The relationship between

and

is a parabola. The temperature

at which the

thermoelectric power is maximum is called the neutral temperature . The temperatures


and

at which a small change in the difference of the junction temperatures leads to a

change in the sign of emf is called the inversion temperature.

A complete understanding of Seebeck effect requires knowledge of behaviour of electron in a metal


which is rather complicated. The Seebeck coefficient depends on factors like work functions of the two
metals, electron densities of the two components, scattering mechanism within each solid etc. However,
a simple minded picture is as follows.
Seebeck effect is a manifestation of the fact that if two points in a conductor (or a semiconductor) are
maintained at different temperatures, the charged carriers (electorns or holes) in the hotter region,
being more energetic (and, therefore, having higher velocities) will diffuse towards region of lower
temperature. The diffusion stops when the electric field generated because of movement of charges has
established a strong enough field to stop further movement of charges. For a metal, carriers being
negatively charged electrons, the colder end would become negative so that Seebeck cofficient is
negative. For a p-type semiconductor on the other hand, holes diffuse towards the lower temperature
resulting in a positive Seebeck coefficient.
Performance of a thermocouple is determined by the Seebeck coefficient of the pair of metals forming
the thermocouple. As it is impracticable to list the coefficient of all possible pairs, the Seebeck
coefficients of metals are usually given with respect to Platinum as standard whose Seebeck coefficient
is taken as zero. The following table gives the Seebeck coefficient (in

) of some standard

thermocouple material
at 0

C.

Material

Seebeck Material
coefficient

Seeback Material
coefficient
4 Iron

Seebeck
coefficient

Bismuth

Lead

19

Constantan

Tantalum

4.5 Nichrome

25

Nickel

Rhodium

6 Antimony

47

Potassium

Gold

6.5 Germanium

300

Sodium

Silver

6.5 Silicon

440

6.5 Tellurium

500

7.5 Selenium

900

Mercury

0.6 Copper

Carbon

3 Cadmium

Aluminium

3.5 Tungsten

7.5

An alloy of copper (60%) and nickel (40%)


An alloy of Ni-Cr-Fe-Si
Example -1
Seebeck

voltage

for

Copper-

, where

Constantan

(Cn)

thermocouple

is

given

is the absolute temperature of the hot junction and

by

the
and

linear relation
are constants

given by
Cu :
Cn :

mV

mV/K

mV

mV/K

Calculate the thermoelectric power when the hot junction is at 100

C.

Solution

The thermoelectric power

Thermoelectric Series
A thermoelectric series is an ordering of thermocouple elements in such a way that when any pair of
materials in the series is used to form a thermocouple, Seebeck current at the cold junction flows from
the member occuring earlier in the series to that occuring later. One should be careful in using the series
as the opposite convention is sometimes used. As one uses pairs of members which are closer in the
series, the thermoelectric yield goes down when one reduces the temperature difference. The following
table gives the thermal emf (in
the hot junction is at

) with respect to Platinum when the cold junction is at

C.

Material

thermo-emf Material

thermo-emf

C while

Chromel
Fe
W
Cd

2810 Rh

700

1980 Pb
1120 Al
900 Ta

440
420
330

Au

780 Alumel

1290

Cu

760 Ni

1480

Zn

760 Constantan

3510

Ag

740 Bi

7340

An alloy of Ni-Cr-Mn
An alloy of Ni-Al-Mn
If the temperatures of the two junctions are kept fixed at

and

, then the emf of a

circuit consisting of a pair of metals A and B is equal to the difference between the emfs of two circuits,
one consisting of the pair A-C and the other of the pair B-C, i.e.

In view of this, it is convenient to measure the thermo-emf with respect to a reference metal C.
Platinum is taken to be such a reference.
Example-2
Compute the thermo-emf of a copper-constantan thermocouple with its junctions at

C and

C.

Solution
From the table above we have

and

. Thus

Thus the emf yield of a copper-constantan thermocouple is 4.27 mV.


Exercise
The thermoelectric power of a thermocouple whose cold junction is maintained at
junction at

(1)
(2)

C and the hot

C is given by the following graph.

Sketch the thermo-emf with temperature.


Determine the neutral temperature and the temperature of inversion. How will these two temperatures
change if the cold junction were maintained at

C?

(Ans. (ii)

C,

C; No change,

C)

Exercise
A chromel-constantan thermocouple with the cold junction at
circuit voltage for temperatures of the hot junction between
emf when the hot junction is maintained at
the same temperature] (Ans. 9.5 mV)

C has a linear variation of the open


C and

C. Calculate the thermo-

C. [ Hint : Emf is zero when both the junctions are at

Peltier Effect
In 1834 Jean Peltier, a french watch maker, discovered a second thermoelectric effect. If a current flows
through a circuit containing junction of two dis-similar metals, it leads to an absorption or liberation of
heat at the junctions. Heat is given out or absorbed depending on the pairs of metals and the direction
of the current. The phenomenon of heat evolution is different from the Joule heat as Peltier effect is a
reversible process while Joule loss is irreversible.
If the direction of the current at the junction is same as the direction of the Seebeck current, heat is
liberated if the Seebeck junction is a hot junction or is absorbed if the junction is cold. Thus for a copper
- constantan thermocouple, if the current flow at the junction is from copper (+) to constantan (-), heat
is absorbed. On changing the direction of the current, heat will be liberated at the same junction,
showing that the phenomenon is reversible.

The amount of heat

liberated to (or absorbed from) the surroundings in order that the junction may

be kept at the same temperature is proportional to the current

where the constant

passing through the junction

is called the Peltier coefficient . The Peltier coefficient depends on the pair

of materials A and B of the junction and also on the junction temperature.


Thomson Effect
William Thomson (later well known as Lord Kelvin) discovered a third thermoelectric effect which
provides a link between Seebeck effect and Peltier effect.
Thomson found that when a current is passed through an wire of single homogeneous material along
which a temperature gradient exists, heat must be exchanged with the surrounding in order that the
original temperature gradient may be maintained along the wire. (The exchange of heat is required at all
places of the circuit where a temperature gradient exists.)
Thomson effect may be understood by a simple picture. A conductor has free charge carriers, which are,
electrons in metals, electrons and holes in semiconductors and ions in case of ionic conductors. Consider
a section of such a conductor whose one end is hotter than the other end. Charge carriers at the hot
end, being more energetic, will diffuse towards the colder end. The charge separation sets up an electric
. Diffusion of carriers would stop when the attractive force on the carriers due to this field
field
strong enough to retard the motion of the carriers due to thermal effect.

is

We can represent the effect of the thermal gradient responsible for the diffusive motion of the carriers
by an effective field
as

where

is known as the Thomson coefficient for the material of the conductor. The Thomson

electromotive force

where

. This effective field is proportional to the thermal gradient and can be written

and

is given by

are the temperatures at the two ends of the rod.

Thomson effect is a manifestation of the Thomson emf described above. Clearly, one cannot
demonstrate the existence of the emf by using it to drive a current in a close circuit. This is because if
one uses a single metal with a temperature gradient, the integral
around a close loop is zero. For
dis-similar metals, Peltier effect dominates over Thomson effect.
is passed through a homogeneous conductor with a temperature gradient, the rate of
When a current
heat production per unit volume is given by

where

is the resistivity of the sample. The first term is the irreversible Joule heat. The second term is

due to Thomson emf.


In metals such as copper and zinc, the hotter end is at a higher potential (as shown in the figure above).
In such a situation if the current due to an external supply is in the same direction as the direction of
decreasing potential, there is additional evolution of heat due to Thomson effect and the net heat
produced is more than the Joule heat. If the direction of the current is reversed, heat energy is
converted to electrical energy due to Thomson effect and the rate production of heat is reduced. This is
known as positive Thomson effect .
An anomalous situation occurs in metals such as cobalt and iron. In these metals the hotter end is at a
lower potential so that charge carriers move against the thermal gradient. The effect is opposite of what
happens in case of positive Thomson effect. Such anomalous effect is known as negative thomson
effect . Lead shows zero Thomson effect. The simple physical picture given above cannot explain the
strange behaviour.
Recap
In this course you have learnt the following
A current flows in a circuit of two dissimilar metals, when the two junctions are kept at different
temperatures. The junction circuit is called thermocouple.
The magnitude and sign of the thermo-emf developed depends on the material of the two metals and
thetemperatures of the two junctions. The metals can be arranged in an order called the thermoelectric
series such that in a thermocouple made with two metals in the series, the current in the hot junction is
from the metal preceding in the series to the one occuring later.
The inverse effect in which heat is evolved or absorbed at the junctions of two dissimilar conductors if a
current is flowing in the circuit is known as Peltier effect.
Pelter effect and Seebeck effect are linked by Thompson effect according to which when a current is
passed through a single wire in which temperature gradient exists, heat exchange with the surround
must take plac if the temperature gradient is to be maintained.

Module 4 : THERMOELECTRICITY
Lecture 22 : Temperature Measurement Using Thermocouple
Objectives
In this lecture you will learn the following
Use of thermocouples for measurement of temperatures particularly in higher range of temperatures.
Empirical laws of thermoelectricity, viz., law of homogeneous circuits, law of successive temperatures
and aw of intermediate metals.
Application of laws of thermodynamics to thermoelectric phenomena.

Temperature Measurement Using Thermocouple


The emf of thermocouple is rather too small for it to be used as a device for converting heat energy to
electrical energy. However, thermocouples are of great practical use in measurement of temperatures,
particularly in high temperature region, where the tolerable sensitivity is about
measurement is to use the Seebeck equation

C. The principle of

where
is the change in voltage,
is the Seebeck coefficient for the thermocouple pair and
is the change in temperature. (The relationship is not quite linear and one can use a nonlinear
relationship with tabulated values of coefficients.) We can measure the emf of the thermocouple with
one of the junctions at a known temperature (called the reference temperature) and use the above
equation to determine the temperature of the second junction. The system whose temperature is to be
measured can easily be brought into thermal equilibrium with the second junction because of the small
mass of the thermocouple. In scientific work, it is standard to use a thermocouple with Pt as one of the
materials and an alloy of Pt-Rh as the other, to measure temperatures in the range of

C to

C.
Seebeck voltage cannot be directly measured because of the need to connect a voltmeter in the circuit,
whose terminals introduce additional junctions. Consider, for instance an iron-constantan thermocouple
whose open circuit voltage is to be measured using a voltmeter with copper leads.

In doing so we introduce two additional junctions, one of Cu-Fe and the other of Cu-Cn whose effects
must be compensated. An alternative is to introduce a reference junction, as shown in the figure to the
right. With this arrangement both the junctions with the copper leads are with the same material with
the voltages across the pair of junctions are equal and opposite so that their effects cancel. It is common
C so that the reading of the
to keep the compensating junction at a reference temperature of
voltmeter can be directly calibrated to give the temperature of the junction whose temperature is being
measured.
Empirical Laws of Thermoelectricity

Experiments with thermocouples have led to formulation of three empirical laws. They are as follows :
Law of Homogeneous Circuits
It is observed that electric current cannot be sustained in a circuit with a single homogeneous metal by
application of heat alone. A consequence of this law is that given that junctions between two dissimilar
metals are maintained at some temperatures, the thermal emf is not affected by the lead wires even
when a thermal gradient exists along the lead wires.
Law of Successive temperatures
If the thermal emf given by a thermocouple is
and

and is

when the junctions are maintained at temperatures

when the junction temperatures are

emf given by the thermocouple when the junction temperatures are

and
and

, then the
is given by

The conclusion is obvious from Seebeck's expression for emf as

The law is useful in relating the value of emf found using some arbitrary but known reference junction
temperature to that which one would obtain if a standard reference junction temperature (like freezing
point of water) were used.
Law of Intermediate Metals
According to this law, inserting an wire of arbitrary material into a thermocouple circuit has no effect on
the thermal emf of the original circuit, if the additional junctions introduced in the circuit are at the same
temperature.

The law is useful in using thermocouples with elements which are both different from the material of the
lead wires of a voltmeter.
The most convenient way to analyze a thermocouple circuit is to use Kirchhoff's voltage law, which
states that the algebraic sum of voltages around a closed loop is zero.
Thus when the junction of two metals
circuit voltage

so that

and

are maintained at a temperature

is measured under isothermal condition at a temperature

and the open

, we may write

Values of

and

determine the polarity of the voltmeter. For instance, in the Cu-Cn junction

shown,

If
is the temperature of the hot junction,
is negative and therefore, if the circuit were closed, the
current would flow from Cu to Cn at the cold junction.
Example-1
Consider an Iron-Constantan thermocouple. If the lead wires of the voltmeter are made of copper,
suggest a method for measuring the temperature of the junction.
Solution
One of the ways to handle this is to introduce an intermediate metal in such a way that both the
junctions with the copper terminals are with the same material, as shown in the following figure.

Applying Kirchhoff's loop law, we have

giving

Exercise-2
Consider an Iron - copper thermocouple. Suggest a method of measuring the temperature of a junction
using a voltmeter with copper lead wires.
Exercise
For the circuit shown below, calculate the voltage measured at the copper terminals under isothermal
conditions. Use the table for thermo-emf. (Hint. you do not need thermo emf data at

C.)(4.1 mV)

Application of Thermodynamics to Thermocouple


Consider a junction of two dissimilar metals A and B, with the cold junction at a temperature
hot junction at a temperature

If a small charge

. Let

and the

be the thermo-emf generated in the circuit.

is made to circulate around the loop in time

, it gives rise to a current

. We

consider the system to be adiabatic. In order to conserve energy, the work done on the charge carriers
by

must be equal to the sum of the following :

Heat absorbed and liberated at the junctions due to Peltier effect.

Heat exchange within the individual conductors due to Thomson effect.


We have neglected the Joule heat. Applying Kirchhoff's loop law, we have, for the power delivered,

Here

is the rate at which Peltier heat is absorbed at the hot junction while
is that given out at the cold junction. Likewise, the term with is the Thomson heat

absorbed by the metal B the term is the heat evolved by the metal A.
If the temperature difference

is small, the emf is written and one can approximate the right hand

side of the above by an expansion keeping only the linear term in

so that in the limit of

one has

which shows that Seebeck effect is due to the combined effect of Peltier effect and Thomson effect.
Equation (1) is known as Kelvin's First Relation and is a consequence of the first law of
thermodynamics.
If the Joule heat is ignored, the exchange of heat is reversible. According to the second law of
thermodynamics the change in entropy around the closed loop is zero,

The Peltier heat is evolved or absorbed only at the junctions where as Thomson heat is exchanged
throughout the wire. Thus

In the limit of

we get

Equation (2) is known as Kelvin's Second Relation . Carrying out the differentiation above explicitly,
we get

Using Eqn. (1) in the above, we get a relation between the Thermoelectric Power and the Peltier
coefficient

A second differentiation of Eqn. (3) gives

Since

represents the entropy of the carriers, according to the third law of thermodynamics,

Using this we may integrate Eqn. (2) to get

The integral above should be convergent, which requires

Since
and
are material dependent, it is necessary that not just their difference
must individually vanish in the limit of absolute zero temperature.

, and

Exercise
From the table of Seebeck coefficients, calculate the Peltier coefficient of an Iron-copper thermocouple at
C. (Ans. 3.4 V)
Example
The thermoelectric power of a thermocouple is given by the expression (in

/K)

where
is in K. Determine (i) the temperature at which the Peltier coefficient is maximum and the
value of the maxiumum Peltier coefficient, (ii) the temperature at which the Thomson coefficients of both
the components of the thermocouple are equal.
Solution
Peltier coefficient is given by
The maximum of occurs when

Solving for the quadratic equation, we get the maximum Peltier voltage to occur at
Substituting in the expression for

the value of the maximum Peltier voltage is

K.
volt.

From Eqn. (4) above, the Thomson coefficients of the two components are equal at temperature at which
vanishes, i.e. when
Solving,
Exercise
A chromel-constantan thermocouple with the cold junction at

C has a linear variation of the open

circuit voltage with the temperature of the hot junction. Using the thermo-emf values at
the table, calculate (i) the thermo-emf when the hot junction is maintained at
coefficient at

C from
C, (ii) Peltier

C and (iii) the difference between Thomson emf at this temperature. [Hint : Emf is

zero when the hot junction is at


(Ans. (i) 9.5 mV (ii) 26.7 mV (iii) 0)

C]

Recap
In this course you have learnt the following
The Seebeck equation is useful in using thermocouples for measurement of temperature by maintaining
one of the thermocouple junctions at a known temperature and calibrating the thermocouple voltage by
maintaining the other junction at different temperatures. Usually, thermocouples are used for
measurement of high temperatures.
A set of three empirical laws have been established for the behaviour of thermocouples. Thermo-emf
cannot be sustained in a circuit with a single homogeneos metal by application of heat.
Laws of thermodynamics can be applied to a circuit of dissimilar metals to obtain expressions for
variation of thermo-emf with temperature. The Thompson heat evolved for all material vanish at
absolute zero as a consequence of third law of thermodynamics.

Module 5 : MODERN PHYSICS


Lecture 23 : Particle and Waves
Objectives
In this lecture you will learn the following
Radiation (light) exhibits both wave and particle nature.

Laws governing black body radiation, like Stefan's law and Wien's law.

Inadequacy of wave theory in explaining blackbody radiation spectrum.

Planck's hypothesis on atoms absorbing radiation in quanta of energy.


Particle and Waves
In classical physics have come to regard matter and waves as two disctinct entities. A particle is an idealized
point object which is characterized by
a mass

position (a particle cannot be in more than one position at the same time)

momentum
In practice entities like electrons, protons, atoms, molecules etc. are approximated as particles. In classical
dynamics it is also common to consider macroscopic objects like a billiard ball as a particle. Particle transmit
energy from one point in space to another by collisions with other particles during which transfer of
momentum also takes place.
Wave is an extended disturbance in space which can transmit energy from one point to another without
imparting a net motion to the medium through which it propagates. Examples of waves are mechanical waves
like sound waves, water waves etc. which require a material medium to propagate and electromagnetic waves
(light waves, radio waves, x-rays etc.) which can propagate in space without requiring a medium. A wave is
characterized by
wavelength

frequency

Traditionally, the wave and particle properties have been considered distinct. For instance, the following
phenomena can be only understood in terms of wave properties :
Interference

Diffraction

Polarization

Similarly, the process of collision or the concept of temperature as energy of vibrating molecules are
understood in terms of particle properties. However, some experimental observations made in the late 19th
century and early 20th century seemed to indicate that the strict behavioural pattern stated above is not
always valid.
Particle Nature of Waves
Light was accepted to have wave nature in view of well established experiments on diffraction. However,
Photoelectric Effect could be understood only by assuming that light consisted of streams of particles
possessing energy and momentum. The first phenomenon which was observed to be in disagreement with the
wave nature of light is the black body radiation problem.
Black Body Radiation :
A black body, by definition, is an object which absorbs all radiation that fall on it. Since it does not reflect any
light, it appears black.
In a laboratory, one could approximate a blackbody by a cavity with highly polished walls. If the walls of the
cavity has a small hole, any radiation that enters through the hole gets trapped in the cavity. Stars may also
be approximated as black bodies as any radiation directed at them gets absorbed.

A black body is also a perfect emitter of radiation. It can emit at all wavelengths. However, the radiation from
a black body is observed to obey the following two laws :
Stefan's Law : The intensity of emitted radiation for a given wavelength is proportional to the fourth power
of the temperature of the black body.
Wien's law : For a given temperature, the spectrum of emitted radiation has maximum intensity for a
wavelength

, which is inversely proportional to the temperature of the black body. Thus relatively

colder bodies appear red as their maximum intensity is in the red end of the spectrum while hotter bodies
appear bluish. Because of this, when we heat a metal wire it firs becomes red hot and then as the
temperature increases it become "white hot".
In classical physics, radiation is considered as waves and the calculation of radiant energy emitted by a black
body is carried out in the following steps.

(1)
(2)

We consider the black body to be in the shape of a cubical metal cavity of side
with a small hole in it.
Any radiation which falls on the hole is lost inside the cavity. The radiation which emerges from the hole has
the characteristics of the radiation that is trapped inside the cavity.
The waves inside the cavity form standing wave pattern with nodes at the walls of the cavity since the electric
field must vanish inside a metal.
If we consider standing waves in one dimension, the electric field having nodes at
given by

is shown. The frequency

where

is given by

and

is

is a positive integer. The pattern of the standing wave

Extending to three dimensions, the electric field is given by

where

is a set of positive integers. (If any of these inegers is zero, it gives zero field. Taking

negative values of the integers do not give different fields as it amounts to simply multiplying

by a sign

factor.) Substituting Eqn. (1) in the electromagnetic wave equation

we get

The frequency

is given by

For a given frequency, the equation above represents a sphere of radius

in the three

dimensional space of

and

and each value of

represents a distinct point in this space. Since

can only take integral values, the number of points per unit volume is one. If we treat
continuous variable, the number of modes for frequency less than some given

where

is given by

is the volume of the cavity. In the above, the factor of 1/8 comes because we are restricted to the
can only be positive. The factor of 2 takes into account the fact that there are

positive octant as

two transverse modes. The number of modes in the frequency interval

(3)

as a

As the average energy of a mode is

and

is

, the radiant energy density, which is defined as the average energy

per unit volume is given by


(2)
This is known as Rayleigh - Jeans' Law
Exercise 1
Show that, in terms of the wavelength interval, the Rayleigh Jeans' law can be expressed as

(4)

The radiant intensity can be obtained from the expression for the energy density by multiplying the above
expression by

. The curious factor of 1/4 arises because

At any instant, on an average, half of the waves are directed towards the wall of the cavity and another half
is directed away from it. This gives a factor of 1/2.
We need to average over all angles. In computing the radiant power, we get a factor of

, which

averages to 1/2. The radiant intensity is given by

Black Body Radiation :


Rayleigh- Jeans' law is roughly in agreement with the thermal radiation curves at long wavelengths. However,
at short wavelengths, it gives infinite energy density as

as

. This is clearly unphysical.

The failure of the classical wave theory to explain the observed radiation curve in the ultraviolet end of the
electromagnetic spectrum is known as ultraviolet catastrophe .

See the animation


Planck's Theory :
In 1900, Max Planck suggested that oscillating atoms could emit or absorb energy in tiny bursts of energy
called quanta . The energy of the quanta is proportional to the frequency of radiation. Planck's suggestion
imparts a discrete or particle nature to radiation. If the frequency of radiation is
quantum associated with it is

where the constant of proportionality

, the energy of the

is called Planck's constant . Its value in SI units is

J-s. Thus the possible energy of a mode with frequency

is

. According to Boltzmann distribution, the probability of a mode having an energy


temperature

is given by

, where

. Here,

(3)

Prove Eqn. (3).

at a

is the Boltzmann constant and

is the absolute temperature. Thus the average energy of a mode is

Exercise 2

where

(Hint

Treat

as

continuous

variable

and

show

that

the

right

hand

side

is

).

Using
(2), by

to be the average energy of the mode instead of

, the energy density is given, instead of Eqn.

(4)

Exercise 3
Show that Eqn. (4) reduces to Rayleigh - Jeans' expression for long wavelengths i.e. as
for

. [ Hint : use

Exercise 4
Show that, in terms of wavelength, the expression for radiant intensity is given by

(5)

Example-1
Find the temperature for which the radiant energy density at a wavelength of 200 nm is four times that of the
density at 400 nm.
Solution

Substituting values of

and

, we get

which gives, on simplification

. On solving, the temperature works out

approximately to be 18,500 K.
Stefan's Law
The power radiated by the black body per unit area is

To evaluate the integral, substitute

, so that

. We get

The value of the integral

is known to be

, so that

where

is known as Stefan's constant.


For a body with emissivity (the ratio of radiation emitted by a body to that predicted by PLanck's law for an
ideal black body)

, the power radiated from a unit area of the surface is

In addition to emitting radiation, a body at temperature


temperature is

also absorbs radiation. If the surrounding

, the power absorbed per unit surface area is

Example-2
Estimate the radiant energy emitted by a blackbody at 6000 K.
Solution
According to Stefan's law the radiant power emitted per unit area is

Example-3
Estimate the fraction of radiant power of Example 1 which is emitted in the visible region of the spectrum.
Solution
According to Planck's radiation formula, the power per unit area is given by

Substituting

where
Taking

and

, the expression reduces to

are respectively the upper and the lower limits of


Hz and

Hz, we get

corresponding to visible spectrum.


and

. Thus

The integral above has to be done numerically, for instance, by Simpson's method. A crude estimate gives the
value of the integral to be approximately 2.41. Thus

watts, which is about 36% of the

total emitted radiation.


Exercise 5
A spherical black body of radius 2m is at 27

C. Find the power radiated.

[Ans. 22077 watts]


Exercise 6
Total energy radiated from a blackbody source is collected for one minute and is used to heat a quantity of
water. The temperature of water is found to increase from

C to

C. If the absolute temperature of

the blackbody were doubled and the experiment repeated with the same quantity of water at
temperature of water.(Ans. 28

C, find the

C)

Example-4
The earth receives 1.4 kW of power from the sun. Assume that both earth and the sun to be black bodies. If
the radius of the sun is

m and the earth-sun distance is

m, calculate the

temperature of the sun.


Solution
According to Stefan's law, the power radiated by the Sun per unit area is
sun, the total power radiated is

total power is equal to the surface area of a sphere of radius

Thus

is the radius of the

This power radiates outward from the sun. If at a distance

See the animation

. If

, the power received per unit area is


times this amount. Thus,

, the

Substituting

and

m, we get

K.

Exercise 7
Using the above distances and the calculated temperature of the sun, estimate the equilibrium temperature of
the earth.
( Hint : First determine the total amount of power collected by the earth by observing that

section of

the earth collects all the power falling on the earth. In equilibrium, this amount is equal to the power radiated
from the earth..Ans.

K.)

Wien's Displacement Law :


The wavelength at which the radiant intensity is maximum is inversely proportional to the temperature of the
black body. According to this law, hotter objects emit most of their radiation at shorter wavelength, which
would make them appear more bluish. Similarly, cooler objects radiate in the red end of the spectrum,
making them appear red.

See the animation


The radiant intensity at a given temperature has a maximum when

0
which gives

This equation is to be solved numerically. Substituting

, the equation becomes

Black Body Radiation :


Numerical solution of this equation gives

See the animation


Substituting the values of

where

and

we get Wien's law

is the wavelength at which the radiation intensity is maximum at a temperature

. If

is expresses in cm, the relationship is given by

Exercise 8
The surface temperature of the sun is about 6000 K. What is the wavelength at which the sun emits its peak
radiation intensity ?
(Ans. 483 nm)
Exercise 9
Taking the mean temperature of the surface of the earth to be 10

C, calculate the wavelength at which the

earth emits maximum radiation.


(Ans. 10

, i.e. the earth emits mostly in infrared.)

Example 5
The exercise above shows that the sun emits mostly in the visible region. Compare the total intensity of
radiation emitted by a star of similar size as the sun whose surface temperature is 7200 K.
Solution
The total intensity is given by Stefan's law,

However, the star emits its peak intensity in the blue end of the spectrum as the wavelength at which the
radiation intensity being inversely proportional to the temperature is given by

Exercise 11
The black body spectrum of an object A has its peak intensity at 200 nm while that of another object of same
shape and size has its peak at 600 nm. Compare radiant intensities of the two bodies.
(Ans. A radiates 81 times more than B)
Cosmic Microwave Background
According to the big bang theory , the universe, at the time of creation was a very hot and dense object.
Subsequently it expanded, bringing down the temperature, the present temperature of the universe
isapproximately 2.7 K. As a result, the peak intensity of radiation is given by

The wavelength lies in the microwave region. This is known as the cosmic microwave background. The
energy density at this temperature is obtained by multiplying
J/m

by

, which gives

Recap
In this lecture you have learnt the following
Traditional picture of light being a wave cannot explain several phenomena such as blackbody radiation and
photoelectric effect.
A blackbody is a an object whic absorbs all the radiation that falls on it.

A blackbody is also a perfect emitter, i.e. it emits all the radiation that it absorbs.
Intensity of radiation emitted by a blackbody is proportional to the fourth power of its temperature (Stefan's
law).
The wavelength at which the emitted radiation has the maximum intensity is inversely proportional to its
temperature (Wien's law).

Classical Rayleigh-Jeans' formula leads to the ultraviolet catastrophe at short wavelength.


Planck proposed that oscillating atoms emit or absorb radiation in quanta. Using Boltzmann distribution, he
derived a formula for radiation which satisfactorily explains the blackbody radiation spectrum.

Module 5 : MODERN PHYSICS


Lecture 24 : PHOTONS
Objectives
In this lecture you will learn the following
Radiation itself is quantized and consists of a collection of particles called photons.

The phenomenon of photoelectric effect and its characteristics.

How classical wave theory of radiation fails in explaining photoelectric effect.

Einstein's theory of photoelectric effect.


PHOTONS
Planck's explanation of black body radiation was revolutionary as it suggested that atoms could exchange
energy only in multiples of quantum of energy. Five years later, in 1905, Einstein put forward a theory of
photoelectric effect which suggested that the quantum of energy was not a property associated with the
radiation emitted by atoms but is a property of radiation itself. Radiation, according to Einstein's theory
consists of discrete bundles of energy, called photons. Thus, electromagnetic energy is seen as a collection
of photons. A photon is characterized by an energy

Further, each photon carries a momentum

, related to the frequency by the relationship

given by

In Einstein's special theory of relativity, the energy of a particle of rest mass

and momentum

is given

by

which implies that photons have zero rest mass.


Exercise 1
Calculate the wavelength of a 2eV photon. (

J.)

(Ans. 620 nm)


Example-6
Earth receives 1.4 kW of energy from the sun. If it is assumed that the sunlight consists of monochromatic
radiation of wavelength 600 nm, how many photons arrive at the earth every second ?
Solution
If
per second is

nm, the corresponding frequency is

Hz. Thus the number of photons

Example-7
Assuming the formula for black body radiation to the valid for the universe, calculate the number density of
photons in the universe due to cosmic microwave background.
Solution
Taking the expression for energy density in the interval

and

the number density of photons with energy in this frequency interval is obtained by dividing the above
. The total number density of photons is obtained by integrating the above expression
expression by
over all frequencies

Substitute

The integral has to be done numerically, say by using Simpson's rule. The value of the integral is 2.4, which
gives the number density of photons of the cosmic radiation to be

per m

Exercise 2
Assuming the sun to be a black body, calculate the number of photons emitted by the sun every second.
(Ans.

Photoelectric Effect
When light falls on certain metals, electrons are ejected from the surface of the metal. In the arrangement
shown in the figure, the wire marked anode is held at positive potential with respect to the curved plate
marked cathode
When light of certain minimum frequency falls on the cathode, electrons are emitted in all directions. These
electrons are called photoelectrons . Some of these electrons reach the anode wire which provides a path
to the electrons to give rise to a mesurable photo-current . By making the anode more positive with
respect to the cathode, more electrons are attracted towards the anode and the photo-current increases.
When the anode potential is such that all the emitted electrons reach the anode, any further increase in the
anode voltage does not increase current any further.

See the animation


If the voltage is reversed making the anode negative with respect to the cathode, the electrons get
decelerated and only the more energetic of the electrons can reach the anode.

If the reverse voltage is such that even the electrons which are ejected with the maximum kinetic energy
cannot overcome the potential, the photo-current becomes zero. The reverse voltage which is just enough to
stop the most energetic photoelectrons is called the stopping potential. If
energy of the electrons, the stopping potential is

defined by

The photoelectric effect exhibits the following features :

is the maximum kinetic

Photoelectrons are not ejected unless the frequency of incident light is above a certain threshold value
. The value of

depends on the material of the cathode.

If the frequency of incident radiation


photoelectrons to be emitted. If
emitted. For

is greater than

, even a light of very weak intensity will cause

, even the most intense light will not cause photoelectrons to be

, the photo-current increases linearly with the intensity of light.

The maximum kinetic energy of the photoelectrons depend on the frequency of incident radiation and not on
its intensity.
The emission of photoelectrons is almost instantaneous. i.e. there is no time lag between the emission of
electrons and switching on of the light source.
Failure of Classical Wave Theory
According to the classical wave theory, when electromagnetic wave falls on the surface of a metal, an atom
on the surface will absorb energy from the electric field of the wave. The rate at which the energy is
absorbed depends on the surface area of the atom. An electron can be dislodged from an atom once it
absorbes sufficient amount of energy. By increasing the intensity of light (irrespective of its frequency) more
energy can be transferred to the atom causing electrons to be ejected.
What is observed is that unless

photoelectrons are not emitted, no matter how intense the

radiation is.
Further, according to wave theory, the kinetic energy of emitted electrons would increase with the intensity
of light as it would impart more energy to an electron. However, the kinetic energy of photoelectrons is
found to depend only on the frequency of radiation and not on the intensity.

Another problem with the classical theory is that it would predict a time lag between the time light falls on a
surface and the instant photoelectrons are emitted. The reason why one would expect such a time lag is that
the surface are of an atom is very small, as a result of which an atom can only absorb a small fraction of
energy that falls on the surface. The following example gives a rough estimate of the expected time lag.
However, it is observed that the emission of electrons is practically instantaneous, with time lag, if
any, being less than

seconds.

Example-8
Consider a light source such as a laser with a power output of 1mW spread over a narrow beam of cross
section 0.1 cm
theory.

falling on a surface of a metal. Estimate the time lag of photoelectron emission as per wave

Solution
Taking atomic diameter to be of the order of

cm, the area exposed to the beam is

The fraction of light energy absorbed is


every second is

cm

. Thus the energy absorbed from the beam


J, which is approximately 6 eV (1 eV =

J). The

amount of energy required to ionize an atom by dislodging an electron is typically 10 eV. Thus it takes about
1.6 seconds to absorb the required energy which is rough estimate of the time lag.
Einstein's Photoelectric Equation :
According to Einstein's explanation, photoelectric effect occurs due to absorption of a single photon by an
electron in the atom. When radiation falls on a metal surface, an electron may absorb one quantum of
. Some of the absorbed energy, W, will be used to separate the
energy and increase its energy by
electron from the metal surface. The surplus energy appears as the kinetic energy of the emitted electron

The electrons which are more tightly bound to the metal (e.g. electrons which lie two or three atomic layers
below the surface) require more energy to be removed. We define Work Function

of a metal as the

minimu energy that must be supplied to an electron at the metal surface to dislodge it from the metal. Such
electrons are emitted with maximum possible kinetic energy. Thus Einstein's equation becomes

Since kinetic energy cannot be negative, the above equation implies the existence of a minimum frequency
for photoemission to take place

Using this, we can reqrite rewrite Einstein's equation as

To stop such maximum energy electrons from reaching the anode, we must apply a reverse potential

. Thus

given by

Photoelectric Effect
We can experimentally determine the stopping potential corresponding to various values of incident light
frequency

. The linear plot of

curve is given by

versus

enables us to determine the work function. The slope of the

, which is used to determine the Planck's constant.

Work Function for some metals in eV


Metal Work Function
Cs

1.9

Na

2.3

Co

3.9

Al

4.1

Cu

4.7

Ag

4.7

Pt

6.4

Exercise 3
The minimum energy required to remove an electron from a metal is 2.5 eV. What is the longest wavelength
of radiation that can cause photoelectrons to be emitted from such a metal ?
(Ans. 495 nm)
Exercise-4
The work function of Potassium is 2 eV. If the surface of the metal is illuminated by a radiation of 360 nm,
what
will
be
(i)
stopping
potential,
(ii)
energy
of
the
fastest
photoelectron ?
(Ans. (i) 1.45 eV (ii) 1.45 V)
Exercise 5

The maximum kinetic energy emitted from the surface of a metal has a value equal to twice its work
function. By what factor should the frequency of incident radiation be increased so that the kinetic energy is
doubled ?
(Ans. 5/3)
Example 9
A monochromatic source of light with a wavelength 200 nm and power output of 2 watts is held at a distance
of 0.1m from the surface of an aluminium foil. Aluminium has a work function of 4.2 eV and an atomic radius
of 0.15 nm. Take the photo-emission efficiency to be 2.5%. Calculate
the kinetic energy of the fastest and the slowest photoelectron emitted,

the average number of photons falling on an atom of Al per second,

the number of photoelectrons emitted per unit area per second.


Solution
200 nm corresponds to a photon energy of

J, which is equal to 6.2 eV. Thus the kinetic

energy of the fastest electron is

eV. The kinetic energy of the slowest

electron is zero.
The light falling on an unit area at a distance
W/m

from the source has an intensity of


. The amount of radiation captured by an atom of radius

is

The number of photons is obtained by dividing this by the energy of a single photon. Thus the number of
photons captured by an atom on the foil is

per second.

The photo-emission efficiency is the ratio of the number of photoelectrons emitted from a surface to the
number
of photons falling on the surface in a given time. The number of electrons falling on unit area of the foil is
per second. With 2.5% efficiency, the number of electrons
emitted is

/m

-s.

Example 10
Radiation from a black body at 6000 K strikes the surface of a metal with work function 2 eV. What fraction
of the black body's total radiant intensity is effective in producing photoelectrons ?
Solution
The work function 2 eV corresponds to a threshold wavelength

nm. Thus irradiance due to

very small wavelengths up to 621 nm will cause photoelectrons to be emitted. Using the expression for the
radiant intensity, the fraction of the total intensity is

where

. The integral in the numerator has to be done numerically while the

value of the denominator is known to be

. The value of the numerator is found to be

2.775, which gives the fraction to be 0.43.


Exercise-6
Radiation from a black body at a temperature of 500 K falls on a metal with a work function of 0.2 eV. Find
the longest wavelength of the spectrum capable of releasing photoelectrons from the surface. What
percentage of the total radiant energy of the black body contributes to the process ?
(Ans. 6.19

m, 29%)

Recap
In this lecture you have learnt the following
Planck had suggested that atoms can emit or absorb radiation in bursts called quanta. Einstein proposed that
quantum nature is an inherent property of radiation itself.
When light falls on some metals, electrons are emitted from their surfaces. This is known as the photoelectric
effect.
Classical theory is inadequate in explaining several features of photoelectric effect. For instance, the emission
of photoelectrons is practically instantaneous without any measurable time lag between shing of light and
emission of photoelectrons.
The photoelectrons are not emitted, no matter how intense is the radiation, unless the frequency of incident
radiation exceeeds a critical value.
Einstein's was able to explain photoelectric effect on the basis of photon theory of light. Einstein's euation
states that the maximum kinetic energy of emitted photoelectrons is the difference between the energy of
the incident quantum and the work-function for the material.

Module 5 : MODERN PHYSICS


Lecture 25 : Compton Effect
Objectives
In this course you will learn the following
Scattering of radiation from an electron (Compton effect).

Compton effect provides a direct confirmation of particle nature of radiation.

Why photoelectric effect cannot be exhited by a free electron.


Compton Effect
Photoelectric effect provides evidence that energy is quantized. In order to establish the particle nature of
radiation, it is necessary that photons must carry momentum. In 1922, Arthur Compton studied the
scattering of x-rays of known frequency from graphite and looked at the recoil electrons and the scattered
x-rays.
According to wave theory, when an electromagnetic wave of frequency

is incident on an atom, it would

cause electrons to oscillate. The electrons would absorb energy from the wave and re-radiate
electromagnetic wave of a frequency

. The frequency of scattered radiation would depend on the

amount of energy absorbed from the wave, i.e. on the intensity of incident radiation and the duration of
the exposure of electrons to the radiation and not on the frequency of the incident radiation.
Compton found that the wavelength of the scattered radiation does not depend on the intensity of incident
radiation but it depends on the angle of scattering and the wavelength of the incident beam. The
wavelength of the radiation scattered at an angle

.where

is given by

is the rest mass of the electron. The constant

is known as the Compton

wavelength of the electron and it has a value 0.0024 nm.


The spectrum of radiation at an angle

consists of two peaks, one at

and the other at

effect can be explained by assuming that the incoming radiation is a beam of particles with

. Compton

Energy

Momentum
In arriving at the last relationship, we use the energy - momentum relation of the special theory of
relativity , according to which,

where

is the rest mass of a particle. Since photons are massless (

), we get

Compton's observation is consistent with what we expect if photons, considered as particles, collide with
electrons in an elastic collision.
Derivation of Compton's Formula
Consider a photon of energy

and momentum

colliding elastically with an electron at

rest. Let the direction of incoming photon be along the x-axis. After scattering, the photon moves along a
direction making an angle

with the x-axis while the scattered electron moves making an angle

the magnitude of the momentum of the scattered electron be

. Let

while that of the scattered photon be

See the animation


Conservation of Momentum
x-direction :
(1)
y-direction :
(2)

From Eqns. (1) and (2), we get

(3)
Conservation of Energy : (relativistic effect)
If the rest mass of the electron is taken to be

, the initial energy is

and the final energy is

. Thus

(4)
From Eqn. (4), we get, on squaring,

Thus,

On substituting expression (3) for

Recalling

Using

and

in the above equation, we get

and on simplification, we get

, we get Compton's formula

is known as the Compton Wavelength of an electron.

Exercise 1
Show that the angle

by which the electron is scattered is related to the scattering angle

of the

photon by

Exercise 2
Is Compton effect easier to observe with I.R., visible, UV or X-rays ? In a Compton scattering experiment
the scattered electron moves in the same direction as that of the incident photon. In which direction does
the photon scatter ?
(Answer : X-rays,

.)

Exercise 3
A 200 MeV photon strikes a stationary proton (rest mass 931 MeV) and is back scattered. Find the kinetic
energy of the proton after the scattering.
(Ans. 60 MeV)

Reason for the unshifted peak in the spectrum


When a photon strikes an atom (say carbon atom in a graphite crystal), it may scatter from a loosely
bound electron, which is essentially free. In this case there is a measurable shift in the wavelength of the
scattered photon. It is also likely that the photon scatters from an electron that is tightly bound to an
atom. In such a case, the mass appearing in Compton's formula must be replaced by the mass of the
carbon atom itself, which is approximately 20,000 times heavier than an electron. The maximum
wavelength shift of the photon for scattering from a free electron is twice the Compton wavelength of an
nm. In case of scattering from the carbon atom, the maximum wavelength

electron, i.e.
shift is approximately

nm, which is very small. Thus we find an intensity maximum at an

wavelength which is essentially equal to that of the incident wavelength.


A free electron cannot absorb a photon and increase its energy as doing so would violate energymomentum conservation.
Consider a free electron at rest which absorbs a photon of energy

). The final

. According to relativistic principle, if the momentum of the

energy of the electron would be


electron is

(and momentum

, the total energy is given by

. When the electron absorbs the incident

photon, the momentum of the photon would be transferred to the electron. Since the electron was initially
at rest (i.e. with zero momentum), its final momentum is

which simplifies to

. Thus we have

, which is not possible.

The reason why an electron bound to an atom can absorb a photon ( as in Compton effect) is that the
electron can share some of the resulting momentum with the ion which has a much larger mass
Example 11
A photon of wavelength 6000 nm collides with an electron at rest. After scattering, the wavelength of the
scattered photon is found to change by exactly one Compton wavelength. Calculate (i) the angle by which
the photon is scattered, (ii) the angle by which the electron is scattered and (iii) the change in the energy
of the electron due to scattering.
Solution :
Since the change in wavelength is one Compton wavelength,

, i.e.

. Thus

the photon is scattered at right angles to the incident direction.

Initial momentum of the photon is

The final momentum of the photon is

Thus the final momentum of the electron is

. The angle that the

final direction of electron makes with the x-axis is

The change in the energy of the electron is negative of the change in the energy of the photon which is
keV.
Exercise 4
A photon of wavelength 6000 nm scatters from an electron at rest. The electron recoils with an energy of
60 keV. Calculate the energy of the scattered photon and the angle through it is scattered.
(Ans. 147 keV,

Example 12
A photon of frequency

scatters from an electron at rest and moves in a direction making an angle of 60

with the incident direction. If the frequency of scattered photon is half that of incident photon, calculate
the frequency of the incident photon.
Solution
Since frequency is halved, wavelength is doubled. Thus change in wavelength

is equal to

Using Compton's formula

The corresponding frequency is

Hz., which is a gamma ray photon.

Exercise 5
A photon scatters from a proton, initially at rest. After the collision, the proton is found to scatter at an
angle of 30
with the original direction of the incident photon with a kinetic energy of 100 MeV. Find (i)
the initial energy of the photon and (ii) the angle through it is scattered
Hints : The rest mass of proton is 938 MeV. Total energy of a relativistic particle is

Use these to determine momentum of the scattered proton. Use momentum and energy conservation.
Answer (i)

MeV (ii)

Exercise 6
Find the smallest energy that a photon can have in order to be able to transfer half of its energy to an
electron at rest (rest mass of an electron is 0.5 Mev)
(Ans. 0.256 Mev)
Exercise 7
A photon has the same wavelength as the Compton wavelength of an electron. What is the energy of the
photon in eV ?

(Ans. 0.51 MeV)


Recap
In this lecture you will learn the following
Compton effect is a process in which x-rays collide with electrons and are scattered.
Unlike the prediction of classical wave theory, the wavelength of the scattered radiation does not depend
on the intensity of radiation but depends on the scattering angle and the wavelength of the incident beam.
Compton effect can be explained by considering radiation to be of particulate nature.

Compton effect is best exhibited with short wavelength radiation like x-rays.
A free electron cannot absorb a photon because it is not possible to simultaneously satisfy energymomentum conservation.

Module 5 : MODERN PHYSICS


Lecture 26 : Wave Nature of Particle - the de Broglie Hypothesis
Objectives
In this course you will learn the following
Matter at very small length scale behave like waves.

de Broglie hypothesis associates a wavelength

with matter waves.

Electron diffraction from crystals which confirm wave nature of electrons.

Bohr's model of hydrogen atom and its relationship with matter waves.

Double slit experiments performed with electrons give results similar to Young's experiment for light.

Heisenberg uncertainty principle.


Wave Nature of Particle - the de Broglie Hypothesis
In experiments like photoelectric effect and Compton effect, radiation behaves like particles. de Broglie, a
french physicist asked whether in some situations, the reverse could be true, i.e., would objects which are
generally regarded as particles (e.g. electrons) behave like waves ? In 1924 de Broglie postulated that we
can associate a wave with every material object. In analogy with photons, he proposed that the wavelength
associated with such a matter wave is related to the particle momentum

where

through the relationship

is the Planck's constant

Example 13
Calculate the wavelength associated with a cricket ball of mass 0.2 kg moving with a speed of 30 m/s.
Solution :

Exercise 1
Neutrons produced in a reactor are used for chain reaction after they are ``thermalized", i.e., their kinetic
energies are reduced to that of the energy of air molecules at room temperature. Taking the room
temperature as 300 K, estimate the de Broglie wavelength of such thermal neutrons. (mass of neutron =
kg.)
(Ans. 0.145 nm)

Exercise 2
Calculate de Broglie wavelength of a proton moving with a velocity of
(Ans.

m/s.

m/s)

Example 14
What is the speed of an electron if its de Broglie wavelength equals its Compton wavelength ?
Solution :
We need to use relativistic formula for momentum

where

is the rest mass of the electron. We have

Solving,

Exercise 3
The resolving power of a microscope is approximately equal to the wavelength of light used to illuminate
the object. In an electron microscope , instead of light, the object is irradiated with a beam of electron. If
the resolving power of an electron microscope is 0.01 nm, find the kinetic energy of the electrons used.
(Ans. 15 keV)
Wavelike behaviour of a macroscopic object is difficult to detect as the wavelength is very small.
However, wave nature of particles may be detected in diffraction experiments where the dimensions of the
obstacles are comparable with the wavelength of matter wave incident on the obstacle.
Electron Diffraction from a Crystal
Consider

beam

of

electron

with

speed

m/s

corresponding

to

wavelength

nm. Such a wave may be diffracted by gratings with separation of similar order as
that of the wavelength. Crystals provide such natural gratings.
Davisson - Germer Experiment :
Experimental confirmation of de Broglie hypothesis was provided in 1926 by Davisson and Germer, who
studied diffraction of a beam of electrons from the surface of a nickel crystal.

A beam of electrons from a heated filament, accelerated through a potential difference


is made to strike
the surface of a crystal of Ni. Electrons are scattered in all directions and may be detected by an array of
detectors located at various angles of scattering. It is found that the intensity of scattered beam is
maximum at some particular angles of incidence, in the same manner as the case when a beam of x-rays
strikes the crystal
In Davisson - Germer experiment, the electron beam was accelerated through a potential difference of
volts. The kinetic energy

of the electron is thus 54 eV. The de Broglie wavelength associated

with an electron accelerated through a potential difference

Using numerical values of

and

may be expressed as

, we get a relationship of the form

The wavelength of the electrons in Davisson - Germer experiment can be calculated from the above to be
0.167 nm.
Exercise 4
Through what potential difference should an electron be accelerated to have a de Broglie wavelength of 1?
(Ans. 150 volts)
Exercise 5
An electron is released at a large distance from a proton. What will be the wavelength of the electron when
it is at a distance of (i) 1 m (ii) 0.1 nm from the proton ? [Hint : The potential through which the electron

moves is

.]

m (ii)

(Ans. (i)

m)

Bragg condition is satisfied when the path difference between beams scattered from two adjacent planes is
. For constructive interference of order

a multiple of the wavelength. The path difference is


have

we

Electron Diffraction from a Crystal


In case of Ni crystals, the interplane separation
satisfied for first order (

i.e. for

nm, so that one expects Bragg condition to be

) for

. Thus we expect a strong interference effect for scattering angle

Example 15
One of the diffraction peaks observed by Davisson and Germer for a 65 keV electron beam was at a
direction such that the angle between the incident beam and the scattered beam is 60
crystal spacing is this peak seen in the first order ?

. For what value of

Solution :
For

, the angle

. A kinetic energy of 65 keV corresponds to


J. Equating this to

, the momentum

kg-m/s. The wavelength corresponding to this momentum is


m. The crystal spacing is given by

Exercise 6

Thermal neutrons having a wavelength of 0.145 nm are diffracted by a crystal of lattice spacing 0.29 nm.
Find the angle at which the first order diffraction maximum occurs.
(Ans. 14

Bohr Model :
Bohr's model of an atom, which was very successful in explaining the spectra of hydrogen like atoms is
based on the following postulates :
Electrons move in stationary orbits around the nucleus. As long as an electron is in such an orbit, it does
not y
radiate. However, it emits ( absorbs) radiation when it makes a discontinuous transition form an orbit with
energy
b

to an orbit of lower (higher) energy

. The frequency of emitted (absorbed) radiation is given

The angular momentum of an electron in a stationary orbit is an integral multiple of

where
is an integer.
Using Bohr model, one can show that the wavelength of the radiation emitted when an electron makes a
transition from an orbit with quantum number

where

to an orbit with

is called the Rydberg constant, which has a numerical value

, is given by

The de Broglie hypothesis may be used to derive Bohr's formula by considering the electron to be a wave
spread over the entire orbit, rather than as a particle which at any instant is located at a point in its orbit.
The satble orbits in an atom are those which are standing waves. Formation of standing waves
require that the circumference of the orbit is equal in length to an intergral multiple of the wavelegth. Thus,
if

is the radius of the orbit

which gives the angular momentum quantization

Exercise 7
Calculate the wavelength of an electron the ground state of hydrogen atom. (First Bohr radius of hydrogen
atom is 0.053 nm)
(0.33 nm)
Example 16
If an electron makes a transition from
radiation and (ii) the recoil speed of the electron.

to

, determine (i) the wavelength of emitted

Solution :
The wavelength of emitted radiation is

The wavelength is

nm. The momentum associated with this radiation is

By conservation of momentum, this is also the magnitude of the momentum imparted to the atom as a
whole. The recoil speed of the electron is

Double Slit Interference with electrons


We know that when a coherent source of light is incident on a Young's Double slit, an interference pattern is
observed. The intensity when only the slit

is open is

, while the intensity with

When both slits are open, the expression for the intensity at any position

where

is the phase difference between the waves arriving at

phase difference

open is

on the screen is given by

from

and

. Points where the

, the intensity is a maximum

If the phase difference is an odd integral multiple of

Note that at the minima less energy is received with both slits open than is received with any one slit open,
which is strange.
Let us repeat the experiment with beams of particles, such as electrons, traditionally considered localized
objects. The screen consists of an array of detectors which can record arrival of an electron. Experiment is
done with a beam of electrons and the number of times a detector records the arrival of electron is noted.
This is plotted as a histogram. The distribution of the electrons in the detectors at various points on the
screen is what corresponds to the intensity pattern stated above. The plot can be looked upon as a
probability distribution curve.
Let us focus our attention on a detector located at some position
determine the probability of an electron arriving at

are detected by the detector at

total number of electrons that are emitted by the source


is open, let this fraction be

. Similarly, with

. We close one of the slits and

. This is done by simply finding out the fraction of the

closed but with

. When

open, we determine the

corresponding fraction at the same position. If we open both the slits, we would expect the fraction to be
, as every electron must pass either through
experimentally is that

or through

. However, what one finds

Clearly, the result is absurd in the sense that it says that when both slits are open, there are particles which
neither goes through the slit

nor goes through the slit

. Let us try to look at the actual situation. To

keep track of which electron came through which slit is easy enough. We put a source of light near each of
the slits, so that when an electron passes through one of the slits, it scatters light and we can see a flash.
If we do the experiment this way, keeping track of the particle, we find that

and there is no

contradiction. However, if we do not keep track of which slit each electron goes through, we get the
distribution pattern shown in curve C. What is even more funny is that in curve C there are some points
(minima) where the number of particles is even less than that which reach these points when only one slit is
kept open.
We define a probability amplitude

such that

. In terms of probability amplitude, it turns out

that if we keep track of electrons by watching them,

However, if we do not observe the electrons the probability distribution is given by

The reason behind this paradox is that when we observed the electron, the electron has to interact with
whatever probe we use for observing, in this case, with the light source. This interaction can alter the
chance of arrival of the electron at the point P.
Can we use a weaker source of light to reduce the effect ? The answer clearly is no because a weaker
source does not mean photon energy is different, it simply means that there are less number of photons. If
there are less number of photons, some of the electrons will escape without being detected by them. Those
which are detected are distributed according to

(1)

while those which escape being detected are distributed according to

(2)

and the net result that we get is a weighted mean of the two distributions.
Principle of Complementarity :
The above behaviour illustates an important principle of physics called the principle of complementarity.
When we tried to determine which slit the electron went through, we were investigating the particle nature
of an electron. However, only a wave can simultaneously go through both the slits and the interference
effect was a manifestation of the wave nature. According to the principle of complementarity, an object has
both the particle property and wave property. However, in a given experiment either one or the other
property can be determined. It is not possible to siumultaneously get information on both the
particle nature and the wave nature of an object in the same experiment.
What the photons do to the electron is to transfer momentum so that the electrons are scattered in
different directions from their original directions. Can we then reduce the effect by using light of longer
wavelength which can impart less momentum causing less disturbance ? The answer once again is no
because if the wavelength is too long we will not be in a position to say whether the scattered electron
or from

came from the slit

as the position cannot be determined with a precision better than the

wavelength.
Heisenberg Uncertainty Principle
The clue to this apparent paradox is in Heisenberg's uncertainty principle, according to which, there are
limitations on the accuracy with which experiments aimed at determining which hole a particular electron
goes through, may be performed, while still not disturbing the interference effect.
Classically, one can simultaneously measure the position and momentum of a particle to infinite precision.
In the quantum mechanical world, there is an uncertainty associated with every measurement. Whenever
we attempt to measure the position of an object with a precision

, there is some uncertainty

in

the momentum measurement, which we cannot get rid of, because, in order to make a measurement we
must disturb the system.
Thought Experiment
In the thought experiment gedanken shown in the figure, the electron is assumed to be at restly below the
objective of the microscope. The electron is illuminated by gamma rays, which have a short wavelength.
Using a short wavelength yields high resolution. According to principles of optics, a microscope can resolve
objects to within

, which is related to the wavelength

through

See the animation


However, gamma rays behaves like a particle with momentum

and is Compton scattered. In

order to be observed by the microscope,

ray must be scattered within an angle

of the cone. Since

the electron is initially at rest, the total moment of the electron- photon system is the momentum of the
photon.
Consider two extreme limits when the gamma ray photon is scattered by an angle
to the left extreme
wall of the microscope. The x-component of the total momentum is the sum of the x component of the
momentum of the scattered electron,

which is

and the x component of the momentum of the scattered photon,

, i.e.

,where

is the wavelength of the scattered photon.

Similarly, when the photon is scattered to the extreme right, the total momentum is

,where

is the wavelength of the scattered photon. As the angle


is small, and we may take

is small, the Compton shift


. The total x-component of the

momentum incases must be the same, each being equal to momentum of the incident photon.
Thus

so that the momemntum uncertainty of the electron is

giving

It may be noted that one can always determine the momentum along the y-direction with any desired
degree of accuracy when there is position uncertainty along the x-direction.
Exercise 8
The position of an electron is determined with an accuracy of 0.01 nm. Find the uncertainty in its
momentum.(Ans.

kg m/s)

Example 17
A beam of electrons with a de Broglie wavelength of
Calculate the angular spread of the beam by diffraction.

m passes through a slit

m wide.

Solution :
Initial momentum of the electron is along the x-direction.

Due to the uncertainty in the position of the electron along the slit direction (y-direction), there is a
momentum spread along the y-direction, given by

See the animation

The angular spread of the beam is given by

so that the spread


.
Similar uncertainty relationship exists for other pairs of observables. For instance, if we wish to measure the
energy

of a particle with an accuracy

. According to uncertainty principle, we may determine the

time at which the particle has such an energy only with an uncertainty

where

Example 18
Find the wavelength spread of a 1 nano-second pulse from a ruby laser with a wavelength of 630 nm.
Solution :
Since
Using

, and

, we have

. Thus

Hz.

, we get

so that

Recap
In this course you have learnt the following
The dual nature not only is exhibited by radiation but is also associated with matter. In some experiments
matter shows wave character.

de Broglie hypothesis poastulates a wavelength of

with a particle having a momentum

Experimental confirmation of wave nature of matter comes from experiments such as Davisson Germer
experiments on electron diffraction from crystals. It is seen that the intensity of scattered beam is
maximum at those points where one would expect Laue spots in x-ray diffraction assuming the electronsare
waves with de Broglie wavelength.
Bohr model can be understood by postulating that stable orbits in atoms are those which are standing
waves of electrons.
One can perform double slit experiment with electrons, similar to the way Young's double slit experiment is
performed with light waves. The intensity pattern obtained on a screen is very similar in both cases.
According to the principle of complementarity one cannot obtain information on both the wave nature and
particle nature of matter or radiation in the same experiment.
Heisenberg uncertainty principle states that one cannot precisely measure both position and momentum of
a particle in the same experiment.

Module 5 : MODERN PHYSICS


Lecture 27 : Nuclear Energy
Objectives
In this course you will learn the following
Mass energy equivalence.

Nuclear binding energy.

Nature of nuclear forces.

Energy released in nuclear fission.

Controlled chain reaction and principle of a nuclear reactor.

Nuclear fusion.
Nuclear Energy
Energy is one of the main requirement of sustaining human civilization. Conventional sources of energy, which
account for over 85% of the total energy consumed by the mankind, are fossil fuels. Fossil fuels were formed
over millions of years from remains of plants and animals by action of pressure and heat and are now found
beneath the earth's surface. When such fuels are burnt, they release chemical energy trapped in them. Fossil
fuels consist of coal, petroleum and natural gases. Coal is a solid hydrocarbon with nearly 75% of the world's
deposit being found in China, USA and the Russian Federation. Though most abundant and the lesat expensive
of the three, burning coal causes a lot of environmental problems as it releases harmful chemical SO

, which

can cause acid rain. In addition, release of large amount of carbon dioxide is responsible for global warming.
Oil and natural gas deposits are limited and they are being depleted at a very fast rate. Though the nature
makes them, fossil fuels are non-renewable sources of energy as it takes nature millions of years to make
them while humans consume them at a fast rate. Nuclear energy is produced from practically unlimited
amount of energy trapped in all matter. Production of nuclear energy does not produce particulate impurities
like NO

, SO

etc. and also does not release CO

. In this sense, nuclear energy is a clean fuel. However,

there are several issues connected with disposal of radioactive nuclear waste and decommissioning of old
nuclear power plants, which makes the production of nuclear energy a contentious subject. In addition, the
technology that produces nuclear power can also be used to produce nuclear bombs, which makes transfer of
nuclear technology a delicate issue. Nuclear energy is made available by fission and by fusion . In the
following, we will discuss the principle behind production of nuclear energy.
Mass - Energy Equivalence
According to Einstein's special theory of relativity, a particle of mass
given by the relation

where

has equivalently an amount of energy

is the speed of light in vacuum which has a numerical value (approximately)

m/s

. The

theory of relativity introduces the concept of rest mass , which is the mass an object has when it is at rest
relative to an inertial frame. If the mass of an object in such a frame is
energy given by

, the object has an equivalent

. In addition to the rest energy, the object may have a kinetic energy

because of motion that it has with respect to the inertial frame. The total energy of the object may be written
as

Since the product of mass and the square of the velocity of light has the dimensions of energy, it is possible to
express

as a product of some mass


, where

times

, so that the energy of the object may be written as

. In this expression

is the relativistic mass of the body, which

depends both on the rest mass of the body and the state of motion of the body. According to the special
theory of relativity, for a body moving with a speed

with respect to an inertial frame,

Atomic Mass Unit


Nuclear mass is usually expressed in terms of an unit known as the atomic mass unit , denoted by the letter u
.One atomic mass unit is defined as one twelfth the mass of an atom of the most abundant isotope of Carbon ,
C
viz.,

In high energy physics, it is common to use mass and energy interchangeably and frequently one expresses
mass in terms of MeV, which actually stands for the corresponding energy equivalent. In terms of the above
units, the masses of a neutron and a proton are as follows : Mass of a neutron =

939.57 MeV/c

Mass of a proton =

= 1.007 825 u

=1.008 665 u

938.28 MeV/c

Nuclear Binding Energy :


The constituents of a nucleus are neutrons and protons, collectively known as neucleons . Mass of a nucleus
is always less than the sum of the masses of its constituent nucleons, the difference between the two is called
the mass defect and the equivalent energy the binding energy of the nucleus. Binding energy of a nucleus
is the amount of energy required to separate the nucleus into its constituent nucleons.
Example 19
The Helium nucleus

He (also called the alpha particle ) has a mass of 4.001 506 u. Determine its binding

energy in terms of the atomic mass units and in MeV.


Solution :

Alpha particle has two protons and two neutrons. Binding energy calculation in atomic mass unit is shown
MeV, i.e. 7.074 MeV per

alongside. In terms of MeV, the binding energy is


nucleon.
Exercise 1
An isotope of Uranium

U has a nuclear mass of 235.043930 u. Calculate the binding energy per nucleon in

MeV.
(Ans. 7.59 MeV)
Exercise 2
The most abundant isotope of Uranium

U has a nuclear mass of 235.050788 u. Calculate the binding

energy per nucleon in Mev.


(Ans. 7.57 MeV)
For a nucleus

X of mass

with atomic number

and mass number

, the binding energy is given by

The adjoining figure gives a plot of the binding energy per nucleon of the elements. It may be noted that the
nucleus
Fe is the most tightly bound nucleus with a binding energy of about 8.8 MeV per nucleon. This is
the reason why the iron group of nuclei are the most stable of the nuclei.

See the animation


Nuclear Force
The large binding energy suggests that the nucleons in an nucleus are very tightly bound. As a nucleus
consists of protons which, being positively charged, would repel each other by Coulomb force, and neutrons
which are charge neutral, the nucleons in a nucleus obviously cannot be glued together by electromagnetic
forces which would actually make them fly apart. What binds the nucleons together is a new type of force,
called the strong force . The characteristics of the strong force are
the force is extremely short ranged - the force exists when the distance between nucleons becomes of the
order of nuclear dimensions (

1 Fermi =

m).

it is charge independent - the force is the same whether it is between a pair of protons or a pair of neutrons
or between a proton and a neutron.
It is strong and attractive , relative to electrmagnetic forces it is about a hundred times stronger.
Inside a nucleus there is a competition between the repulsive Coulomb force and the attractive strong force. In
most nuclei the strong force dominates. However, for heavier nuclei, there is a delicate balance between the
two which can be easily disturbed leading to a fission of such nuclei. Spontaneous fission is of rare occurrence
as the half life for fission is very high. The way to effect fission in a nucleus is to excite the nucleus - the
energy required to initiate fission being between 5 to 6 MeV.
Fission :
Nuclear energy has been harnessed for power production, primarily through fission of Uranium, Thorium and
Plutonium, with Uranium being the most common fissile material today. Looking at the binding energy curve,
one can see that lighter nuclei have less binding energy than the heavier ones. Thus if a heavy nucleus breaks
up into fragments, the sum of energies of the fragments would be less than the energy of the initial nucleus.
This is the principle behind release of energy through fission. As the released energy is in MeV, fission released
a million times more energy than chemical processes like burning of coal or oil which release energy of the
order of electron volts.
Fission of

U:

Natural uranium consists of 98.275% of

U, 0.72% of

U and 0.005% of

U. Most of the nuclear

energy produced in the world today is produced by fission of


U. (An isotope of uranium
is also
fissionable though it does not naturally occur but is produced by exposing thorium to neutrons.) The process
of fission is initiated by capture of a slow neutron, called thermal neutron as the kinetic energy of such a
neutron is of the order of thermal energy of air molecules, by uranium nucleus. Upon capuring a neutron, the
uranium nucleus gets into an excited state and the delicate force balance inside the nucleus gets disturbed.
The excited nucleus breaks up into two fragments of nearly comparable size.

See the animation


Various fragments that are produced by fission of
U are distributed as shown in the adjoining figure. Some
of the possible reactions that take place are as follows :

See the animation

Chain Reaction
Notice that in each of the reactions neutrons are produced. The average number of neutrons produced in
fission reactions is 2.5. Thse neutrons can then be used to produce Chain Reaction.
Consider reactions where 2 neutrons are produced. The first neutron causes one uranium nucleus to split into
two fragments producing 2 neutrons, which in turn can cause two more uranium to fission, producing four
U fuel is spent. This is
fragments and four new neutrons. The chain reaction will continue till all the
roughly what happens in an atom bomb. The run away chain reaction needs to be controlled, if one is to use it
for power generation. This is done by removing excess neutrons so that the chain reaction can proceed at a
slow pace till all the fuel is burnt. Removing excess neutrons is achieved by inserting control rods which
contain neutron absorbing material. Cadmium and boron rods are usually used for such purpose.

Fission :

Nuclear Power Reactor


When fission reaction occurs tremendous amount of energy is liberated. While some energy appears as
radiation, most of the energy is in the form of kinetic energy of the fragments. The fragments dissipates most
of their kinetic energy as heat, which can be used to boil water and the resulting steam can be used to turn
turbines to generate electricity. The following are the stages in the working of a power reactor.
Enrichment
The percentage of
U in the naturally occurring uranium is only about 0.7%. In a nuclear reactor, the
percentage of U-235 is increased by a process called enrichment . This is usually achieved either by gaseous
diffusion or by a centrifuge. In the diffusion process, natural uranium, which is first converted to gaseous

uranium hexafluorid (UF

) is passed through a series of semi-porus membranes which permit passage of the

gas that contains the lighter U-235 but not the component containing the heavier isotope U-238. The
technique is somewhat inefficient and it requires several stages of diffusion barriers to achieve the desired
concentration of about 4 to 5% of U-235. In the centrifuge technique the natural UF

is spun at high speed

which separates the heavier and the lighter components as they are subjected to different centrifugal forces.
Newer techniques of enrichment using lasers are now available.
Reactor Core
The core is the centre of the reactor which contains the fuel which is used in running the reactor. Control rods
to absorb excess neutrons are introduced into the core. Usually a heavy steel vessel surrounds the core. A
reactor which produces enough neutrons to sustain controlled chain reaction is called critical . Criticality not
only depends on the nuclear reaction producing neutrons but also depends on availability of enough uranium
which must absorb a released neutron. Enrichment of the fuel ensures this for a reactor. The mass of the
fissile material required to sustain chain reaction is said to be critical mass . If a reactor cannot sustain chain
reaction, it is called sub-critical. On the other hand if the chain reaction proceeds uncontrolled, as in an atom
bomb, the reactor is called supercritical .
Moderators
A neutron produced in a nuclear reaction must be slowed down before it can be capured by another fuel
element. Such thermalization of neutrons is performed by collision with substances which have light masses
such as water, heavy water or graphite. These are known as moderators.
Neutron absorbers
We have seen that control rods of neutron absorbing materials like Cd are inserted to make sure that the
reactor does not become supercritical. It is not possible to mechanically control the rate of insertion or
withdrawal of control rods which are emitted the instant the nuclear reaction takes place. However, the
radioactive decay of the fission fragments produce additional neutrons which are called delayed neutrons.
For instance, in the first mentioned fission reaction above, U-236 fragments into

Cs and

Rb, in addition

to releasing two prompt neutrons. These fission fragments themselves are unstable and undergo different
types of radioactive decays. Delayed neutrons are associated with fragments which undergo beta decay. In
case of

, about 0.03% of the decay is through beta emission, half life against beta decay being 25

seconds

Similarly

Rb undergoes beta decay to

Sr in about 1.4% cases producing neutrons with a time lag of

about six seconds. The delayed fraction allows mechanical control to be established for controlling both prompt
and delayed neutrons.
Coolant
The heat generated in the core is to be removed for being used to heat water to generate steam and eventual
generation of electricity. The thermal energy is removed by a liquid coolant that flows throgh pipes in the
reactor core. Based on the method used for exatracting fission energy from the core of the reactor, there are
two primary types of reactor. They are
Boiling - Water Reactor (BWR)
In a boiling water reactor, water in pipes circulates inside the core. The water gets heated, generates steam
which is then used to drive turbines. As the water enters the core, there is a possibility of its becoming
radioactive. Further, in case of a rapture of the pipe due to extreme heat, it could lead to accidents.
Pressurized - Water Reactor (PWR)
In these reactors water is extracted in two steps. The primary coolant, as in the case of BWR circulates inside
the core. However, the water circulates under great pressure so that it does not become steam. The heat is
transferred through a heat exchanger to a secondary coolant, which may be used to drive a turbine. As the
secondary coolant does not enter the core, it does not become radioactive.

In addition to the above, liquid metal reactors in which the coolant used is a metal like liquid sodium are also
in existence.
Fast Breeder Reactor
While fission of U-235 can yield substantial energy, the world's supply of U-235 is limited. However, it is
possible to use the neutrons given out by fission reaction to breed fuel from non-fissionable isotopes like U238. As U-238 is more plentiful than U-235, it is an attractive option. U-238 can absorb a fast neutron given
out by a fission reaction to yield fissionable plutonium. A typical breeding reaction is as follows :

U-238 absorbs a fast neutron and undergoes two successive beta decay to give rise to plutonium. The core of
a fast breeder reactor consists of an inner part which contains the fission fuel consisting of enriched
and

PuO

and an outer part which consists of U-238, depleted uranium and

UO

Th in some combination.

The fast neutrons that are produced during fission of the inner part is captured by the outer layer to breed
fissionable plutonium and U-233. Thus the reactor breeds more fuel than it consumes, which accounts for the
name. No moderators are used in the reactor as the neutrons should not be slowed down. This is also the
reason why the primary coolant used is not water which could slow down neutrons. The usual coolant used in
the primary loop is liquid sodium.

Example 20
Calculate the energy released in the fission reaction

Solution :
The masses of the reactants and the product (in units of u)are as follows :
Reactants :

mass of
mass of

U
n

=235.043929
=

1.008665

------Total mass of reactants=236.052594


Products :
mass of

Rb

92.922042

mass of

Cs

= 140.920046

mass of 2

2.017330
-------

Total mass of product = 235.859418


The products being lighter than the reactants by

u. This

amounts to

MeV of energy.

Fission produces about 200 Mev of energy of which about 175 MeV is the kinetic energy of fission fragments,
the reamaining energy is distributed as the kinetic energy of neutrons and energy associated with photons,
neutrinos and other radioactive products.
Exercise 3
Calculate the energy released in the fission reaction

The masses of the product are as follows :


Kr = 88.917630 u and

Ba = 143.922953 u.

(Ans. 173.27 MeV.)


Nuclear Fusion
Energy may also be produced by two light nuclei fusing to give a heavier nucleus. This would happen at the
initial part of the binding energy curve where the rise in binding energy per nucleon with atomic number is
sharp. For instance, if two deuterium (heavy hydrogen) nuclei fuse, we may get

We can calculate the energy balance as follows


Reactants :
mass of two

Products :
mass of

He

= 3.016029

mass of

= 1.008665
-------

Total mass of product = 4.024694


The mass deficit is

u, corresponding to 3.267 MeV. Though energy

from fusion can be an enormous source of power because of abundance of deuterium in sea water, fusion of
nuclei can be achieved only at extremely high temperatures (of the order of over a million degree !). At such
temperatures, no solid container can be used to contain the material used for fusion (the reactants and
products would be completely ionized - the mixture of the electrons and the nuclei being in a state of plasma).
Controlled thermonuclear fusion has only been possible on a laboratory scale and fusion reactor remains a
distant possibility. It may be mentioned that it is the energy released from fusion that keeps our sun (and
other stars) hot and luminous. Fusion is possible in sun because of extremely high density of protons at the
centre of the sun.
Exercise 4
Calculate the energy released in the fusion process (use mass data given above)
(Ans. 17.59 MeV)
Environmental Issues Connected With Nuclear Reactors :

He +

Though nuclear energy can be plentiful in availability and, unlike fossil fuel, is a clean fuel as it does not give
rise to particulate impurities, there are several issues which have been causes of concern. Some of the issues
are as under :
Nuclear waste (which consists of fission products) contains highly radioactive material. Though the level of
radioactivity for most components goes down fast, the nuclear waste have to be kept away from human
contact as they could cause serious illnesses like cancer. Safe disposal of nuclear waste is a cause of major
concern.
Like any major power station, nuclear reactors are also likely to have accidents due to faulty design or things
out of control. The worst recorded accident which led to several deaths and other fallouts from the released
radioactivity occurred in Chernobyl, near Kiev (Ukraine). A faulty design led to a fire and explosion of the
graphite core. The radioactivity spread over erstwhile USSR and also to parts of Europe, including Scandinavia.
In 1979 one of the plants in Three Mile Island, near Harrisberg in USA, lost coolant leading to over heating
and partial meltdown of the core which caused radioactive leak.
Because of their high cost, nuclear power plants are designed to produce very high power. This leads to
thermal emissions which can lead to local warming at a level much higher than that for conventional power
stations. Though the nuclear power stations do not give out more heat to the surroundings than thermal power
stations per kilowatt of power produced for electricity, the effect of local heating is more for large power
stations.
Recap
In this course you have learnt the following
Einstein's relation
and energy.

obtained from the principle of relativity establishes the equivalence of mass

Mass of a nucleus is less than the sum of the masses of constituent nuclei, the difference is the mass defect
and the equivalent energy is known as the binding energy of the nucleus.
Nucleus is held together by a force that is strong, short ranged and attractive.
The nuclear binding energy is more for heavier nuclei than for lighter nuclei, as a result of which when a
heavier nucleus splits into smaller fragments (fission), energy gets released.
Fission is effected by a thermal neutron striking a heavy nucleus such as
U and disturbing the delicate
balance of forces. Nucleus thereby fissions and in the process liberates more neutrons, which, in turn, can
cause more fission. A nuclear reactor is based on the principle of controlling this chain reaction.
When two light nuclei coalesce to form a heavier nucleus, the process is called fusion. Thermonuclear fusion
can also be a source of great energy provided the process can be controlled.

Module 5 : MODERN PHYSICS


Lecture 28 : Principle of Laser
Objectives
In this course you will learn the following
Principle of Laser - population inversion.

Einstein relation for spontaneous and stimulated emission.

Microwave amplification by stuimulated emission.

Different types of lasers and their applications.


Principle of Laser
We have learnt that the energy levels in an atom are discrete. The lowest possible energy level is known as
the ground state and higher energy levels are called the excited states. As the energy of the excited states
increase, the separation between the adjacent energy levels become smaller and smaller until the separation
becomes so small that the energy levels appear continuous. Such continuous spread of energy is called the
continuum.

There are three ways in which an incident radiation can interact with the energy levels of atoms.
Absorption :
An electron in one of the lower level (ground state or a lower lying excited state) with an energy
make a transition to a higher level having an energy
occur only when the frequency of the incident radiation

can

by absorbing an incident photon. Absorption can


is given by

If this condition is not satisfied, the matter become transparent to incident radiation.

Spontaneous Emission :
Atoms which are in excited states are not in thermal equilibrium with their surroundings. Such atoms will
eventually return to their ground state by emission of a photon. If

is the energy in the excited state

and
the energy of a lower lying state (which could be the ground state), the frequency of the emitted
photon is given by

Stimulated Emission :
In 1917, Einstein showed that under certain conditions, emission of light may be stimulated by radiation
incident on an excited atom. This happens when an electron is in an excited state

and a photon whose

energy is equal to the difference between


and the energy
of a lower lying level (could be the ground
state) is incident on the atom. The incident photon induces the electron in the excited state to make a
transition to the lower level by emission of a photon. The emitted photon travels in the same direction as the

incident photon. Significantly, the new photon has the same energy as that of the incident photon and is
perfectly in phase with it. When two waves travel in the same direction with a constant phase relationship,
they are said to be coherent .
LASER - Light Amplification by Stimulated Emission of Radiation :
In 1958, Charles H. Townes and Arthur L. Schawlow showed that the effect of stimulated emission can be
amplified to produce a practical source of light, which is coherent and can travel long distances without
appreciable spread of the beam width. Such a light source is called LASER, an acronym for Light
Amplification by Stimulated Emission of Radiation.
The principle behind such amplification is simple. Suppose we start with one photon which strikes an atom in
an excited state and releases a photon, we would have two photons and an atom in the ground state. These
two photons, in turn, may be incident on two more atoms and give rise to four photons, and so on.

In the figure above, the excited state atoms are shown in red while those in the ground states are in green.
However, the simple picture above does not work in practice because of the following :
The time for which an electron remains in an excited state is approximately
seconds. Thus it is
difficult to keep atoms in excited states till they are stimulated to radiate a photon. The excited atom is
more likely to de-excite spontaneously . Photons released through spontaneous processes are emitted in
random directions and are not coherent with the incident photon.
The photons that are incident and those which are generated may be absorbed by atoms in ground states,
leading to depletion in the number of photons.
Einstein Relations : A and B Coefficients
Consider a two level system. Let there be
per unit volume in a higher energy level

number of atoms per unit volume in the energy level


. Let

and

. Total population density in the

system is

If the atoms are in thermal equilibrium with the surrounding at a temperature


the two levels are given by Boltzmann distribution

, the relative population in

This equation shows that as the temperature increases, the population of excited states increase. However,
the population of an excited state always lies lower than the population of the ground state, under
equilibrium condition.

See the animation


The distribution of atoms in the two energy levels will change by absorption or emission of radiation. Einstein
introduced three empirical coefficients to quantify the change of population of the two levels.
Absorption - If

is the probability (per unit time) of absorption of radiation, the population of the

upper level increases. The rate is clearly proportional to the population of atoms in the lower level and to the
energy density

of radiation in the system. Thus the rate of increase of population of the excited state

is given by

Spontaneous Emission - If
the excited state is

is the spontaneous emission probability, the rate of depletion of atoms in


.

gives the average

of an atom in the excited level before the

atom returns to the ground state.


Stimulated Emission - Stimulated or induced emission depends on the number of atoms in the excited
level as well as on the energy density of the incident radiation. If

be the transition be the transition

probability per unit time per unit energy density of radiation, the rate of decrease of the population of the
excited state is

The rate equation for the population of the upper level is

When equilibrium is reached, the population of the levels remain constant, so that
rate of emission equals rate of absorption, so that

Using the Boltzmann facor

, and simplifying, we get

and the

If we regard the matter to be a blackbody and compare the above expression for the energy density with the
corresponding energy density expression derived for the blackbody radiation, viz.,

we get

and

The last equation shows that the probability of stimulated emission is equal to that of absorption. In view of
this we replace the two coefficients by a single coefficient
spontaneous emission coefficient will be called the
probability to the stimulated emission probability is

so that for low temperatures, when

and term them as

- coefficient. The

- coefficient. The ratio of spontaneous emission

, spontaneous emission is much more probable than

induced emission and the latter may be neglected. For high enough temperatures, stimulated emission
probability can be significant though for optical frequencies, this requires very high temperature. For
microwave frequencies the stimulated emission processes may be significant even at room temperatures.
Exercise 1
Find the ratio of the probability of spontaneous emission to stimulated emission at 300 K for (a) microwave
photons (

(Ans. (a)

Hz.) and (b) optical photons (

(b)

Hz).

.)

Recap
In this course you have learnt the following
Atoms and ions have sharp energy levels. Electrons occupy these energy levels from bottom upwards.
The pouplation of energy levels is determined by Boltzmann distribution according to which the population in
the lower energy levels is more than that in the upper levels. If for some reason the population in the upper
levels become more than that in the lower level (population inversion), the atoms would make transition to
the lower levels by emission of radiation.
Einstein equations (called A and B coefficients) establish connection between spontaneous and induced
emission. Laser is based on the principle that under certain conditions, induced emission may dominate over
other processes.
It is not possible to establish a lase with only two energy levels. Different lasers based on three or four levels
are realised in practice.

The utility of laser lies in the fact that it provides a highly monochromatic and coherent beam with a very
high degree of directionality.

Module 5 : MODERN PHYSICS


Lecture 29 : Maser
Objectives
In this course you will learn the following
Population inversion in atomic systems

Principle of MASER

Principle of three level and four level Laser

Different types of Lasers, their properties and applications.


Population Inversion
We have seen that when atoms are in equilibrium with the surrounding, the population of atoms in the
ground state is more than that in any of the excited states. Population of excited states can be increased by
absorption of radiation. However, the life time in the excited states being typically of the order of
seconds, atoms which make transitions to the excited states fall back to the ground state soon thereafter.
This is also indicated by the ratios of the Einstein coefficients. It is, therefore, not possible to keep the
population in the excited states higher than that in the ground state.
The basic principle involved in the operation of laser is population inversion , a situation in which the
population of the excited state is kept higher than that of the ground state.
MASER :
The frequency of microwave photons is
Hz, corresponding to an energy of the order of 0.01 eV. The
energy is of the same order as that of the thermal energy of air molecules. In such a case, the population of
the excited states is comparable to that of the ground state at room temperatures. The process of
stimulated emission can then be used to amplify microwave signal. MASER is an acronym for Microwave
Amplification by Stimulated Emission of Radiation .
Ammonia Maser is such a device for generating electromagnetic waves. Ammonia molecule has two resonant
states with a small energy difference
eV. Geometrically, the two states may be picturised as follows.
The three hydrogen atoms are at the vertices of an equilateral triangle which forms the base of a pyramid
with the nitrogen atom at the apex of the pyramid. The nitrogen atom may be in two possible positions,
either above the hydrgen plane or below it. (Physically, the two states are distinguished by the direction of
their dipole moment in the presence of an electric field.) The molecules make transition from one state to
another by absorption or emission of radiation.

The principle of ammonia maser is to separate the two types of molecules which have different energies
and

. This is done by subjecting the beam to an inhomogeneous electric field in a transverse

direction.

The higher energetic beam is passed through a cavity to which it delivers energy. This is done by having a
in the cavity. If the frequency of the electric field is tuned such

time varying electric field


that

, resonance condition is satisfied and the molecules make a radiative

transition from states with higher energy to that with lower energy.
Three Level Laser :
For optical frequencies, population inversion cannot be achieved in a two level system. In 1956 Bloembergen
proposed a mechanism in which atoms are pumped into an excited state
( such as by an electric pulse or by optical illumination).

The system, in addition to the state

, has an excited state

which is a metastable state, i.e. a state

inwhich the atom has a long life time. Atoms from the upperlevel
state
level

. Life time in the level

by an external source of energy

decays spontaneously to this metastable

is such that the rate of spontaneous decay from level

is slower than the rate at which atoms decay from

between the metastable level and the ground state.

to

to the ground

. This results in a population inversion

The emitted photons are confined to a laser cavity to stimulate further emission from the excited atoms.
Ruby laser works on the principle of a three level system. The pumping power required for such a system is
very high because more than half of the ground state atoms have to be pumped into the upper level to
achieve population inversion.

Four Level Laser :


One of the most popular and low cost lasers is helium-neon laser, which works on the basis of a four level
system with two levels intermediate between the ground state

and the pumpng level

The ground state atoms are electrically pumped to a short lived state
decay to a metastable state

. Between

and

. Atoms from this state undergo fast

, yet another short lived excited state

population inversion takes place between this intermediate state


which the lasing transition occurs.

and the metastable state

exists. A
, between

Properties of Laser Beam :


Laser beams are characterized by the following special properties :
Coherence : Laser beam is highly coherent, i.e, different parts of the beam maintain a phase relationship
for a long time. this results in interference effect. When a laser beam reflects off a surface, the reflected
light can be seen to have bright regions separated by dark regions.
Monochromaticity: Laser beam is highly monochromatic with the spread of wavelength being very small.
Directionality : Laser beam is highly collimated and can travel long distances without significant spread in
the beam cross section.
Types of Lasers and Applications :
Lasers have found wide applications in areas as diverse as optical communications, medical surgery, welding
technology, entertainment electronics etc. What makes such veritable use of lasers possible is the highly
collimated nature of the laser beams and the consequent possibility of delivering a very high energy density
in a limited region of space. Depending on the material used for the active medium , lasers are broadly
classified as

(i)
(ii)
(iii)
(iv)

conventional or gas lasers


solid state lasers
liquid lasers and
semiconductor lasers.
Among the gas lasers, some of the most commonly used ones are Helium - Neon laser, Carbon dioxide laser
and Argon- ion laser.
Helium-Neon Laser :
Helium-neon laser consists of an active medium of a gas mixture with about 80% He and 20% Ne, kept in a
glass chamber at at low pressure. The ends of the chamber are silvered with one end having a perfectly
reflecting mirror while the other end has a mirror which reflects 98%.
Pumping helium atoms to their excited states is provided by electrical discharge at about 1 keV. The mirrors
reflect light back and forth extending the path travelled by light which increases the probability of stimulated
emission. The emergent laser light is primarily in the red region of spectrum at
principle of lasing is as follows.

nm. The

Among many excited states of helium, one of the states is a metastable (long lived) state with an energy
which is 20.6 eV above the ground level. (An electron in this level is not permitted to return to the ground
state by emission of a photon as it would violate conservation of angular momentum.) The ground state of
neon has an electronic configuration of
of levels corresponding to a configuration

. Neon atoms have various excited levels of which there is a set


which are coincidentally removed from the ground state of

neon by 20.6 eV with a small spread of 0.04 eV. Helium atoms which are pumped into the excited state may
collide with the neon atoms in their ground states and transfer their energy to the neon atoms, taking the
latter to their excited levels. The small energy spread of 0.04 eV can be accounted for by the kinetic energy
of colliding atoms. The following figure shows the transitions that takes place. (The figure shows additional
energy levels of helium and neons which are also involved, the principle, however, remains the same.)

Neon has lower lying energy levels at about 18.7 eV above its ground state corresponding to the atomic
configuration

. At any instant there are more atoms in the

than in any of the lower levels,

resulting in a population inversion. Lasing transition takes place between the

level and the

level which emits in the red at a wavelength of 632.8 nm. Lasing also occurs in infrared and far infrared with
emissions at 3.39

m and 1.15

m as shown in the figure. Less prominent emissions in the green part of

the spectrum (543 nm) also takes place.


Helium-neon laser, which is a low cost device is not particularly an energy efficient device, its energy output
being a few milliwatts whereas the pumping power is between 10 to 100 watts. However, its primary utility
lies in the coherence and directionality of the beam as well as the energy that can be delivered over a small
area because the power, though small, is concentrated over a small beam diameter giving a power density
between 0.1 to 1 kW/m . Coherence of the beam is useful in interferometric and holographic applications.
Collimation and the ability to traverse traverse long distances is used in measuring and sensing devices, as
barcode scanners etc. As the emission is in the red - green region of the visible region, He-Ne lasers are
used as tools in advertising in light shows and in entertainment electronics.
Carbon dioxide Laser :
Carbon dioxide laser was invented by C. K. N. Patel of Bell Laboratories in 1964. It is basically a three level
system, though a fourth level is also involved in the transition. It contains a mixture of gases of CO

, N

and He, approximately in the ratio 1:1:8. Note that inspite of the name, the primary constituent is helium. In
case of CO

laser, in addition to the quantized electronic levels of the atoms, the vibrational and the

rotational states of the molecules are also involved in the transition. Electrical discharge is used to excite the
nitrogen molecules to higher excited states, which are long lived and cannot decay by emission of photons.
The excited nitrogen molecules collide with carbon dioxide molecules which happen to have a second excited
level (the pumping level) precisely at the excitation energy of the nitrogen molecules. A population inversion
occurs and lasing transition takes place in far infrared (

m) and also at

m. Helium has an

important role to play in the laser operation. Carbon dioxide molecules to return to the ground state by
collision with helium atoms. In addition, helium improves the thermal conductivity of the gas mixture without
which the gas would become hotter and would have an increased population in the excited levels negating
the effect of population inversion.
Exercise 1
The adjacent figure shows the energy levels of nitrogen and carbon dioxide molecules. Using this, explain
laser action in CO

lasers.

Nitrogen-carbon dioxide laseer system is the most efficient and powerful among all the lasers, the output

efficiency being about 12%. Because of such high power, CO

lasers are used in industrial applications like

cutting and welding. It has been used extensively in military applications for rangefinding through a
technology called LIDAR (Light Detection and Ranging), which is very similar to RADAR which uses radio
waves in that the distance of an approaching aircraft may be determined by measuring time delay in arrival
of a laser pulse at its source after reflection. CO

lasers have been used in medical applications in thoracic

and retinal surgery.


Argon-ion Laser
Argon-ion laser is an example of a continuous wave (CW) laser, which is a type of laser in which a
coherent beam is generated continuously as an output. Because of this it has applications in communication
technology. It can be operated in CW mode at about 25 different wavelengths from 350 nm in the ultraviolet
to 520 nm in the blue green, but the strongest lines are 488 nm and 514 nm. Because of such prominent
lines in the visible region, the laser is used in light shows.
The energy levels of neutral argon as well as that of a singly ionized argon (Ar
) is shown alongside. The
neutral atom is pumped into its excited state. As we are dealing with noble gases, the excitation energy is
very high. Ionization of neutral Ar atoms requires a voltage pulse of about 10 kV. The process of ionization
creates argon ions in their ground as well as excited states. The ground state of Ar
configuration of

, which has a

, is about 15.75 eV above the ground level of neutral Ar. Stimulated emission

occurs between the

excited state of the ion and the

excited of the ion. The ion

in the latter excited state drops to the ground state of the ion by a spontaneous emission at 74 nm. From
this state electron capture returns the ion to the ground state of the neutral atom.

See the Animation


High energy required for ionizaton makes the laser rather energy inefiicient. However, high energy outputs
up to about 20 W can be obtained in CW mode. Because of this, argon ion lasers have applications in
scientific studies where high power is required. In addition, the laser is used extensively in opthalmic and
general surgery.
Solid State Lasers :
Typical examples of solid state lasers are Ruby lasers, Nd-glass laser etc. Ruby laser consists of rods of
ruby, which are Al

with about 0.05% Cr with a highly polished mirror at one end and a semi-

transparent mirror at the other. A xenon flash bulb is used to excite chomium atoms to their excited states.
Lasing transition at 694.3 nm takes place between states of chromium. Pulsed beam with bursts lasting

seconds can be generated with such a laser. Power output of solid state lasers are high and
they have wide variety of applications like cutting, welding, printing and xeroxing, medical and surgical
applications etc.
Semiconductor Lasers :
Semiconductor lasers make use of junction between different semiconductors as the active medium. Laser
action is achieved by heavily doping the junction which ensures availability of a large concentration of
electron hole pairs for recombination. Ends of the device are polished so that spontaneously emitted light
travels back and forth enabling stimulated emission. Emission wavelengths span a wide range from near red
into far infrared. Power output of semiconductor lasers can be from a few milliwatts to several watts under
cw conditions while pulsed power of several hundreds of watts may be made available. Semiconductor lasers
have wide range of applications. These include their use in communication systems, environmental sensing,
audio compact discs, laser printing etc.
Recap
In this course you have learnt the following
The basic principle of laser operation is to create conditions so that the population at a higher level is more
than that in the ground state. This is known as population inversion.
Maser works on the principle of stimulated emission of atoms from an excited state to the ground state.
When microwave is incident on a cavity containing excited atoms, it triggers the atoms or molecules to
radiate energy with the same wavelength as that of the incident wave. The wavelength produced is so stable
that it can be used as a clock standard.
It is not possible to achieve population inversion in a two level system. It is, however, possible to achieve
population inversion between a metastable state which lies between the ground state and an upper excited
state. Electrons pumped into the upper level decay fast to the metastable level which undergoes lasing
transition to the ground state.
Laser beams are highly monochromatic, have directionality and phase coherence. Lasers are now used in a
wide variety of applications such as welding and cutting retinal surgery, compact discs, optical
communications etc.

Module 6 : PHYSICS OF SEMICONDUCTOR DEVICES


Lecture 30 : Energy Band Diagram
Objectives
In this course you will learn the following
Difference between electrical properties of metals, semiconductor and insulators.

Band theory of metals and semiconductors.

Free electron theory of metals and concept of Fermi energy.

Calculation of density of state for free electrons.


Modern electronics, which has revolutionized our way of life, is based on interesting properties of a class of material
known as semiconductors.
Semiconductors have resistivity values intermediate between those of metals and insulators. While typical metallic
resistivity is between

to

ohm-m, that of silicon, which is a representative semiconductor is

ohm-m. Materials with resistivities higher than


plastics are typical insulators.

ohm-m are considered to be insulators. Glass, rubber and many

Temperature coefficient of resistivity :


The resistivity of metals increase with the increase of temperature. This is because with increase in temperature, ions
in a solid vibrate more causing electrons to scatter more frequently from them. The semiconductors, on the other
hand, have a negative temperature coefficient of resistivity, i.e., their resistivity decreases with increase in
temperature.
An empirical formula for resistivity of metal is

where

is the resistivity at a reference temperature

, ususlly taken to be 0

is known as thecoefficient of resistivity, which is a small positive number (For Cu

C or 20
/

C. The constant
C.

An corresponding formula for resistivity of semiconductors (or of insulators) is

where

is the resistivity at 0

C (i.e. at 273 K) and

is Boltzmann constant and


measured. Here
order of 1 eV and much larger for insulators.

is the absolute temperature at which the resistivity is

is the bandgap energy, which, for semiconductors is of the

Charge carriers
Conductivity arises due to motion of charge carriers in a material when placed in an electric field. The primary charge
carriers in a metal are electrons. In gases and solutions, the carriers could be charged ions. The density of carriers in a
per m .
metal is very high, being of the order of
Charge carriers in semiconductors are electrons and vacancies, the latter being known as holes . Typical carrier
density in a semiconductor is about
carrier densities

which is substantially lower than that of metals. Insulators have negligible

Energy Band Diagram


Electrical properties of materials are best understood in terms of their electronic structure. We know that the energy
levels of isolated atoms are discrete. When atoms are brought together to form a solid, these energy levels spread out
into bands of allowed energies. The effect is qualitatively understood as follows by considering what happens when a
collection of

atoms, which are initially far apart are brought closer.

When the spacing

between adjacent atoms is large, each atom has sharply defined energy levels which are denoted

by

etc. As the atoms are far apart their orbitals do not overlap. In particular if each atom is in its

ground state, the electrons in each atom occupy identical quantum states. As the distance starts decreasing, the
orbitals overlap. The electrons of different atoms cannot remain in the same state because of Pauli Exclusion
Principle. Pauli principle states that a particular state can at most accommodate two electrons of opposite spins. Thus
atoms are brought together, the levels must split to accommodate electrons in different states. Though they
when
appear continuous, a band is actually a very large number of closely spaced discrete levels.

See the animation


Conductors, Insulators and Semiconductors :
When an electric field is applied to any substance, the electrons can absorb energy from the field and can move to
higher energy levels. However, this is possible only when empty states with higher energies exist close to the initial
states in which the electrons happen to be in. If there is a substantial energy difference between the occupied electron
state and the higher unoccupied state, the electron cannot absorb energy from the electric field and conduction cannot
take place. Thus conduction takes place only in partially occupied bands.
In case of a metal, the bands which arise from different atomic orbitals overlap and the electrons can absorb energy
from an electric field (or absorb thermal or light energy). The electrons in such partially filled bands are called free
electrons .

For an insulator there is a wide gap (

eV) between the lower occupied band,known as the valence band ,

and the higher unoccupied band, called the conduction band . No electron can exist in this forbidden gap To
promote electrons from lower levels to higher levels would require a great amount of energy. It is incorrect to say that
electrons in an insulator are not free to move around. In fact, they do. However, as there are as many electrons as
there are states, the electrons only trade places resulting in no net movement of charges.

Semiconductors, like insulator have band gaps. However, the gap between the top of the valence band and the bottom
of the conduction band is much narrower than in an insulator. For comparison, the gap in case of Silicon is 1.1 eV
while that for diamond, which is an insulator is about 6 eV.
Quantum Mechanical Concepts
Formation of bands can only be understood on the basis of quantum mechanics. Earlier, we had seen that an object
behaves both as particle and as wave. According to de Broglie theory, an electron having a momentum
associated wave with a wavelength

has an

. Schrdinger proposed an equation for the wave associated with a particle of

having a total energy


which is moving in a potential
. The Schrdinger equation, which is as
mass
fundamental to quantum mechanics as Newton's laws are to classical mechanics, is given by

According to quantum mechanical hypothesis, the wavefunction


a particle of energy

being at a point

is interpreted as the probability amplitude of

. The square of the wavefunction

the point, so that the probability of finding the particle anywhere in space given by

gives the probability density at


is unity.

This is called the normalization of the wavefunction. It is also postulated that the wavefunction and its first derivative
are continuous and single valued.

Wavefunction of a Free Electron


The energy- momentum relationship for a free electron is given by

The

relationship (known as the dispersion relation ) is a parabola.

Free Electron Model of a Metal


The valence electrons in a metal are detatched from the individual atoms and move in the metal like a gas of free
particles. This leaves the atomic cores positively charged. In the free electron model of a metal, it is assumed that this
positive charge of the cores is uniformly distributed throughout the metal. Such an assumption essentially removes any
details of the crystal structure of the metal. Nevertheless, it is a good starting point in understanding behaviour of
electrons in a metal.
As the interaction between the electrons themselves are also ignored, one can simply consider the motion of a single
electron which is moving in a constant potential, which can be conveniently taken to be zero. The electron can be
within which the potential is constant (taken to be zero)

taken to be confined within a box of dimension

and outside which it is infinite. The Schrdinger equation for the electron within the box is

Defining

, the equation reads

The equation may be solved by separation of variables by substituting

in eqn. (A) and dividing the resulting equation by

Since

. We get

is constant and the first three terms depend upon

for all values of

separately, the above equation can be satisfied

only if each of the three terms is constant, i.e.

with

The solutions of the above with boundary condition (i.e. wanishing of wavefunction at the walls) gives

where

being any non-zero positive integer. Thus the complete solution (with normalization

constant) is

and the energy

where
is the volume of the crystal.
At absolute zero temperature, electrons fill up available states from minimum energy upwards satisfying Pauli exclusion
principle. Each distinct energy level, specified by a combination of

and

is occupied by two electrons of

opposite spins. The maximum value of energy is known as Fermi energy and is denoted by

Density of States
Density of states at an energy

is the number of states per unit volume available per unit enit energy interval with

energy between

and

. This would require counting of states, i.e., enumeration of different values of

corresponding to the energy of states within this interval. This is obviously a difficult task. However,
given the large dimension of a crystal, the states are very closely packed and and one can essentially treat the
values as continuous.
Equation of constant energy given by eqn. (B) is a sphere in

space with a radius

As the points in this space are separated from the adjacent ones by one unit in each direction, each point effectively
occupies a volume
states. As each

in the

space. Thus a unit volume in

space contains

number of

state can accommodate two electrons (corresponding to two distinct spin states), the number of

electrons per unit volume of

space is

Since the constant energy surface in

space is a shhere, we can define Fermi wave vector

a sphere corresponding to the Fermi energy

The volume of Fermi sphere being

which gives the density

as equal to

as the radius of

by

, the number of electrons

that lie within such a sphere is

Using eqn. (C), we can obtain an expression for the density of states
number of states lying within energy interval
energy

from the number below

The unit of density of states

and

. Since, by definition,

is the

, we may simply subtract the number of states below

. We have

is (eV)

Density of States and Fermi Energy

See the animation


Free electron theory can be used to explain, reasonably satisfactorily, several qualitative properties of metals, such as,
thermal and electrical conductivity, magnetic properties, heat capacity etc. However, as the theory totally ignores the
crystalline structure of metals, it fails to provide a distinction between metals, semiconductors and insulators. The
atoms in a crystal are arranged in a periodic arrangement. Consequently, the electrons in the crystal are subject to a
periodic potential rather than a uniform potential assumed in the free electron model. If this potential is assumed to be
weak, the electrons do not become completely free but nearly so. The model is known as nearly free electron
model.
Recap
In this course you have learnt the following

Metals have free charge carriers. With increase in temperature, the resistivity of a metal increases.
Pure (intrinsic) semiconductors are very similar to insulators. For a semiconductor, the resistivity decreases with
increase in temperature.
Metals have partially occupied band which allow charge carriers to move even when a small amount of energy is
supplied. Semiconductors have occupied valence band and unoccupied conduction band. Carriers can be thermally
excited from valence band to conduction band.
Insulators are similar to semiconductor in their band structure. However, the band gap is much larger than in a
semiconductor.
Behaviour of simple metals can be described by a free electron theory in which an electron is taken to move in a
constant potential. At zero temperature the electron states are occupied from the lowest energy state to a state of
maximum energy, known as Fermi energy.
Density of state is the number of states per unit volume of a substance per unit energy interval. For free electrons
the density of state is proportional to the square root of energy.

Module 6 : PHYSICS OF SEMICONDUCTOR DEVICES


Lecture 31 : Electron in a Potential Well
Objectives
In this course you will learn the following
Motion of electrons in a potential well.

Electron in a periodic potential - Kronig Penny model.

Concept of Brillouin zone.

Origin of gap in the energy spectrum at the zone boundary.

Electron in a Potential Well


A simplified model of an electron inside a crystal is to consider the electron to move in a periodic potential
well structure.
The essential features of the problem is brought out by considering the crystal lattice to be a regular
arrangement of atoms along a straight line with inter-atomic separation
. The variation of potential energy
with distance on a line along the centre of the atoms is as shown in the figure.

See the animation


Before discussing this problem, let us consider the motion of an electron in a single square well potential .
The potential is given by

Having a finite total energy the electron cannot be in the region

or

. Thus the electron

, the one dimensional Schrdinger

wavefunction in these regions vanishes. For the region


equation is

which has a solution

where

and

are constants and the wave number

is given by

By continuity of wavefunction at the boundaries

where

at

which gives

at

which gives

is an integer

, and

. (

inside the well vanish; negative values of


whole wavefunction by

is not possible because it would make the wavefunction


does not give new solution as it amounts to multiplying the

.) Thus the electron energy in the potential well are

The energy levels corresponding wave functions are shown in the figure.

See the animation


The figure below shows the probability densities corresponding to various states.

Exercise 1
Verify that the normalization factor

for the wavefunction for a particle in a potential well is

Electron in a Periodic Potential


Inside a lattice the electron is subjected to a periodic potential, i.e., the form potential repeats itself in
space. Thus if

is the inter-atomic distance in a one dimensional lattice, we have

For potentials that are periodic, the wavefunction satisfies Bloch theorem which states that the form of the
wavefunction is

where

is a periodic function with the same periodicity as that of the lattice, i.e.,

Substituting this in Schrdinger equation for

, we would obtain an equation for

which must be

solved.
A simple model often used to mimic the periodic potential is known as the Krnig-Penny model , the form
of which is shown in the figure.

The potential is periodic with a period


resulting in some values of

. In this case the electrons diffract from the periodic lattice

for which no solution exists. The reason can be traced to formation of standing
. For a crystal with periodicity

waves for such values of

form if the electron wavelength

, as shown above, standing waves would

is an integral multiple of

. Thus the condition for formation of

standing waves is

so that

For each region of k-space in which the energy is continuous is said to be a Brillouin Zone . Thus the First
Brillouin Zone is for
from

values running from


to

diagram where the values of

to

and from
extends from

, the second Brillouin zone


to

to

and so on. The

is called an extended zone scheme .

diagram in a scheme known as the reduced zone scheme in which

One can present

values are

restricted to their values in the first Brillouin zone. The energy band structure in such a scheme is
discontinuous as the bands in the picture are folded back to the first zone using periodicity.

The parameter

in these diagram does not have the interpretation of momentum; however, in analogy with

the case of free particle


to an external force

is known as crystal momentum" . When an electron in a crystal is subjected


, it is this crystal momentum which satisfies the Newton's laws

Origin of Gap in Energy Spectrum


From the energy diagram shown, it can be seen that there are discontinuities that arise at certain values of
. Such gaps occur when the wavelength of electron is such that the condition for Bragg diffration by the
periodic lattice structure is satisfied.
We know that an electron wave incident on such a lattice undergoes reflection if Bragg condition

is satisfied. For waves travelling along the line of atoms in the crystal, the angle of incidence is
the wave vector of the electron for which Bragg condition is satisfied is

Thus energy is continuous in the region


The region between

and

. At

, so that

there is a discontinuity.

is the first Brillouin zone.

We have seen that the solution of the Schrdinger equation for an arbitrary value of

is a plane wave

. This, however, is not true at the zone boundary where Bragg reflection takes place. At such a
boundary the wave function has two components, viz.,

and

. One can form two standing waves

using these two wavefunctions, viz.,

and

. The corresponding

electron densities are proportional to

and

. These two densities have their

maxima respectively at the locations of the atoms and midway between atoms, as shown.

For wave vectors other than at the zone boundaries, the wavefunction being a travelling wave, the
probability density has uniform value. When we calculate the average of the potential energy for the three
probability densities, i.e.

, the potential energy is the largest for the cosine wave and the

smallest for the sine wave, the case of uniform density falling in between. The difference between the two
energies is the energy gap at the zone boundary.
Reciprocal Lattice
We have seen that Bragg diffraction occurs for values of

vectors which are multiples of

. Bragg

diffraction is a consequence of periodicity of the lattice and diffraction pattern forms a geometric image
which bears an inverse relationship with lattice periodicity. Because the scale associated with the points at
which Bragg condition is satisfied has the dimensions of inverse length (as seen in the one dimensional
relation

), the geometrical structure defined by diffraction peaks is called the reciprocal lattice

corresponding to the real space lattice which is also referred to as the direct lattice .
An equivalent way of describing diffraction in crystal is known as von-Laue formulation, according to which
the condition of constructive interference is satisfied if

where
and

is a direct lattice vector,

is the wave vector of the incoming wave,

is any integer.

One can define reciprocal lattice vector

where

is an integer.

Another way of stating Laue condition is

that of outgoing wave

Squaring both sides and using

Since

, we get

is also a reciprocal lattice vector, one can rewrite the above equation as

Geometrically, this implies that Laue condition is satisfied if

lies in a plane that bisects

perpendicularly.

Example-1
The density of free electrons in silver is

per cubic meter. Calculate the Fermi energy of silver in

eV.
Solution
The Fermi momentum is given by

The Fermi energy

Dividing by electronic charge, the Fermi energy is 5.65 eV.


Exercise 2
Taking the density of state of silver to be of the free electron form, calculate the number of free electrons
per unit volume in silver having energy between 4 eV and 4.1 eV.

(Ans.

Exercise 3
Obtain an expression for the free electron density of states in (i) one dimension and in (ii) two dimensions.

(Ans. (i)

(ii)

Fermi Function
At a finite temperature, the electron states are filled by a probability density function

where

is the probability of a particular energy state

being occupied. Electrons and other particles

which follow the above distribution function are called fermions . At

At all non-zero temperatures, the value of

at

given by

is a step function

is 1/2. Thus one of the ways in which Fermi

energy can be defined is the energy level where half of the available energy states are filled.
Exercise 4
Show that

Recap
In this course you have learnt the following
Motion of an electron in a one dimensional potential well was studied. The energy levels are discrete and the
wavefunctions are standing waves with nodes at the wall boundaries.

Motion of an electron in a periodic potential was discussed. The wavefunction satisfies Bloch theorem
accoding to which the solution is a product of a free electron function and another function having the
periodicity of the lattice.
An electron wave moving in a periodic lattice undergoes Bragg diffraction at points in

spacewhere

Braggcondition is satisfied. This leads to creation of a forbidden gap in the energy spectrum inside which no
physical state exists.

Module 6 : PHYSICS OF SEMICONDUCTOR DEVICES


Lecture 32 : Bonding in Solids
Objectives
In this course you will learn the following
Bonding in solids.

Ionic and covalent bond.

Structure of Silicon

Concept of effective mass.

Bonding in Solids
Crystals can be classified on the basis of bonding of atoms to form a solid. There are primarily four types of
bonding in solids, viz., molecular, ionic, covalent and metallic bonds.
Molecular solids are formed by weak inter-molecular forces between molecules. Because of weak binding,
the solids have low melting points
Metallic solids are characterized by free electrons which move freely through the crystal being bonded to
different atoms at different times.
Ionic Bond
In ionic solids, one of the specis of atoms donates electrons to another specis so that each of the atoms may
become more stable by having a noble gas (octet) configuration of electrons. For instance in sodium chloride
crystal,

Sodium with an atomic number 11 has an electronic configuration


atomic number 17 has the configuration

while chlorine with the

. When the atoms come together, Na gives

away one electron and becomes positively ionized while Cl receives one electron and acquires a net negative
charge. The negative an the positive ions are held together by electrostatic interaction. Ionic solids are hard,
brittle, have high melting points and are poor electrical conductors.
Covalent Bond
Atoms can also achieve stable octet configuration by sharing of electrons. For instance, in forming hydrogen
molecule a pair of hydrogen atoms share two electrons.

.
Other examples of covalent bonded crystals are diamond, graphite, quartz (SiO

) etc. Covalently bonded

crystals are very hard, have high melting point ( diamond has a melting point of 3550
electrical conductors. Each electron that is shared is attracted to both the nuclei.

C) and are poor

Example-2
Draw a picture of bond formation in Methane molecule.
Solution
Methane molecule has one carbon atom with the configuration

and requires four electrons

to complete octet. This is done by sharing two electrons with each of the hydrogen atom so that carbon atom

and all the hydrogen atoms are in noble gas configuration.

Exercise 1
Draw a picture of covalent band formation in Ammonia molecule.
(Hint : Nitrogen has 7 electrons with electronic configuration

).

Silicon, which is one of the prominent elemental semiconductors is in the same group as carbon in the
periodic table and like carbon, it has four valence electrons. These electrons form covalent bonds. Crystalline
silicon has the same structure as diamond. The structure of diamond consists of two interpenetrating face
centered cubic lattices which are displaced along the body diagonal by one fourth the distance.

Effective Mass
For a free electron moving under the action of an external force

the equation of motion is given by

Identifying

as the momentum of the particle, we have

We may, therefore, express the mass of the particle in terms of the second derivative of the energy with
respect to the wavenumber

The velocity of the particle can be expressed as

When an electron moves in a lattice (i.e. in a periodic potential), in addition to the external forces, it is
subjected to forces within the lattice. These foreces are generally quite large. We may write the equation of
motion for the electron in the lattices as

One can define the effective mass of the electron in a lattice to be mass of a particle, which, when
subjected to an external force

Using the expression for

The effective mass

would give an acceleration

defined by eqn. (A), i.e.,

above, we can write

is given by

For a band with anisotropy, (i.e., where


mass is direction dependent

relationship is not spherically symmetric), the effective

Note that unlike real mass, the effective mass can even be negative as it is proportional to the curvature of
the band.

Example-3
Calculate the effective mass for a simple cubic lattice whose band structure is given by

at the point (0,0,0) of the k-space.


Solution
Because of symmetry, we have

On the other hand

is the same for all

is given by

and

for

. Thus

Example-4
The

relationship near the conduction band of GaAS is given by

where

. How does it affect the effective mass of electrons near the conduction band minimum ?

Solution
We have

Thus

For values of

removed from

increases with

Exercise 2
Which one of the bands has higher effective mass near

Exercise 3
The energy of an electron in the valence band of a certain one dimensional semiconductor may be written as

where

and

are constants. Sketch the variation of the energy and of the effective mass of the

electron as functions of

. Calculate the effective mass at

and at

given that

and

eV.

(Ans.

kg)

Exercise 4
Electrons in a two dimensional square lattice are in a band whose structure is given by

where

and

edges (

are constants. Analyze the behaviour of electrons near the centre (0,0) and at the
) of the Brillouin zone and show that near these points, the structure may be

approximated as

where
(Ans.

is the effective mass. Determine the value of the effective mass at

kg)

Exercise 5
The

where

relationship for the conduction band of Ge is given by

is the energy of the bottom of the conduction band. The longitudinal effective mass

times the free electron mass while the transverse effective mass

is 1.6

is 0.082 times the free electron mass.

Show that the energy surface is an ellipsoid of revolution with the major and minor axes being respectively
and

Recap
In this course you have learnt the following
Atoms combine to form solids through different types of bonds such as ionic, covalent, molecular and
metallic bonds.
In ionic solids, atoms give out and receive electrons so that each atom completes octet configuration.

Semiconductors such as silicon and germanium are bonded covalently. In such bonding the atoms share
electrons so that each one completes octet.
The effect of lattice potential on an otherwise free electron can be taken into account by defining an effective
mass. Effective mass is proportional to the curvature of the band.

Module 6 : PHYSICS OF SEMICONDUCTOR DEVICES


Lecture 33 : Electrons and Holes
Objectives
In this course you will learn the following
Concept of holes in a semiconductor.

Drift velocity and mobility.

Hall effect in semiconductors.


Electrons and Holes
We have seen that the charge carriers in metals are electrons which are in partially occupied conduction
band. Electrons in fully occupied valence bands do not contribute to net current. We may understand this by
a consideration of the effective mass of electrons near the top of the valence band. It may be seen that near
the top of a band, the effective mass is negative. This means that in an electric field, the electrons which are
near the top of the valence band are accelerated in the direction of the field while those near the bottom are
accelerated in the opposite direction. The overall current in a filled band

is zero. Physically, this

happens because there being no empty state in a filled band, the best that electrons can do is to trade
places.
In semiconductors, the width of the forbidden band is small enough so that electrons near the top of a
valence band can easily jump to the unoccupied conduction band.
Consider what happens if one electron moves from near the top of valence band to the bottom of conduction
band. The net effect of such a transition is to give a negative contribution to the current because had the
electron been there at the top of the valence band it would have moved in the direction of the current
(because of negative effective mass). Equivalently, the effect is to have one more electron moving opposite
to the direction of the field. Mathematically, the current in a nearly filled band is given as follows.

Thus, the vacancy in the valence band behaves like a particle of positive effective mass and positive charge
as far as its behaviour in an electric field is concerned. This vacancy is known as a hole . In a
semiconductor, the conduction takes place by motion of electrons in the conduction band and that of holes in
the valence band.

If

is the energy of the bottom of the conduction band and

is that of the top of the valence band,

the density of states of electrons and holes in these two bands are respectively given by

Mobility of Electrons and Holes


When an electric field
is applied to a metal or a semiconductor, the carriers are accelerated; the equation
of motion being given by Newton's law

Since the carriers continuously collide with atoms, the above equation is valid only during the time between
two successive collisions. The change in velocity between two successive collisions with an interval
by

is given

In the absence of a field the carriers exhibit random motion due to chaotic changes in their thermal velocities
so that the average change in velocity over a long period of time is zero.

However, if

, the carriers have a net motion in the direction of the field (for

) over which the

random motion is superimposed. This is known as the drift velocity . The figure shows drift of an electron in
the field, the abrupt changes in the direction is due to a collision with an atom. The drift velocity is
proportional to the strength of the electric field, the constant of proportionality being known as mobility.
Taking the average time between collisions (called the relaxation time) to be

where

, the average increase in

is given by

velocity between collisions is given by

is the mobility of the carrier.

For a semiconductor we need to replace


by
, the effective mass of the carrier. Mobility depends on
the relaxation time - the longer the interval between successive scattering, greater is the increase in
incremental velocity. Likewise a smaller effective mass means larger acceleration and consequently a higher
velocity.
/V-s.

The unit of mobility is m


Example-5

The average distance travelled by a carrier between collisions is called the mean free path . The electron
mobility in GaAs at 77 K (the temperature at which nitrogen becomes a liquid) is 30 m
effective mass of electron is 0.067

/V-s. If the

, determine the mean free path.

Solution
The relaxation time can be calculated from mobility

The thermal velocity is given from kinetic theory to be

Thus the mean free path is

m.

Exercise 1
Electrons in the conduction band of silicon have effective mass 0.25
holes in one of the valence bands have effectives mass 0.54
free

electron

mass.

Determine

the

relaxation

times

and mobility 0.14 m

and mobility 0.048 m


for

the

carriers.(

/V while the

/V-s,

being
s

s.)
Exercise 2
A sample of copper has an electron drift velocity of 2.5 m/s in an electric field of 500 V/m. Determine (i)
electron mobility and (ii) relaxation time.
(Ans. (i)

/V-s (ii)

s.)

Hall Effect
Hall effect provides a direct evidence of the existence of holes in semiconductors. In a typical Hall effect
experiment, a magnetic field is applied in a direction perpendicular to the direction of current in a flat strip of
a semiconductor. In the figure shown, the current flows along the length (positive x-direction) of the strip.

When a magnetic field is applied in a direction transverse to the current direction, a potential difference
appears across the direction perpendicular to both current and the magnetic field. This transverse voltage is
called the Hall voltage .
The origin of Hall voltage is Lorentz force that acts on charge carriers. The magnetic force acting on the
charges

deflects the charge carriers in a direction perpendicular to the direction of the velocity.

This leads to charge separation because of which an electric field


appears across the strip. The bending
of the trajectory stops when the force due to the electric field balances that due to the magnetic field, i.e.
. The forces being in the opposite directions, their magnitudes are equal, i.e
The current density

is given by

, where

is the density of carriers. Thus we have, on

equating the forces,

By measuring the current density


determine the carrier density. Since

, the magnetic field strength


and

and the electric field

, we may

, we get

The direction of Hall voltage depends on the sign of the charge of the carrier. Hall effect, therefore, provides
a direct way of establishing the existence of holes. We define Hall constant

through

If

is the resistivity of the material of the strip, the mobility of the carrier is given by

When both electrons and holes are present, they drift in opposite directions. with holes moving in the
direction of current and electrons in the opposite direction. As the sign of their charges are opposite, the
Lorentz force acting on them will be in the same direction. Thus both the carriers are deflected towards the
same side and they pile up along one of the edges. As there is no closed path for current to flow in the ydirection, the sum of the electron and the hole current in the y-direction must be zero

where

and

are respectively the electron and the hole densities and

and

the y-component of

the respective drift velocities.


Example-6
A long strip of material of width
along a direction parallel to the edge
of the carriers (of density

cm and thickness
per m

) is 5 m

/V-s.

(1)

Determine the magnitude and direction of the net electric field.

(2)

If

to the width
Solution

T, calculate the potential difference


.

mm is subjected to a magnetic field

. A current of 1 A flows in the strip parallel to its length. The mobility

between the opposite points of the strip parallel

The current

which gives the drift velocity as

Thus the electric field parallel to the length is

The magnitude of the electric field perpendicular to the direction of drift velocity and the magnetic field is

Thus the net electric field is

The transverse potential difference is

Exercise 3
Hinall effect experiment is made in a sample of a flat semiconductor of length 1 cm and width 0.3 cm. The
mobility of carriers in the sample is 4500 cm /V-s. If the voltage along the length of the conductor is 1
volt, determe the Hall voltage across the width when a magnetic field of 0.02 T is applied.
(Ans. 2.7 mV)
Recap
In this course you have learnt the following
Charge carriers in semiconductors are negatively charged electrons and positively charged holes. Holes are
essentially vacancies in an otherwise filled band so that when an electron moves to such a vacancy, the
effect is equivalent to movement of a hole in reverse direction.
In the absence of an electric field, charge carriers move randomly so that their average velocity is zero.
When an electric field is applied, the positve charge carriers move in the direction of the field and the
negative charge carriers move against the field direction. This directed motion is superimposed over the
random direction and is called drift.
Drift velocity is proportional to the direction of the field, the constant of proportionality is the mobility of the
carrier.
If a magnetic field is applied on a flat strip of semiconductor in a direction perpendicular to the direction of
current flow, a voltage develops in a direction which is perpendicular to both the direction of the field and

the magnetic field. This is known as Hall voltage.


Hall voltage arises due to Lorentz force that acts on charge carriers and provides a direct means of verifying
existence of holes.

Module 6 : PHYSICS OF SEMICONDUCTOR DEVICES


Lecture 34 : Intrinsic Semiconductors
Objectives
In this course you will learn the following
Intrinsic and extrinsic semiconductors.

Fermi level in a semiconductor.

p-type and n-type semiconductors.

Compensated semiconductors.

Charge neutrality and law of mass action.


Intrinsic Semiconductors
An intrinsic semiconductor is a pure semiconductor, i.e., a sample without any impurity. At absolute zero
it is essentially an insulator, though with a much smaller band gap. However, at any finite temperature
there are some charge carriers are thermally excited, contributing to conductivity. Semiconductors such
as silicon and germanium, which belong to Group IV of the periodic table are covalently bonded with
each atom of Si(or Ge) sharing an electron with four neighbours of the same specis. A bond picture of
silicon is shown in the figure where a silicon atom and its neighbour share a pair of electrons in covalent
bonding.

Gallium belongs to Group III and bonds with arsenic which belongs to Group V to give a III-V
semiconductor. In GaAs, the bonding is partly covalent and partly ionic. Other commonly known III-V
semiconductors are GaN, GaP, InSb etc. Like the III-V compounds, Group II elements combine with
Group VI elements to give semiconductors like CdTe, CdS, ZnS etc. Several industrially useful
semiconductors are alloys such as Al

GA

As.

The number of carriers in a band at finite temperatures is given by

is the density of state and

thermal probability. If

, where

is the Fermi function which gives the

, we may ignore the term 1 in the denominator of the Fermi

function and approximate it as

Using this the density of electrons in the conduction band (

) may be written as follows.

where we have substituted

The integral

is a gamma function

whose value is

. Substituting this

value, we get for the density of electrons in the conduction band

where

One can in a similar fashion one can calculate the number density of holes,
expression

, by evaluating the

where

is the Fermi function for the occupancy of holes which is the same as

the probability that an electron state at energy

is unoccupied. For

, the density

of holes is given by

where

The following table gives generally accepted values of some of the quantities associated with the three
most common semiconductors at room temperature (300 K).

in eV

/m

Si

1.12

1.08

0.56

Ge

0.66

0.55

0.37

GaAs

1.4

0.04

0.48

/m

/m

Exercise 1
Derive expression (B).
For an intrinsic semiconductor the number of electrons in the conduction band is equal to the number of
holes in the valence band since a hole is left in the valence band only when an electron makes a
transition to the conduction band,

Using this and assuming that the effective masses of the electrons and holes are the same one gets

giving

i.e. the Fermi level lies in the middle of the forbidden gap . Note that there is no contradiction with
the fact that no state exists in the gap as

is only an energy level and not a state.

By substituting the above expression for Fermi energy in (A) or (B), we obtain an expression for the
number density of electrons or holes (

where

is the width of the gap.

Exercise 2
Derive the expression (D).
Exercise 3
For a two band model of silicon, the band gap is 1.11 eV. Taking the effective masses of electrons and
and

holes as

, calculate the intrinsic carrier concentration in silicon

at 300 K.
(Ans.

.)

Exercise 4
Show that, if the effective masses of electrons and holes are not equal, the position of the Fermi energy
for an intrinsic semiconductor is given by

Current in an intrinsic semiconductor


For semiconductors both electrons and holes contribute to electric current. Because of their opposite
charge, their contribution to the current add up. For an intrinsic semiconductor with a single valence
band and a conduction band, the current density is given by

where

and

are respectively the electron density and speed while


and

and speed. Using

and the fact that

and

are the hole density


, we have

which gives the conductivity as

Example 7
Estimate the electrical conductivity of intrinsic silicon at 300 K, given that the electron and hole
mobilities are

/V-s and

/V-s.

Solution
The conductivity arises due to both electrons and holes

The intrinsic carrier concentration

was calculated to be

at 300 K. Thus

Exercise 5
A sample of an intrinsic semiconductor has a band gap of 0.7 eV, assumed independent of temperature.
Taking

and

, find the relationship between the conductivity at 200 K and

300 K.
(Ans. ratio of conductivity = 2014.6,

eV )

Extrinsic Semiconductors
An extrinsic semiconductor is formed by adding impurities, called dopants to an intrinsic semiconductor
to modify the former's electrical properties. There are two types of such impurities - those which provide
electrons as majority carriers are known as n-type and those which provide holes as majority carriers
are known as p-type .
Using this and assuming that the effective masses of the electrons and holes are the same one gets
n- type Semiconductors
Consider a matrix of silicon where the atoms are covalently bonded.

If we add a pentavalent atom (As, P etc.) as an impurity, the dopant atom replaces a silicon atom
substitutionally. As the dopant has five electrons, only four of these can be used in forming covalent
bonds while the fifth electron is loosely bonded to the parent atom. This electron can become detached
from the dopant atom by absorbing thermal energy.

In the band picture, the energy level of the additional electron lies close to the bottom of the conduction
band. Such an energy level

is called a donor level as it can donate an electron to the empty

conduction band by thermal excitation. We may see this by assuming that the fifth electron of the donor
is orbiting around a hydrogen-like nucleus consisting of the core of the donor atom with the following
modifications made into the formula for the energy of an electron in the hydrogen atom.

permittivity of the free space

is replaced by

, the permittivity of the medium (silicon).

free electron mass being replaced by the effective mass of the donor electron.
Recalling that the energy of an electron in the the hydrogen atom is given by

where

, we need to replace

relative dielectric constant of the medium. Using

by

and
for Si and

by

, where

is the

, the free electron

mass, the ionization energy of the electron bound to the donor atom is

eV, if

the electron is in the ground state. Thus the donor energy level lies close to the bottom of the
conduction band. In case of semiconductors, the donor ionization energy is defined as the energy
required to elevate the donor electron to the conduction band.

Exercise 6
Calculate the ionization energy of a donor impurity in Ge. The effective mass of electrons is
and the dielectric constant is 16.
(6.4 meV)
p- type Semiconductors
If the Si matrix is doped with Group III impurities like boron or aluminium, it cannot provide electrons to
complete the covalent bonds. However, the impurity readily accepts an electron from a nearby Si-Si
bond to complete its own bonding scheme. A hole is thereby created which can freely propagate in the
lattice.

In the band picture, the acceptor energy level

lies close to the top of the valence band. Electrons

near the top of the valence band can be thermally excited to the acceptor level leaving holes near the
top of the valence band. In these semiconductors, known as P-type semiconductors, the primary current
is due to majority carriers which are holes.

Example 8
In an n-type semiconductor 25% of the donor atoms are ionized at 300 K. Determine the location of the
Fermi level with respect to the donor level.
Solution
As 25% of donor atoms are ionized, the occupation probability of donor level is 0.75. Thus

Solving,

eV.

Exercise 7
In a p-type semiconductor 40% of atoms are ionized at 300 K. Find the location of the Fermi level with
respect to the acceptor level.
eV)

A compensated semiconductor contains both donor and acceptor impurities. The compensation is said
to be complete if

in which case the semiconductor behaves like an intrinsic semiconductor.

Example 9
Solution
Given

we get
temperature, we get

which gives

. Using

eV

corresponding

to room

eV. Rewriting this as

. The occupation probability of the donor level is

Thus 71% of donor atoms are ionized. The Fermi level is situated 0.0236 eV below the donor level.
Condition of Charge Neutrality
In the absence of an electric field, a bulk material is charge neutral. Let
number density of electrons
number density of holes
For an intrinsic semiconductor

so that the number density of electrons may be written as

Let the density of donor atoms be denoted by

and that of acceptor atoms by

corresponding densities of ionized donors and acceptors are


neutrality condition for the bulk sample becomes

If all the donors and acceptors are ionized, then,

and

. If the

respectively, the charge

Using

, we get

Thus we get a quadratic equation for the electron density

with solution

Example 10
Pure germanium has a band gap of 0.67 eV. It is doped with

per m

of donor atoms. Find

the densities of electrons and holes at 300 K. (effective masses

Solution
For Ge, the intrinsic concentration is

Substituting given numerical values,


/m

. The density of donor atoms is

. Thus the electron density

Thus

Using

is given by

we

get,

for

the

density

.
Exercise 8
A sample of Ge at 300 K is doped with

of donor atoms and

acceptor atoms. Find the densities of electrons and holes at 300 K.


(Ans.

Fermi Energy
Let

be the Fermi level for a n-type semiconductor. The electron density is given by

of

holes

where

We may rewrite the above equation as follows. Denoting the intrinsic Fermi energy as

where

is the intrinsic electron density. In a similar way one can show that for

type impurities, the

concentration of holes is given by

where

is intrinsic hole density. Thus

This relationship is known as the Law of Mass Action .


Taking the logarithm of the equations for

and

type and p-type semiconductors are given by

, the shift in the Fermi energies due to doping for n-

Since

for an n-type material,

material,

lies above the intrinsic Fermi level, whereas, for a p-type

and the Fermi level lies below the intrinsic Fermi level.

Example 11
Silicon crystal is doped with

atoms per m

. The donor level is 0.05 eV from the edge of the

conduction band. Taking the band gap to be 1.12 eV, calculate the position of the Fermi level at 200 K.
Solution
The intrinsic carrier concentration can be obtained from the known carrier concentration in Si at 300 K.
As the carrier concentration at 300 K is

, the carrier concentration at 200 K is


. As the doping concentration is much larger

than

, we can take

. Thus

Exercise 9
Germanium has ionized acceptor density of

/m

and donor density of

/m

Taking the band gap to be 0.67 eV, calculate the equilibrium density of majority and minority carriers at
450 K and also the Fermi energy. [Hint : Using the intrinsic concentration at 300 K, find
and use the expression for
(Ans.
Recap

.]
eV)

at 450 K

In this course you have learnt the following


At very low temperatures, semiconductors are like insulators as there are no free carriers in their
conduction band. As temperature is raised, thermal excitation of carriers takes place to the conduction
band leading to non-zero conductivity. Such semiconductors are called intrinsic.
The band gap in semiconductors is much smaller than that in insulators, which facilitates thermal
excitation in the former.
For an intrinsic semiconductor, the Fermi level lies in the middle of the forbidden gap.
Electrical properties of semiconductors can be modified by introducing impurities (known as dopants).
Dopants can have excess valence electrons per atom over that of the host material. Such
semiconductors are called n-type semiconductors for which the majority carriers are electrons.
In a p-type semiconductor, the dopant atoms have a deficit of valence electrons and in such cases the
majority charge carriers are holes.
A compensated semiconductor is one in which both types of dopants exist.

Module 6 : PHYSICS OF SEMICONDUCTOR DEVICES


Lecture 35 : Photoconductivity
Objectives
In this course you will learn the following
Photoconductivity.

Diffusion and drift.

Einstein relations.
Photoconductivity
A consequence of small band gap (

) in semiconductors is that it is possible to generate additional

carriers by illuminating a sample of semiconductor by a light of frequency greater than

. This leads to

an increased conductivity in the sample and the phenomenon is known as photoconductivity . The effect is
not very pronounced at high temperatures except when the illumination is by an intense beam of light. At
low temperatures, illumination results in excitation of localized carriers to conduction or valence band.
Consider a thin slab of semiconductor which is illuminated by a beam of light propagating along the direction
of its length (x-direction). Let

be the radiation intensity (in watts/m

) at a position

from one end of

the semiconductor. If
absorption coefficient per unit length, the power absorbed per unit length is
. The change in the intensity with distance along the sample length is given by

which has solution

If we define

as the quantum efficiency , i.e. the fraction of absorbed photons that produce electron-hole

pairs, the number of pairs produced per unit time is given by

In principle, the process of illumination will lead to a continued increase in the number of carriers as the
amount of energy absorbed (and hence

and

) will increase linearly with time. However, the

excited pairs have a finite life time (


to
s). This results in recombination of the pairs. The
relevant life time is that of minority carriers as a pair is required in the process. Recombination ensures that
the number of excess carriers does not increase indefinitely but saturates.

Consider an n-type semiconductor. If the recombination life time for the minority carriers is

, the rate of

change of carrier concentration is given by

Under steady state condition

, which gives

This excess hole density leads to an additional conductivity

Diffusion
Diffusion is the process by which particles move from a region of higher concentration to a region of lower
concentration. If the process is left undisturbed, it would result in uniform density of particles in the medium.
The following points are to be noted about diffusion :
Diffusive motion takes place because of random thermal motion.

Diffusion takes place irrespective of whether the particles are charged or not.
Diffusion of particles takes place along the concentration gradient, i.e. from a region of higher concentration
to that of lower concentration.
If the particles of the medium are charged carriers, like electrons and holes in a semiconductor, the diffusion
along the concentration gradient results in a current, known as diffusion current. Unlike the drift current
, the diffusion current depends on gradient of concentration rather than on the concentration itself. If the
carriers are electrons, the diffusion current is proportional to

, where

is the electron density.

Likewise, the hole diffusion current is proportional to

Consider the diffusive motion of holes in one dimension. If


, i.e., if the concentration is increasing where

is the concentration at
is the intrinsic Fermi level and

Fermi level in the presence of acceptor impurities (we have dropped the redundant superscript

Fermi level in a material remains uniform inside a material, i.e.

, and
is the

). Thus

Thus

Thus diffusion takes place when there exists an intrinsic Fermi level gradient. In order to evaluate the
gradient of

, consider the valence band. The kinetic energy of the holes is the difference between the

energy of the top of the valence band and the total energy of the holes. Thus the energy at the top of the
valence band is the potential energy of the holes. Similarly, the energy at the bottom of the conduction band
is the potential energy of the electrons. As the intrinsic Fermi energy depends on these two energy levels,
temperature and other constants, the change in the electron or hole energy is also given by the difference
between the intrinsic Fermi energy

The electric field,

and the (uniform) Fermi energy

is then given by

Assume quasi-charge neutrality for which the concentration of holes is equal to the concentration of

acceptors,

. Using one dimensional form of (B),

which gives

Substituting the above in (A) and cancelling common terms, we get

A similar relation can be established for electrons. The relations

Are known as bf Einstein relations between mobility and diffusion constant.


Example 12
Electron concentration in a semiconductor varies linearly from a value

per m

per m
over a distance of 0.1 cm. If the diffusion constant is 0.025 m
density in the sample.

to

/s, find the diffusion current

Solution

Thus the diffusion current density

Exercise 1
An

n-

type

semiconductor
per m

has

graded

impurity

. Find the electric field at

concentration

along

the

x-axis

given

by

at room temperature

(Ans. 2.6 V)
Exercise 2
i)For the semiconductor in the above exercise, calculate the diffusion coefficient at 300 K if the electron
mobility is 1500 cm

/V-s. (ii) Calculate the diffusion current density. Explain the direction of diffusion

current.(Ans. (i)

(ii) 624 A/m

(Hint : Use Einstein relation to find

Recap
In this course you have learnt the following
As the band gap in semiconductors is less than the energy of optical photons, carriers can move across the
gap when light is incident on semiconductor. This is known as photoconductivity.
When there is a gradient of concentration, charge carriers move from a region of higher concentration to
that of lower concentration. The process is known as diffusion.
Einstein relations provide connectivity between mobility of carriers and temperature for diffusive motion.

You might also like